JURISDICTION-DIGESTED-OCHOA.docx

94
JURISDICTION CASE DIGEST TROPANG POTCHI BELGICA VS OCHOA FACTS: In the Philippines, the “pork barrel” (a term of American-English origin) has been commonly referred to as lump-sum, discretionary funds of Members of the Legislature (“Congressional Pork Barrel”). However, it has also come to refer to certain funds to the Executive. The “Congressional Pork Barrel” can be traced from Act 3044 (Public Works Act of 1922), the Support for Local Development Projects during the Marcos period, the Mindanao Development Fund and Visayas Development Fund and later the Countrywide Development Fund (CDF) under the Corazon Aquino presidency, and the Priority Development Assistance Fund under the Joseph Estrada administration, as continued by the Gloria-Macapagal Arroyo and the present Benigno Aquino III administrations. The “Presidential Pork Barrel” questioned by the petitioners include the Malampaya Fund and the Presidential Social Fund. The Malampaya Fund was created as a special fund under Section 8, Presidential Decree (PD) 910 by then- President Ferdinand Marcos to help intensify, strengthen, and consolidate government efforts relating to the exploration, exploitation, and development of indigenous energy resources vital to economic growth. The Presidential Social Fund was created under Section 12, Title IV, PD 1869 (1983) or the Charter of the Philippine Amusement and Gaming Corporation (PAGCOR), as amended by PD 1993 issued in 1985. The Presidential Social Fund has been described as a special funding facility managed and administered by the Presidential Management Staff through which the President provides direct assistance to priority programs and projects not funded under the regular budget. It is sourced from the share of the government in the aggregate gross earnings of PAGCOR. Over the years, “pork” funds have increased tremendously. In 1996, an anonymous source later identified as former Marikina City Romeo Candazo revealed that huge sums of government money went into the pockets of legislators as kickbacks. In 2004, several citizens sought the nullification of the PDAF as enacted in the 2004 General Appropriations Act for being unconstitutional, but the Supreme Court dismissed the petition. In July 2013, the National Bureau of Investigation (NBI) began its probe into allegations that “the government has been defrauded of some P10 Billion over the past 10 years by a syndicate using funds from the pork barrel of lawmakers and various government agencies for scores of ghost projects.” The investigation was spawned by sworn affidavits of six whistle-blowers who declared that JLN Corporation – “JLN” standing for Janet Lim Napoles – had swindled billions of pesos from the public coffers for “ghost projects” using no fewer than 20 dummy non-government organizations for an entire decade. In August 2013, the Commission on Audit (CoA) released the results of a three-year audit investigation covering the use of legislators’ PDAF from 2007 to 2009, or during the last three (3) years of the Arroyo administration. As for the “Presidential Pork Barrel”, whistle-blowers alleged that “[a]t least P900 Million from royalties in the operation of the Malampaya gas project intended for agrarian reform beneficiaries has gone into a dummy [NGO].” ISSUES: Whether or not (WON) the issues raised in the consolidated petitions involve an actual and justiciable controversy HELD: YES. There exists an actual and justiciable controversy in these cases. The requirement of contrariety of legal rights is clearly satisfied by the antagonistic positions of the parties on the constitutionality of the “Pork Barrel System.” Also, the questions in these consolidated cases are ripe for adjudication since the challenged funds and the provisions allowing for their utilization – such as the 2013 GAA for the PDAF, PD 910 for the Malampaya Funds and PD 1869, as amended by PD 1993, for the Presidential Social Fund – are currently existing and operational; hence, there exists an immediate or threatened injury to petitioners as a result of the unconstitutional use of these public funds. Question involving the constitutionality or validity of a law or governmental act may be heard and decided by the Court unless there is compliance with the legal requisites for judicial inquiry, namely: (a) there must be an actual case or controversy calling for the exercise of judicial power; (b) the person challenging the act must have the standing to question the validity of the subject act or issuance; (c) the question of constitutionality must be raised at the earliest opportunity; and (d) the issue of constitutionality must be the very lismota of the case. As for the PDAF, the Court dispelled the notion that the issues related thereto had been rendered moot and academic by the reforms undertaken by respondents. A case becomes moot when there is no more actual controversy between the parties or no useful purpose can be served in passing upon the merits. The respondents’ proposed line-item budgeting scheme would not terminate the controversy nor diminish the useful purpose for its resolution since said reform is geared towards the 2014 budget, and not the 2013 PDAF Article which, being a distinct subject matter, remains legally effective and existing. Neither will the President’s declaration that he had already “abolished the PDAF” render the issues on PDAF moot precisely because the Executive branch of government has no constitutional authority to nullify or annul its legal existence. FRANCISCO VS. HOUSE OF REPRESENTATIVES FACTS: On July 22, 2002, the House of Representatives adopted a Resolution, sponsored by Representative Felix William D. Fuentebella, which directed the Committee on Justice "to conduct an investigation, in aid of legislation, on the manner of disbursements and expenditures by the Chief Justice of the Supreme Court of the Judiciary Development Fund (JDF)." On June 2, 2003, former President Joseph E. 1 | Page

Transcript of JURISDICTION-DIGESTED-OCHOA.docx

Page 1: JURISDICTION-DIGESTED-OCHOA.docx

JURISDICTION CASE DIGEST TROPANG POTCHI

BELGICA VS OCHOA

FACTS: In the Philippines, the “pork barrel” (a term of American-English origin) has been commonly referred to as lump-sum, discretionary funds of Members of the Legislature (“Congressional Pork Barrel”). However, it has also come to refer to certain funds to the Executive. The “Congressional Pork Barrel” can be traced from Act 3044 (Public Works Act of 1922), the Support for Local Development Projects during the Marcos period, the Mindanao Development Fund and Visayas Development Fund and later the Countrywide Development Fund (CDF) under the Corazon Aquino presidency, and the Priority Development Assistance Fund under the Joseph Estrada administration, as continued by the Gloria-Macapagal Arroyo and the present Benigno Aquino III administrations.

The “Presidential Pork Barrel” questioned by the petitioners include the Malampaya Fund and the Presidential Social Fund. The Malampaya Fund was created as a special fund under Section 8, Presidential Decree (PD) 910 by then-President Ferdinand Marcos to help intensify, strengthen, and consolidate government efforts relating to the exploration, exploitation, and development of indigenous energy resources vital to economic growth. The Presidential Social Fund was created under Section 12, Title IV, PD 1869 (1983) or the Charter of the Philippine Amusement and Gaming Corporation (PAGCOR), as amended by PD 1993 issued in 1985. The Presidential Social Fund has been described as a special funding facility managed and administered by the Presidential Management Staff through which the President provides direct assistance to priority programs and projects not funded under the regular budget. It is sourced from the share of the government in the aggregate gross earnings of PAGCOR.

Over the years, “pork” funds have increased tremendously. In 1996, an anonymous source later identified as former Marikina City Romeo Candazo revealed that huge sums of government money went into the pockets of legislators as kickbacks. In 2004, several citizens sought the nullification of the PDAF as enacted in the 2004 General Appropriations Act for being unconstitutional, but the Supreme Court dismissed the petition. In July 2013, the National Bureau of Investigation (NBI) began its probe into allegations that “the government has been defrauded of some P10 Billion over the past 10 years by a syndicate using funds from the pork barrel of lawmakers and various government agencies for scores of ghost projects.” The investigation was spawned by sworn affidavits of six whistle-blowers who declared that JLN Corporation – “JLN” standing for Janet Lim Napoles – had swindled billions of pesos from the public coffers for “ghost projects” using no fewer than 20 dummy non-government organizations for an entire decade. In August 2013, the Commission on Audit (CoA) released the results of a three-year audit investigation covering the use of legislators’ PDAF from 2007 to 2009, or during the last three (3) years of the Arroyo administration.

As for the “Presidential Pork Barrel”, whistle-blowers alleged that “[a]t least P900 Million from royalties in the operation of the Malampaya gas project intended for agrarian reform beneficiaries has gone into a dummy [NGO].”

ISSUES:

Whether or not (WON) the issues raised in the consolidated petitions involve an actual and justiciable controversy

HELD:

YES. There exists an actual and justiciable controversy in these cases. The requirement of contrariety of legal rights is clearly satisfied by the antagonistic positions of the parties on the constitutionality of the “Pork Barrel System.” Also, the questions in these consolidated cases are ripe for adjudication since the challenged funds and the provisions allowing for their utilization – such as the 2013 GAA for the PDAF, PD 910 for the Malampaya Funds and PD 1869, as amended by PD 1993, for the Presidential Social Fund – are currently existing and operational; hence, there exists an immediate or threatened injury to petitioners as a result of the unconstitutional use of these public funds.

Question involving the constitutionality or validity of a law or governmental act may be heard and decided by the Court unless there is compliance with the legal requisites for judicial inquiry, namely: (a) there must be an actual case or controversy calling for the exercise of judicial power; (b) the person challenging the act must have the standing to question the validity of the subject act or issuance; (c) the question of constitutionality must be raised at the earliest opportunity; and (d) the issue of constitutionality must be the very lismota of the case.

As for the PDAF, the Court dispelled the notion that the issues related thereto had been rendered moot and academic by the reforms undertaken by respondents. A case becomes moot when there is no more actual controversy between the parties or no useful purpose can be served in passing upon the merits. The respondents’ proposed line-item budgeting scheme would not terminate the controversy nor diminish the useful purpose for its resolution since said reform is geared towards the 2014 budget, and not the 2013 PDAF Article which, being a distinct subject matter, remains legally effective and existing. Neither will the President’s declaration that he had already “abolished the PDAF” render the issues on PDAF moot precisely because the Executive branch of government has no constitutional authority to nullify or annul its legal existence.

FRANCISCO VS. HOUSE OF REPRESENTATIVES

FACTS:

On July 22, 2002, the House of Representatives adopted a Resolution, sponsored by Representative Felix William D. Fuentebella, which directed the Committee on Justice "to conduct an investigation, in aid of legislation, on the manner of disbursements and expenditures by the Chief Justice of the Supreme Court of the Judiciary Development Fund (JDF)." On June 2, 2003, former President Joseph E. Estrada filed an impeachment complaint against Chief Justice Hilario G. Davide Jr. and seven Associate Justices of this Court for "culpable violation of the Constitution, betrayal of the public trust and other

high crimes." The complaint was endorsed by Representatives Rolex T. Suplico, Ronaldo B. Zamora and DidagenPiangDilangalen, and was referred to the House Committee. The House Committee on Justice ruled on October 13, 2003 that the first impeachment complaint was "sufficient in form," but voted to dismiss the same on October 22, 2003 for being insufficient in substance. To date, the Committee Report to this effect has not yet been sent to the House in plenary in accordance with the said Section 3(2) of Article XI of the Constitution. Four months and three weeks since the filing on June 2, 2003 of the first complaint or on October 23, 2003, a day after the House Committee on Justice voted to dismiss it, the second impeachment complaint was filed with the Secretary General of the House by Representatives Gilberto C. Teodoro, Jr. and Felix William B. Fuentebella against Chief Justice Hilario G. Davide, Jr., founded on the alleged results of the legislative inquiry initiated by above-mentioned House Resolution. This second impeachment complaint was accompanied by a "Resolution of Endorsement/Impeachment" signed by at least one-third (1/3) of all the Members of the House of Representatives.

ISSUES:

Whether the resolution thereof is a political question – has resulted in a political crisis.

HELD:

From the foregoing record of the proceedings of the 1986 Constitutional Commission, it is clear that judicial power is not only a power; it is also a duty, a duty which cannot be abdicated by the mere specter of this creature called the political question doctrine. Chief Justice Concepcion hastened to clarify, however, that Section 1, Article VIII was not intended to do away with "truly political questions." From this clarification it is gathered that there are two species of political questions: (1) "truly political questions" and (2) those which "are not truly political questions." Truly political questions are thus beyond judicial review, the reason for respect of the doctrine of separation of powers to be maintained. On the other hand, by virtue of Section 1, Article VIII of the Constitution, courts can review questions which are not truly political in nature.

ANGARA VS ELECTORAL COMMISSION

FACTS: In the elections of Sept 17, 1935, Angara, and the respondents, Pedro Ynsua et al. were candidates voted for the position of member of the National Assembly for the first district of the Province of Tayabas. On Oct 7, 1935, Angara was proclaimed as member-elect of the NA for the said district. On November 15, 1935, he took his oath of office. On Dec 3, 1935, the NA in session assembled, passed Resolution No. 8 confirming the election of the members of the National Assembly against whom no protest had thus far been filed. On Dec 8, 1935, Ynsua, filed before the Electoral Commission a “Motion of Protest” against the election of Angara. On Dec 9, 1935, the EC adopted a resolution, par. 6 of which fixed said date as the last day for the filing of protests against the election, returns and qualifications of members of the NA, notwithstanding the previous confirmation made by the NA. Angara filed a Motion to Dismiss arguing that by

1 | P a g e

Page 2: JURISDICTION-DIGESTED-OCHOA.docx

JURISDICTION CASE DIGEST TROPANG POTCHI

virtue of the NA proclamation, Ynsua can no longer protest. Ynsua argued back by claiming that EC proclamation governs and that the EC can take cognizance of the election protest and that the EC cannot be subject to a writ of prohibition from the SC.

ISSUES: Whether or not the SC has jurisdiction over such matter.

Whether or not EC acted without or in excess of jurisdiction in taking cognizance of the election protest.

HELD: The SC ruled in favor of Angara. The SC emphasized that in cases of conflict between the several departments and among the agencies thereof, the judiciary, with the SC as the final arbiter, is the only constitutional mechanism devised finally to resolve the conflict and allocate constitutional boundaries.

That judicial supremacy is but the power of judicial review in actual and appropriate cases and controversies, and is the power and duty to see that no one branch or agency of the government transcends the Constitution, which is the source of all authority.

That the Electoral Commission is an independent constitutional creation with specific powers and functions to execute and perform, closer for purposes of classification to the legislative than to any of the other two departments of the government.

That the Electoral Commission is the sole judge of all contests relating to the election, returns and qualifications of members of the National Assembly.

TOLENTINO VS COMELEC

FACTS:

Petitioners assailed the manner by which the simultaneous regular and special elections of 2001 were conducted by the COMELEC.Petitioners contend that, if held simultaneously, a special and a regular election must be distinguished in the documentation as well as in the canvassing of their results. Thirteen senators were proclaimed from the said election with the 13th placer to serve that of the remaining term of Sen. Guingona, who vacated a seat in the senate.

Petitioners sought for the nullification of the special election and, consequently, the declaration of the 13th elected senator.

Issue:

1Whether or not Court had jurisdiction.

2Whether or not the petition was moot.

3Whether or not petioners had locus standi.

4Whether a Special Election for a Single, Three-Year Term

Senatorial Seat was Validly Held on 14 May 2001

RULING:

On the issue of jurisdiction, Court had jurisdiction because what petitioners were questioning was the validity of the special election on 14 May 2001 in which Honasan was elected and not to determine Honasan’s right in the exercise of his office as Senator proper under a quo warranto.

On the issue of mootness, it was held that courts will decide a question otherwise moot if it is capable of repetition yet evading review.

On the issue of locus standi, the court had relaxed the requirement on standing and exercised our discretion to give due course to voters’ suits involving the right of suffrage, considering that the issue raised in this petition is likely to arise again

On the VAlidity of the Election, the Court held that the May 14, 2001 Election was valid.

The Court held that COMELEC’s Failure to Give Notice of the Time of the Special Election as required under RA 6645, as amended, did Not Negate the Calling of such Election. Section 2 of R.A. No. 6645 itself provides that in case of vacancy in the Senate, the special election to fill such vacancy shall be held simultaneously with the next succeeding regular election. The law charges the voters with knowledge of this statutory notice and COMELEC’s failure to give the additional notice did not negate the calling of such special election, much less invalidate it. Further, there was No Proof that COMELEC’s Failure to Give Notice of the Office to be Filled and the Manner of Determining the Winner in the Special Election Misled Voters. IT could not be said that the voters were not informed since there had been other accessible information resources. Finally, the Court held that unless there had been a patent showing of grave abuse of discretion, the Court will not interfere with the affairs and conduct of the Comelec.

JAVELLANA VS. EXECUTIVE SECRETARY

FACTS: In 1973, Marcos ordered the immediate implementation of the new 1973 Constitution. Javellana, a Filipino and a registered voter sought to enjoin the Exec Sec and other cabinet secretaries from implementing the said constitution. Javellana averred that the said constitution is void because the same was initiated by the president. He argued that the President is w/o power to proclaim the ratification by the

Filipino people of the proposed constitution. Further, the election held to ratify such constitution is not a free election there being intimidation and fraud.

ISSUE: Whether or not the SC must give due course to the petition.

HELD: The SC ruled that they cannot rule upon the case at bar. Majority of the SC justices expressed the view that they were concluded by the ascertainment made by the president of the Philippines, in the exercise of his political prerogatives. Further, there being no competent evidence to show such fraud and intimidation during the election, it is to be assumed that the people had acquiesced in or accepted the 1973 Constitution. The question of the validity of the 1973 Constitution is a political question which was left to the people in their sovereign capacity to answer. Their ratification of the same had shown such acquiescence.

SALVADORA OCAMPO, ET AL., plaintiffs-appellees, vs. TOMAS CABAÑGIS, defendant-appellant.

FACTS: From the decision held by the SC. The defendant is absolved from the complaint without special finding as to costs, and record it to the original court. However, two of the four justices who signed the decision are no longer members of the court.

The appellees now seek the cancellation and annulment of the entry of judgment and the recall of the remittitur and the record of the case to this court. The motion is made upon the theory that no final judgment has ever been entered, and that by reason of the changes in the personnel of the court the more extensive opinion which was contemplated can not now be filed.

Section 15 of Act. No. 136 provides that "in the determination of causes all decision of the Supreme Court shall be given in writing, signed by the judges concurring in the decision, and the grounds of the decision shall be stated as briefly as may be consistent with clearness."

The decision of December 26, 1908, was in writing, and was signed by the four justices who concurred therein, but no grounds are stated for the decision.

ISSUE: WON non compliance will affect the decision

HELD: No. A strict and literal compliance with this statute would often render it impossible for the court to decide a case. The Act declares the manner in which the Supreme Court shall perform the strictly judicial act of giving final expression to its decision, but it does not say that the failure to comply therewith shall render the decision ineffective. The direction is as to a matter which is not of the essence of the thing to be done, and there is nothing to suggest that the Legislature intended that strict compliance therewith is not necessary to the validity of the proceedings.

2 | P a g e

Page 3: JURISDICTION-DIGESTED-OCHOA.docx

JURISDICTION CASE DIGEST TROPANG POTCHI

Lansang v. Garcia

G.R. No. L-33964 and G.R. No. L-33965 and G.R. No. L-33973 | 1971-12-11

Subject: Suspension of the privilege of the writ of habeas corpus (constitutional limitations); Judicial branch’s power to review factual basis of suspension by Executive branch

Facs: In the evening of August 21, 1971, while the Liberal Party was holding a public meeting at Plaza Miranda, two hand grenades were thrown at the platform where the candidates and other persons were, killing and injuring several persons.

Two days later, President Marcos announced the issuance of Proclamation No. 889, declaring that lawless elements have created a state of lawlessness and disorder affecting public safety and the security of the State; and thereby the privilege of the writ of habeas corpus is suspended, for ‘persons detained for the crimes of insurrection or rebellion’. Presently, petitions for writs of habeas corpus were filed by herein petitioners who have been arrested without a warrant and then detained. They likewise assail Proclamation No. 889 as unconstitutional.

The respondent argues that petitioners had been detained ‘on reasonable belief’ that they had participated in ‘the crime of insurrection or rebellion’ and that their continued detention is justified due to the suspension of the privilege of the writ of habeas corpus pursuant to Proclamation No. 889.

It is likewise alleged that the issuance of the proclamation partakes of the nature of political question which cannot be the subject of judicial inquiry and that in making said declaration.

ISSUE: WON the proclamation is constitutional

Held: Yes. In testing the validity of acts of Congress and of the Executive be, the proper standard is not correctness, but arbitrariness. This means that judicial inquiry into the basis of the questioned proclamation can go no further than to satisfy the Court not that the President's decision is correct but that in suspending the writ, the President did not act arbitrarily.

Under the principle of separation of powers and the system of checks and balances, the function of the Court is merely to check, not to supplant the Executive, or to ascertain

merely whether he has gone beyond the constitutional limits of his jurisdiction, not to exercise the power vested in the President or to determine the wisdom of his act.

The President did not act arbitrarily in issuing Proclamation No. 889. Accordingly, the same is not unconstitutional. The President acted on relevant facts gathered thru various intelligence agents of our government.

Vinuya vs Romulo (2010)

G.R. No. 162230 | 2010-04-28

Facts: Members of the MALAYA LOLAS, a non-stock, non-profit organization, established for the purpose of providing aid to the victims of rape by Japanese military forces in the Philippines during the Second World War, claim that since 1998, they have approached the Executive Department through the Department of Justice (DOJ), Department of Foreign Affairs (DFA), and Office of the Solicitor General (OSG), requesting assistance in filing a claim against the Japanese officials and military officers who ordered the establishment of the "comfort women" stations in the Philippines. However, officials of the Executive Department declined to assist the petitioners, and took the position that the individual claims of the comfort women for compensation had already been fully satisfied by Japan’s compliance with the Peace Treaty between the Philippines and Japan.

ISSUE: WON the Executive Department committed grave abuse of discretion in not espousing petitioners’ claims for official apology and other forms of reparations against Japan.

HELD: No. In this case, the Executive Department has already decided that it is to the best interest of the country to waive all claims of its nationals for reparations against Japan in the Treaty of Peace of 1951. The wisdom of such decision is not for the courts to question. Neither could petitioners herein assail the said determination by the Executive Department via the instant petition for certiorari.

Political questions refer "to those questions which, under the Constitution, are to be decided by the peoplein their sovereign capacity, or in regard to which full discretionary authority has been delegated to the legislative or executive branch of the government. It is concerned with issues dependent upon the wisdom, not legality of a particular measure."

The Decisions relating to foreign policy are delicate, complex, and involve large elements of prophecy. They are and should be undertaken only by those directly responsible to the people whose welfare they advance or imperil. They are decisions of a kind for which the Judiciary has neither aptitude, facilities nor responsibility.

MANUEL MAMBA V. EDGAR LARA

G.R. No. 165109

Facts: The Sangguniang Panlalawigan of Cagayan passed several resolutions authorizing Gov. Edgar Lara to negotiate, sign and execute contracts or agreements for the issuance and flotation of bonds to fun the priority projects of the governor and for the construction and development of a New Cagayan Town Center subject to the approval and ratification of by the Sangguniang Panlalawigan. Subsequently, the planning, design, construction and site development of the project was awarded to Asset Builders Corporation.

Petitioners Manuel Mamba, Raymund Guzman and Leonidaz Fausto, a Representative of Cagayan and members of the Sangguniang Panlalawigan, respectively, filed a petition for Annulment of Contracts entered into by Gov. Lara in connection with the New Cagayan Town Center project.

The Court dismissed the petition for lack of course of action, stating among others, that petitioners did not have the locus standi to file the present case as they are not parties to the questioned contract.

Issue: Whether or not it is a political issue

Ruling: No. In filing the instant case before the RTC, petitioners seek to restrain public respondents from implementing the bond flotation and to declare null and void all contracts related to the bond flotation and construction of the town center. In the petition before the RTC, they alleged grave abuse of discretion and clear violations of law by public respondents. They put in issue the overpriced construction of the town center; the grossly disadvantageous bond flotation; the irrevocable assignment of the provincial government's annual regular income, including the IRA, to respondent RCBC to cover and secure the payment of the bonds floated; and the lack of consultation and discussion with the community regarding the proposed project, as well as a proper and legitimate bidding for the construction of the town center.

Obviously, the issues raised in the petition do not refer to the wisdom but to the legality of the acts complained of. Thus, we find the instant controversy within the ambit of judicial review. Besides, even if the issues were political in nature, it would still come within our powers of review under the expanded jurisdiction conferred upon us by Section 1, Article VIII of the Constitution, which includes the authority to determine whether grave abuse of discretion amounting to excess or lack of jurisdiction has been committed by any branch or instrumentality of the government.

3 | P a g e

Page 4: JURISDICTION-DIGESTED-OCHOA.docx

JURISDICTION CASE DIGEST TROPANG POTCHI

FORTUN v. MACAPAGAL-ARROYO

GR no. 190293

FACTS: On November 23, 2009 heavily armed men, believed led by the ruling Ampatuan

family, gunned down and buried under shoveled dirt 57 innocent civilians on a highway

in Maguindanao. In response to this carnage, on November 24 President Arroyo issued

Presidential Proclamation 1946, declaring a state of emergency in Maguindanao, Sultan

Kudarat, and Cotabato City to prevent and suppress similar lawless violence in Central

Mindanao. In the instance, President Arroyo issued Presidential Proclamation 1959

declaring martial law and suspending the privilege of the writ of habeas corpus in that

province except for identified areas of the Moro Islamic Liberation Front.

Two days later or on December 6, 2009 President Arroyo submitted her

report to Congress in accordance with Section 18, Article VII of the 1987 Constitution

which required her, within 48 hours from the proclamation of martial law or the

suspension of the privilege of the writ of habeas corpus, to submit to that body a report

in person or in writing of her action.

On December 9, 2009 Congress, in joint session, convened pursuant to

Section 18, Article VII of the 1987 Constitution to review the validity of the President’s

action. But, two days later or on December 12 before Congress could act, the President

issued Presidential Proclamation 1963, lifting martial law and restoring the privilege of

the writ of habeas corpus in Maguindanao.

ISSUE: WON is it a Political Question

HELD: No. President Arroyo withdrew Proclamation 1959 before the joint houses of

Congress, which had in fact convened, could act on the same. Consequently, the

petitions in these cases have become moot and the Court has nothing to review. The

lifting of martial law and restoration of the privilege of the writ of habeas corpus in

Maguindanao was a supervening event that obliterated any justiciable controversy.

It is evident that under the 1987 Constitution the President and the Congress act in

tandem in exercising the power to proclaim martial law or suspend the privilege of the

writ of habeas corpus. They exercise the power, not only sequentially, but in a sense

jointly since, after the President has initiated the proclamation or the suspension, only

the Congress can maintain the same based on its own evaluation of the situation on the

ground, a power that the President does not have.

Consequently, although the Constitution reserves to the Supreme Court the power

to review the sufficiency of the factual basis of the proclamation or suspension in a

proper suit, it is implicit that the Court must allow Congress to exercise its own review

powers, which is automatic rather than initiated. Only when Congress defaults in its

express duty to defend the Constitution through such review should the Supreme Court

step in as its final rampart. The constitutional validity of the President’s proclamation of

martial law or suspension of the writ of habeas corpus is first a political question in the

hands of Congress before it becomes a justiciable one in the hands of the Court.

Ting HovsTengGuiGR No. 130115 July 16, 2008

Facts:

Felix Ting Ho, Jr., Merla Ting Ho Braden, Juana Ting Ho and Lydia Ting HoBelenzo against their brother, respondent Vicente TengGui.   The controversy revolves around a parcel of land, and the improvements which should form part of the estate of their deceased father, Felix Ting Ho, and should be partitioned equally among each of the siblings. Petitioners alleged that their father Felix Ting Ho died intestate on June 26, 1970, and left upon his death an estate. According to petitioners, the said lot and properties were titled and tax declared under trust in the name of respondent Vicente TengGui for the benefit of the deceased Felix Ting Ho who, being a Chinese citizen, was then disqualified to own public lands in thePhilippines; and that upon the death of Felix Ting Ho, the respondent took possession of the same for his own exclusive use and benefit to their exclusion and prejudice.

Issue:

Whether or not the sale was void

Held:

No, the sale was not void. Article 1471 of the Civil Code has provided that if the price is simulated, the sale is void, but the act may be shown to have been in reality a donatin, or some other act or contract. The sale in this case, was however valid because the sale was in fact a

donation. The law requires positive proof of the simulation of the price of the sale. But since the finding was based on a mere assumption, the price has not been proven to be a simulation.

This is because the Court of law will prevail over court of equity. Under Section 1 of the 1935 Constitution: 

Section 1. All agricultural timber, and mineral lands of the public domain, waters, minerals, coal, petroleum, and other mineral oils, all forces of potential energy and other natural resources of the Philippines belong to the State, and their disposition, exploitation, development, or utilization shall be limited to citizens of the Philippines or to corporations or associations at least sixty per centum of the capital of which is owned by such citizens, subject to any existing right, grant, lease, or concession at the time of the inauguration of the Government established under this Constitution… (Emphasis supplied)

 The fundamental law is very clear in this aspect wherein it states that the right to acquire lands of the public domain is reserved for Filipino citizens or corporations at least sixty percent of the capital of which is owned by Filipinos. Therefore since the father of petitioners and respondent was a Chinese citizen; he was disqualified from acquiring and owning real property in the Philippines. In fact, he was only occupying the subject lot by virtue of the permission granted him by the then U.S. Naval Reservation Office of Olongapo, Zambales. As correctly found by the CA, the deceased Felix Ting Ho was never the owner of the subject lot in light of the constitutional proscription and the respondent did not at any instance act as the dummy of his father.Under the law, a certificate of title issued pursuant to any grant or patent involving public land is as conclusive and indefeasible as any other certificate of title issued to private lands in the ordinary or cadastral registration proceeding. The effect of the registration of a patent and the issuance of a certificate of title to the patentee is to vest in him an incontestable title to the land, in the same manner as if ownership had been determined by final decree of the court, and the title so issued is absolutely conclusive and indisputable, and is not subject to collateral attack.1[17]

Nonetheless, petitioners invoke equity considerations and claim that the ruling of the RTC that an implied trust was created between respondent and their father with respect to the subject lot should be upheld. This contention must fail because the prohibition against an alien from owning lands of the public domain is absolute and not even an implied trust can be permitted to arise on equity considerations.

14 | P a g e

Page 5: JURISDICTION-DIGESTED-OCHOA.docx

JURISDICTION CASE DIGEST TROPANG POTCHIMuller V. Muller

G.R No. 149615, DAugust 29, 2006

Petitioners: ELENA BUENAVENTURA MULLERRespondents: HELMUT MULLER

FACTS:

Petitioner Elena Buenaventura Muller and respondent Helmut Muller were married in Hamburg, Germany on September 22, 1989. The couple resided in Germany at a house owned by respondent’s parents but decided to move and reside permanently in the Philippines in 1992. By this time, respondent had inherited the house in Germany from his parents which he sold and used the proceeds for the purchase of a parcel of land in Antipolo, Rizal at the cost of P528,000.00 and the construction of a house amounting to P2,300,000.00. The Antipolo property was registered in the name of petitioner under Transfer Certificate of Title No. 219438 5 of the Register of Deeds of Marikina, Metro Manila.

Due to incompatibilities and respondent’s alleged womanizing, drinking, and maltreatment, the spouses eventually separated. On September 26, 1994, respondent filed a petition for separation of properties before the Regional Trial Court of Quezon City.

On August 12, 1996, the trial court rendered a decision which terminated the regime of absolute community of property between the petitioner and respondent. It also decreed the separation of properties between them and ordered the equal partition of personal properties located within the country, excluding those acquired by gratuitous title during the marriage. With regard to the Antipolo property, the court held that it was acquired using paraphernal funds of the respondent. However, it ruled that respondent cannot recover his funds because the property was purchased in violation of Section 7, Article XII of the Constitution.

Respondent appealed to the Court of Appeals which rendered the assailed decision modifying the trial court’s Decision. It held that respondent merely prayed for reimbursement for the purchase of the Antipolo property, and not acquisition or transfer of ownership to him. It also considered petitioner’s ownership over the property in trust for the respondent. As regards the house, the Court of Appeals ruled that there is nothing in the Constitution which prohibits respondent from acquiring the same.

ISSUE:

Whether or not the respondent is entitled to reimbursement on the ground of equity.

HELD:

No because the respondent is aware of the constitutional prohibition regarding the disqualification of aliens in acquiring lands of the public domain and private lands. He declared that he had the Antipolo property titled in the name of petitioner because of the said prohibition. Thus, his attempt at subsequently asserting or claiming a right on the said property cannot be sustained.

 The Court of Appeals erred in holding that an implied trust was created and resulted by operation of law in view of petitioner’s marriage to respondent. Save for the exception provided in cases of hereditary succession, respondent’s disqualification from owning lands in the Philippines is absolute. Not even an ownership in trust is allowed. Besides, where the purchase is made in violation of an existing statute and in evasion of its express provision, no trust can result in favor of the party who is guilty of the fraud.2[13] To hold otherwise would allow circumvention of the constitutional prohibition.

 Invoking the principle that a court is not only a court of law but also a court of equity, is likewise misplaced. It has been held that equity as a rule will follow the law and will not permit that to be done indirectly which, because of public policy, cannot be done directly.3[14] He who seeks equity must do equity, and he who comes into equity must come with clean hands. The latter is a frequently stated maxim which is also expressed in the principle that he who has done inequity shall not have equity. It signifies that a litigant may be denied relief by a court of equity on the ground that his conduct has been inequitable, unfair and dishonest, or fraudulent, or deceitful as to the controversy in issue.

 Thus, in the instant case, respondent cannot seek reimbursement on the ground of equity where it is clear that he willingly and knowingly bought the property despite the constitutional prohibition.

 Further, the distinction made between the transfer of ownership as opposed to recovery of funds is a futile exercise on respondent’s part. To allow reimbursement would in effect permit respondent to enjoy the fruits of a property which he is not allowed to own. Thus, it is likewise proscribed by law.

Philippine Carpet Manufacturing Corp. vs. TagyamonG.R. No. 191475               December 11, 2013

Facts:

Respondents Ignacio B. Tagyamon, Pablito L. Luna, Fe B. Badayos, and Cynthia L. Comandao were supervisors of petitioner Philippine Carpet Manufacturing Corp. (PCMC).

2

3

Last March 15, 2004, they received a memorandum of dismissal from the petitioner. They were informed that the petitioner was implementing a retrenchment program in accordance with Article 283 of the Labor Code. They were paid their separation pay and executed deeds of release, waiver and quitclaim.

Claiming that they were aggrieved by petitioner’s decision to terminate their employment, the respondents filed separate complaints against the petitioner for illegal dismissal.

Issue:

Whether or not their acceptance of separation pay and signing of quitclaim is not a bar to the pursuit of illegal dismissal case

Held: Yes. It can safely be concluded that economic necessity constrained the respondents to accept the petitioners’ monetary offer and sign the deeds of release, waiver and quitclaim. That respondents are supervisors and not rank-and-file employees does not make them less susceptible to financial offers, faced as they were with the prospect of unemployment. The Court has allowed supervisory employees to seek payment of benefits and a manager to sue for illegal dismissal even though, for a consideration, they executed deeds of quitclaims releasing their employers from liability (Ariola v. Philex Mining Corp., 503 Phil. 765, 780 (2005) at 789). x xx There is no nexus between intelligence, or even the position which the employee held in the company when it concerns the pressure which the employer may exert upon the free will of the employee who is asked to sign a release and quitclaim. A lowly employee or a sales manager, as in the present case, who is confronted with the same dilemma of whether (to sign) a release and quitclaim and accept what the company offers them, or (to refuse) to sign and walk out without receiving anything, may succumb to the same pressure, being very well aware that it is going to take quite a while before he can recover whatever he is entitled to, because it is only after a protracted legal battle starting from the labor arbiter level, all the way to this Court, can he receive anything at all. The Court understands that such a risk of not receiving anything whatsoever, coupled with the probability of not immediately getting any gainful employment or means of livelihood in the meantime, constitutes enough pressure upon anyone who is asked to sign a release and quitclaim in exchange of some amount of money which may be way below what he may be entitled to based on company practice and policy or by law (Philippine Carpet Manufacturing Corp., et. al. vs. Ignacio B. Tagyamon, et. al., G.R. No. 191475, Dec. 11, 2013, quoting Becton Dickinson Phils., Inc. v. NLRC, 511 Phil. 566, 589-590 (2005)).x xx Laches is a doctrine in equity while prescription is based on law. Our courts are basically courts of law not courts of equity. Thus, laches cannot be invoked to resist the enforcement of an existing legal right. x xx Courts exercising equity jurisdiction are bound by rules of law and have no arbitrary discretion to disregard them. In Zabat Jr. v. Court of

5 | P a g e

Page 6: JURISDICTION-DIGESTED-OCHOA.docx

JURISDICTION CASE DIGEST TROPANG POTCHIAppeals x xx, this Court was more emphatic in upholding the rules of procedure. We said therein:As for equity which has been aptly described as a "justice outside legality," this is applied only in the absence of, and never against, statutory law or, as in this case, judicial rules of procedure. Aequetasnunguamcontravenit legis. The pertinent positive rules being present here, they should preempt and prevail over all abstract arguments based only on equity.Thus, where the claim was filed within the [four-year] statutory period, recovery therefore cannot be barred by laches. Courts should never apply the doctrine of laches earlier than the expiration of time limited for the commencement of actions at law."32An action for reinstatement by reason of illegal dismissal is one based on an injury to the complainants’ rights which should be brought within four years from the time of their dismissal pursuant to Article 114633 of the Civil Code. Respondents’ complaint filed almost 3 years after their alleged illegal dismissal was still well within the prescriptive period. Laches cannot, therefore, be invoked yet.34 To be sure, laches may be applied only upon the most convincing evidence of deliberate inaction, for the rights of laborers are protected under the social justice provisions of the Constitution and under the Civil Code.35

PICHAY V. OFFICE OF THE DEPUTY EXECUTIVE SECRETARY (2012)

ODES no power to try and decide cases’ E.O. No. 13 empowering it is unconstitutionalFACTSOn November 15, 2010, President Benigno Simeon Aquino III issued Executive Order No. 13 (E.O. 13), abolishing the PAGC and transferring its functions to the Office of the Deputy Executive Secretary for Legal Affairs (ODESLA), more particularly to its newly-established Investigative and Adjudicatory Division (IAD).On April 6, 2011, respondent Finance Secretary Cesar V. Purisima filed before the IAD-ODESLA a complaint-affidavit for grave misconduct against petitioner Prospero A. Pichay, Jr., Chairman of the Board of Trustees of the Local Water Utilities Administration (LWUA), as well as the incumbent members of the LWUA Board of Trustees, namely, Renato Velasco, Susana Dumlao Vargas, Bonifacio Mario M. Pena, Sr. and Daniel Landingin, which arose from the purchase by the LWUA of Four Hundred Forty-Five Thousand Three Hundred Seventy Seven (445,377) shares of stock of Express Savings Bank, Inc.On April 14, 2011, petitioner received an Order3 signed by Executive Secretary Paquito N. Ochoa, Jr. requiring him and his co-respondents to submit their respective written explanations under oath. In compliance therewith, petitioner filed a Motion to Dismiss Ex Abundante Ad Cautelam manifesting that a case involving the same transaction and charge of grave misconduct entitled, "Rustico B. Tutol, et al. v. Prospero Pichay, et al.", and docketed as OMB-C-A-10-0426-I, is already pending before the Office of the Ombudsman.ISSUEWhether E.O. 13 is unconstitutional for abrogating unto an administrative office a quasi-judicial function through and E.O. and not through legislative enactment by Congress.

HELDNO.The President has Continuing Authority to Reorganize the Executive Department under E.O. 292. In the case of BuklodngKawaning EIIB v. Zamora the Court affirmed that the President's authority to carry out a reorganization in any branch or agency of the executive department is an express grant by the legislature by virtue of Section 31, Book III, E.O. 292 (the Administrative Code of 1987), "the President, subject to the policy of the Executive Office and in order to achieve simplicity, economy and efficiency, shall have the continuing authority to reorganize the administrative structure of the Office of the President."The law grants the President this power in recognition of the recurring need of every President to reorganize his office "to achieve simplicity, economy and efficiency." The Office of the President is the nerve center of the Executive Branch. To remain effective and efficient, the Office of the President must be capable of being shaped and reshaped by the President in the manner he deems fit to carry out his directives and policies. After all, the Office of the President is the command post of the President. (Emphasis supplied)Clearly, the abolition of the PAGC and the transfer of its functions to a division specially created within the ODESLA is properly within the prerogative of the President under his continuing "delegated legislative authority to reorganize" his own office pursuant to E.O. 292.The President's power to reorganize the Office of the President under Section 31 (2) and (3) of EO 292 should be distinguished from his power to reorganize the Office of the President Proper. Under Section 31 (1) of EO 292, the President can reorganize the Office of the President Proper by abolishing, consolidating or merging units, or by transferring functions from one unit to another. In contrast, under Section 31 (2) and (3) of EO 292, the President's power to reorganize offices outside the Office of the President Proper but still within the Office of thePresident is limited to merely transferring functions or agencies from the Office of the President to Departments or gencies, and vice versa.The distinction between the allowable organizational actions under Section 31(1) on the one hand and Section 31 (2) and (3) on the other is crucial not only as it affects employees' tenurial security but also insofar as it touches upon the validity of the reorganization, that is, whether the executive actions undertaken fall within the limitations prescribed under E.O. 292. When the PAGC was created under E.O. 12, it was composed of a Chairman and two (2) Commissioners who held the ranks of Presidential Assistant II and I, respectively,9 and was placed directly "under the Office of the President."10 On the other hand, the ODESLA, to which the functions of the PAGC have now been transferred, is an office within the Office of the President Proper.11 Since both of these offices belong to the Office of the President Proper, the reorganization by way of abolishing the PAGC and transferring its functions to the ODESLA is allowable under Section 31 (1) of E.O. 292.The IAD-ODESLA is a fact-finding and recommendatory body not vested with quasi-judicial powers.while the term "adjudicatory" appears part of its appellation, the IAD-ODESLA cannot try and resolve cases, its authority being limited to the conduct of investigations, preparation of reports and submission of recommendations. E.O. 13 explicitly states that the IAD-

ODESLA shall "perform powers, functions and duties xxx, of PAGC."Under E.O. 12, the PAGC was given the authority to "investigate or hear administrative cases or complaints againstall presidential appointees in the government" and to "submit its report and recommendations to the President." The IAD-ODESLA is a fact-finding and recommendatory body to the President, not having the power to settlecontroversies and adjudicate cases. As the Court ruled in Cariño v. Commission on Human Rights, and later reiterated in Biraogo v. The Philippine Truth Commission:Fact-finding is not adjudication and it cannot be likened to the judicial function of a court of justice, or even a quasi- judicial agency or office. The function of receiving evidence and ascertaining therefrom the facts of a controversy is not a judicial function. To be considered as such, the act of receiving evidence and arriving at factual conclusions in a controversy must be accompanied by the authority of applying the law to the factual conclusions to the end that the controversy may be decided or determined authoritatively, finally and definitively, subject to such appeals or modes of review as may be provided by law.

G.R. No. 192935 December 7, 2010LOUIS “BAROK” C. BIRAOGOvs.THE PHILIPPINE TRUTH COMMISSION OF 2010

x – – – – – – – – – – – – – – – – – – – – – – -xG.R. No. 193036REP. EDCEL C. LAGMAN, REP. RODOLFO B. ALBANO, JR., REP. SIMEON A. DATUMANONG, and REP. ORLANDO B. FUA, SR.vs.EXECUTIVE SECRETARY PAQUITO N. OCHOA, JR. and DEPARTMENT OF BUDGET AND MANAGEMENT SECRETARY FLORENCIO B. ABAD

FACTS:Pres. Aquino signed E. O. No. 1 establishing Philippine Truth Commission of 2010 (PTC) dated July 30, 2010.

PTC is a mere ad hoc body formed under the Office of the President with the primary task to investigate reports of graft and corruption committed by third-level public officers and employees, their co-principals, accomplices and accessories during the

6 | P a g e

Page 7: JURISDICTION-DIGESTED-OCHOA.docx

JURISDICTION CASE DIGEST TROPANG POTCHI

previous administration, and to submit its finding and recommendations to the President, Congress and the Ombudsman. PTC has all the powers of an investigative body. But it is not a quasi-judicial body as it cannot adjudicate, arbitrate, resolve, settle, or render awards in disputes between contending parties. All it can do is gather, collect and assess evidence of graft and corruption and make recommendations. It may have subpoena powers but it has no power to cite people in contempt, much less order their arrest. Although it is a fact-finding body, it cannot determine from such facts if probable cause exists as to warrant the filing of an information in our courts of law.

Petitioners asked the Court to declare it unconstitutional and to enjoin the PTC from performing its functions. They argued that:

(a) E.O. No. 1 violates separation of powers as it arrogates the power of the Congress to create a public office and appropriate funds for its operation.

(b) The provision of Book III, Chapter 10, Section 31 of the Administrative Code of 1987 cannot legitimize E.O. No. 1 because the delegated authority of the President to structurally reorganize the Office of the President to achieve economy, simplicity and efficiency does not include the power to create an entirely new public office which was hitherto inexistent like the “Truth Commission.”

(c) E.O. No. 1 illegally amended the Constitution and statutes when it vested the “Truth Commission” with quasi-judicial powers duplicating, if not superseding, those of the Office of the Ombudsman created under the 1987 Constitution and the DOJ created under the Administrative Code of 1987.

(d) E.O. No. 1 violates the equal protection clause as it selectively targets for investigation and prosecution officials and personnel of the previous administration as if corruption is their peculiar species even as it excludes those of the other administrations, past and present, who may be indictable.

Respondents, through OSG, questioned the legal standing of petitioners and argued that:

1] E.O. No. 1 does not arrogate the powers of Congress because the President’s executive power and power of control necessarily include the inherent power to conduct investigations to ensure that laws are faithfully executed and that, in any event, the Constitution, Revised Administrative Code of 1987, PD No. 141616 (as amended), R.A. No. 9970 and settled jurisprudence, authorize the President to create or form such bodies.

2] E.O. No. 1 does not usurp the power of Congress to appropriate funds because there is no appropriation but a mere allocation of funds already appropriated by Congress.

3] The Truth Commission does not duplicate or supersede the functions of the Ombudsman and the DOJ, because it is a fact-finding body and not a quasi-judicial body and its functions do not duplicate, supplant or erode the latter’s jurisdiction.

4] The Truth Commission does not violate the equal protection clause because it was validly created for laudable purposes.

ISSUES:1. WON the petitioners have legal standing to file the petitions and question E. O. No. 1;2. WON E. O. No. 1 violates the principle of separation of powers by usurping the powers of Congress to create and to appropriate funds for public offices, agencies and commissions;3. WON E. O. No. 1 supplants the powers of the Ombudsman and the DOJ;4. WON E. O. No. 1 violates the equal protection clause.

RULING:The power of judicial review is subject to limitations, to wit: (1) there must be an actual case or controversy calling for the exercise of judicial power; (2) the person challenging the act must have the standing to question the validity of the subject act or issuance; otherwise stated, he must have a personal and substantial interest in the case such that he has sustained, or will sustain, direct injury as a result of its enforcement; (3) the question of constitutionality must be raised at the earliest opportunity; and (4) the issue of constitutionality must be the very lismota of the case.1. The petition primarily invokes usurpation of the power of the Congress as a body to which they belong as members. To the extent the powers of Congress are impaired, so is the power of each member thereof, since his office confers a right to participate in the exercise of the powers of that institution.

Legislators have a legal standing to see to it that the prerogative, powers and privileges vested by the Constitution in their office remain inviolate. Thus, they are allowed to question the validity of any official action which, to their mind, infringes on their prerogatives as legislators.

With regard to Biraogo, he has not shown that he sustained, or is in danger of sustaining, any personal and direct injury attributable to the implementation of E. O. No. 1.

Locus standi is “a right of appearance in a court of justice on a given question.” In private suits, standing is governed by the “real-parties-in interest” rule. It provides that “every action must be prosecuted or defended in the name of the real party in interest.” Real-party-in interest is “the party who stands to be benefited or injured by the judgment in the suit or the party entitled to the avails of the suit.”

Difficulty of determining locus standi arises in public suits. Here, the plaintiff who asserts a “public right” in assailing an allegedly illegal official action, does so as a representative of the general public. He has to show that he is entitled to seek judicial protection. He has to make out a sufficient interest in the vindication of the public order and the securing of relief as a “citizen” or “taxpayer.

The person who impugns the validity of a statute must have “a personal and substantial interest in the case such that he has sustained, or will sustain direct injury as a result.” The Court, however, finds reason in Biraogo’s assertion that the petition covers matters of transcendental importance to justify the exercise of jurisdiction by the Court. There are constitutional issues in the petition which deserve the attention of this Court in view of their seriousness, novelty and weight as precedents

The Executive is given much leeway in ensuring that our laws are faithfully executed. The powers of the President are not limited to those specific powers under the Constitution. One of the recognized powers of the President granted pursuant to this constitutionally-mandated duty is the power to create ad hoc committees. This flows from the obvious need to ascertain facts and determine if laws have been faithfully executed. The purpose of allowing ad hoc investigating bodies to exist is to allow an inquiry into matters which the President is entitled to know so that he can be properly advised and guided in the performance of his duties relative to the execution and enforcement of the laws of the land.

2. There will be no appropriation but only an allotment or allocations of existing funds already appropriated. There is no usurpation on the part of the Executive of the power of Congress to appropriate funds. There is no need to specify the amount to be earmarked for the operation of the commission because, whatever funds the Congress has provided for the Office of the President will be the very source of the funds for the commission. The amount that would be allocated to the PTC shall be subject to existing auditing rules and regulations so there is no impropriety in the funding.

3. PTC will not supplant the Ombudsman or the DOJ or erode their respective powers. If at all, the investigative function of the commission will complement those of the two offices. The function of determining probable cause for the filing of the appropriate complaints before the courts remains to be with the DOJ and the Ombudsman. PTC’s power to investigate is limited to obtaining facts so that it can advise and guide the President in the performance of his duties relative to the execution and enforcement of the laws of the land.

4. Court finds difficulty in upholding the constitutionality of Executive Order No. 1 in view of its apparent transgression of the equal protection clause enshrined in Section 1, Article III (Bill of Rights) of the 1987 Constitution.

Equal protection requires that all persons or things similarly situated should be treated alike, both as to rights conferred and responsibilities imposed. It requires public bodies

7 | P a g e

Page 8: JURISDICTION-DIGESTED-OCHOA.docx

JURISDICTION CASE DIGEST TROPANG POTCHI

and institutions to treat similarly situated individuals in a similar manner. The purpose of the equal protection clause is to secure every person within a state’s jurisdiction against intentional and arbitrary discrimination, whether occasioned by the express terms of a statue or by its improper execution through the state’s duly constituted authorities.

There must be equality among equals as determined according to a valid classification. Equal protection clause permits classification. Such classification, however, to be valid must pass the test of reasonableness. The test has four requisites: (1) The classification rests on substantial distinctions; (2) It is germane to the purpose of the law; (3) It is not limited to existing conditions only; and (4) It applies equally to all members of the same class.

The classification will be regarded as invalid if all the members of the class are not similarly treated, both as to rights conferred and obligations imposed.

Executive Order No. 1 should be struck down as violative of the equal protection clause. The clear mandate of truth commission is to investigate and find out the truth concerning the reported cases of graft and corruption during the previous administration only. The intent to single out the previous administration is plain, patent and manifest.

Arroyo administration is but just a member of a class, that is, a class of past administrations. It is not a class of its own. Not to include past administrations similarly situated constitutes arbitrariness which the equal protection clause cannot sanction. Such discriminating differentiation clearly reverberates to label the commission as a vehicle for vindictiveness and selective retribution. Superficial differences do not make for a valid classification.

The PTC must not exclude the other past administrations. The PTC must, at least, have the authority to investigate all past administrations.

The Constitution is the fundamental and paramount law of the nation to which all other laws must conform and in accordance with which all private rights determined and all public authority administered. Laws that do not conform to the Constitution should be stricken down for being unconstitutional.

WHEREFORE, the petitions are GRANTED. Executive Order No. 1 is hereby declared UNCONSTITUTIONAL insofar as it is violative of the equal protection clause of the Constitution.

SIMON, JR. vs COMMISSION ON HUMAN RIGHTSG.R. No. 100150, January 5, 1994

FACTS:On July 23, 1990, the Commission on Human Rights (CHR) issued and order, directing the petitioners "to desist from demolishing the stalls and shanties at North EDSA pending the resolution of the vendors/squatters complaint before the Commission" and ordering said petitioners to appear before the CHR.On September 10, 1990, petitioner filed a motion to dismiss questioning CHR's jurisdiction and supplemental motion to dismiss was filed on September 18, 1990 stating that Commissioners' authority should be understood as being confined only to the investigation of violations of civil and political rights, and that "the rights allegedly violated in this case were not civil and political rights, but their privilege to engage in business".On March 1, 1991, the CHR issued and Order denying petitioners' motion and supplemental motion to dismiss. And petitioners' motion for reconsideration was denied also in an Order, dated April 25, 1991.The Petitioner filed aa petition for prohibition, praying for a restraining order and preliminary injunction. Petitioner also prayed to prohibit CHR from further hearing and investigating CHR Case No. 90-1580, entitled "Ferno, et.al vs. Quimpo, et.al".

ISSUE:Is the issuance of an "order to desist" within the extent of the authority and power of the CRH?

HELD:No, the issuance of an "order to desist" is not within the extent of authority and power of the CHR. Article XIII, Section 18(1), provides the power and functions of the CHR to "investigate, on its own or on complaint by any part, all forms of human rights violation, involving civil and political rights".The "order to desist" however is not investigatory in character but an adjudicative power that it does not possess. The Constitutional provision directing the CHR to provide for preventive measures and legal aid services to the underprivileged whose human rights have been violated or need protection may not be construed to confer jurisdiction on the Commission to issue an restraining order or writ of injunction, for it were the intention, the Constitution would have expressly said so. Not being a court of justice, the CHR itself has no jurisdiction to issue the writ, for a writ of preliminary injunction may only be issued by the Judge in any court in which the action is pending or by a Justice of the CA or of the SC.The writ prayed for the petition is granted. The CHR is hereby prohibited from further proceeding with CHR Case No. 90-1580.

Isidro Cariñovs The Commission on Human

CARIÑO vs. COMMISSION ON HUMAN RIGHTSG.R. No. 96681, December 2, 1991

FACTS:Some 800 public school teachers undertook “mass concerted actions” to protest the alleged failure of public authorities to act upon their grievances. The “mass actions”

consisted in staying away from their classes, converging at the LiwasangBonifacio, gathering in peacable assemblies, etc. The Secretary of Education served them with an order to return to work within 24 hours or face dismissal. For failure to heed the return-to-work order, eight teachers at the Ramon Magsaysay High School were administratively charged, preventively suspended for 90 days pursuant to sec. 41, P.D. 807 and temporarily replaced. An investigation committee was consequently formed to hear the charges.

When their motion for suspension was denied by the Investigating Committee, said teachers staged a walkout signifying their intent to boycott the entire proceedings. Eventually, Secretary Carino decreed dismissal from service of Esber and the suspension for 9 months of Babaran, Budoy and del Castillo. In the meantime, a case was filed with RTC, raising the issue of violation of the right of the striking teachers’ to due process of law. The case was eventually elevated to SC. Also in the meantime, the respondent teachers submitted sworn statements to Commission on Human Rights to complain that while they were participating in peaceful mass actions, they suddenly learned of their replacement as teachers, allegedly without notice and consequently for reasons completely unknown to them.

While the case was pending with CHR, SC promulgated its resolution over the cases filed with it earlier, upholding the Sec. Carino’s act of issuing the return-to-work orders. Despite this, CHR continued hearing its case and held that the “striking teachers” “were denied due process of law;…they should not have been replaced without a chance to reply to the administrative charges;” there had been violation of their civil and political rights which the Commission is empowered to investigate.”

ISSUE:Whether or not CHR has the power to try and decide and determine certain specific cases such as the alleged human rights violation involving civil and political rights.

HELD:The Court declares the Commission on Human Rights to have no such power; and that it was not meant by the fundamental law to be another court or quasi-judicial agency in this country, or duplicate much less take over the functions of the latter.The most that may be conceded to the Commission in the way of adjudicative power is that it may investigate, i.e., receive evidence and make findings of fact as regards claimed human rights violations involving civil and political rights. But fact finding is not adjudication, and cannot be likened to the judicial function of a court of justice, or even a quasi-judicial agency or official. To be considered such, the faculty of receiving evidence and making factual conclusions in a controversy must be accompanied by the authority of applying the law to those factual conclusions to the end that the controversy may be decided or determined authoritatively, finally and definitively, subject to such appeals or modes of review as may be provided by law.

G.R. No. 185663 June 20, 2012

8 | P a g e

Page 9: JURISDICTION-DIGESTED-OCHOA.docx

JURISDICTION CASE DIGEST TROPANG POTCHI

REMEDIOS ANTONINO, Petitioner, vs.THE REGISTER OF DEEDS OF MAKATI CITY and TAN TIAN SU, Respondents.

This is a petition for review under Rule 45 of the Rules of Court, assailing the Decision1 dated May 26, 2008 and Resolution2 dated December 5, 2008 of the Court of Appeals (CA) in CA-G.R. SP No. 89145.

FACTS:

Since March 21, 1978, petitioner RemediosAntonino (Antonino) had been leasing a residential property located at Makati City and owned by private respondent Tan Tian Su (Su). Under the governing lease contract, Antonino was accorded with the right of first refusal in the event Su would decide to sell the subject property.3

On July 7, 2004, the parties executed a document denominated as Undertaking Agreement4 where Su agreed to sell to Antonino the subject property for P39,500,000.00. However, in view of a disagreement as to who between them would shoulder the payment of the capital gains tax, the sale did not proceed as intended.5

On July 9, 2004, Antonino filed a complaint against Su with the Regional Trial Court (RTC) of Makati City, for the reimbursement of the cost of repairs on the subject property and payment of damages. The complaint was raffled to Branch 149 and docketed as Civil Case No. 04-802.6 Later that same day, Antonino filed an amended complaint to enforce the Undertaking Agreement and compel Su to sell to her the subject property.7

In an Order8 dated December 8, 2004, the RTC dismissed Antonino’s complaint on the grounds of improper venue and non-payment of the appropriate docket fees. According to the RTC, Antonino’s complaint is one for specific performance, damages and sum of money, which are personal actions that should have been filed in the court of the place where any of the parties resides. Antonino and Su reside in Muntinlupa and Manila, respectively, thus Makati City is not the proper venue. Specifically:

The instant case is an action for specific performance with damages, a personal action, which may be commenced and tried where the plaintiff or any of the principal plaintiffs resides, or where the defendant or any of the principal defendants resides (Section 2, Rule 5 of the Rules of Court). Records show that plaintiff is a resident of 706 Acacia Avenue, Ayala Alabang Village, Muntinlupa City while defendant is a resident of 550 Sto. Cristo St., Binondo, Manila. Hence, the instant case should have been filed in the place of residence of either the plaintiff or defendant, at the election of the plaintiff. Contrary to the claim of plaintiff, the alleged written agreements presented by the plaintiff in her Amended Complaint do not contain any stipulation as to the venue of actions. x x x9

The RTC also ruled that it did not acquire jurisdiction over Antonino’s complaint in view of her failure to pay the correct amount of docket fees. Citing Manchester Development Corporation v. Court of Appeals,10 the RTC ruled that:

Anent the non-payment of filing fees on the Amended Complaint, plaintiff alleges that no new assessment was made when the Amended Complaint was filed since there [were] no additional damages prayed for. The Manchester decision has been recently relaxed as to allow additional payment of the necessary fees if the Honorable Court so orders an assessment thereof.

The Court is not persuaded.

The Amended Complaint, which the Court notes to have been filed at 4:00 o’clock in the afternoon or few hours after the initial complaint was filed, further prays that judgment be rendered "ordering defendant to sell his property located at 1623 Cypress, Dasmariñas Village, Makati City covered by TCT No. 426900 to plaintiff in accordance with the terms and conditions stipulated in their agreement dated July 7, 2004 and ordering defendant to desist from selling his property to any other party other than plaintiff.", which makes the instant case also an action for Specific Performance in addition to the claim for Damages. However, the value of the described property was not stated in the prayer and no docket fees were paid. Thus, following the ruling of the Supreme Court in the case of Manchester Development Corporation vs. Court of Appeals, G.R. No. 75919, May 7, 1987, that the Court acquires jurisdiction over any case only upon the payment of the prescribed docket fee, the instant case is hereby dismissed.11

On December 23, 2004, Su filed an Omnibus Motion,12 praying for the cancellation of the notice of lispendens, which Antonino caused to be annotated on the title covering the subject property and the issuance of a summary judgment on his counterclaims. Su, among others, alleged the propriety of cancelling the notice of lispendens in view of the dismissal of the complaint and Antonino’s failure to appeal therefrom.

On January 3, 2005, Antonino filed a Motion for Reconsideration,13 claiming that her complaint is a real action and the location of the subject property is determinative of its venue. Alternatively, she submitted a certification issued by the Commission on Elections, stating that she is a resident of Makati City. She then prayed for the reinstatement of her complaint and issuance of an order directing the clerk of court to assess the proper docket fees. This was denied by the RTC in an Order14 dated January 6, 2005, holding that there was non-compliance with Sections 4 and 5 of Rule 15 of the Rules of Court.

Antonino thus filed a Motion for Reconsideration15 dated January 21, 2005, claiming that there was due observance of the rules on motions. Antonino alleged that her motion for reconsideration from the RTC’s December 8, 2004 was set for hearing on January 7, 2005 and Su received a copy thereof on January 6, 2005. Antonino pleaded for a liberal interpretation of the rules as Su was notified of her motion before the hearing thereon and was not in any way prejudiced. She also reiterated her arguments for the reinstatement of her complaint.

In a Joint Resolution16 dated February 24, 2005, the RTC denied Su’s Omnibus Motion and Antonino’s January 21, 2005 Motion for Reconsideration. The RTC refused to cancel the notice of lispendens, holding that:

It is quite clear that the dismissal of the Amended Complaint was anchored on two grounds, e.g. (1) for improper venue and (2) for non-payment of docket fee. It is elementary that when a complaint was dismissed based on these grounds[,] the court did not resolve the case on the merits. Moreover, "a court cannot acquire jurisdiction over the subject matter of a case unless the docket fees are paid" x xx. Thus, the cause of action laid down in the complaint remains unresolved for proper re-filing before the proper court. Furthermore, the Supreme Court said: "The cancellation of such a precautionary notice is therefore also a mere incident in the action, and may be ordered by the Court having jurisdiction of it at any given time." x x x17

The RTC maintained its earlier ruling that Antonino’s Motion for Reconsideration from the December 8, 2004 Order is pro-forma and did not suspend the running of the period to file an appeal. The RTC also reiterated that Antonino’s complaint is a personal action such that the proper venue therefore is either the City of Manila or Muntinlupa City.

On April 1, 2005, Antonino filed with the CA a petition for annulment of judgment.18 Antonino prayed for the nullification of the RTC’s Order dated December 8, 2004 dismissing her complaint, Order dated January 6, 2005 denying her motion for reconsideration and Joint Resolution dated February 24, 2005 denying her motion for reconsideration of the January 6, 2005 Order. According to Antonino, the RTC committed grave abuse of discretion amounting to lack of jurisdiction when it ruled that her action for the enforcement of the Undertaking Agreement is personal and when it deprived her of an opportunity to pay the correct amount of docket fees. The RTC’s grave abuse of discretion, Antonino posited, was likewise exhibited by its strict application of the rules on motions and summary denial of her motion for reconsideration.

In its Decision19 dated May 26, 2008, the CA dismissed Antonino’s petition. While the CA recognized Antonino’s faulty choice of remedy, it proceeded to resolve the issues she raised relative to the dismissal of her complaint. Thus:

It should be stressed that in this case, there is neither allegation in the petition, nor sufficient proof adduced showing highly exceptional circumstance to justify the failure of petitioner to avail of the remedies of appeal, petition for relief or other appropriate remedy through no fault attributable to [her] before filing this petition for annulment of judgment. In Manipor v. Ricafort, the Supreme Court held, thus:

If the petitioner failed to avail of such remedies without sufficient justification, he cannot avail of an action for annulment because, otherwise, he would benefit from his own inaction or negligence.

Notwithstanding the foregoing procedural infirmity, and in the interest of justice, we shall look into the issues raised and decide the case on the merit.

9 | P a g e

Page 10: JURISDICTION-DIGESTED-OCHOA.docx

JURISDICTION CASE DIGEST TROPANG POTCHI

A perusal of the allegations of the complaint unambiguously shows that petitioner seeks to enforce the commitment of private respondent to sell his property in accordance with the terms and conditions of their purported agreement dated July 7, 2004. By implication, petitioner does not question the ownership of private respondent over the property nor does she claim, by any color of title, right to possess the property or to its recovery. The action is simply for the enforcement of a supposed contract, and thus, unmistakably a personal action.

Guided by the above rule (Section 2 of the 1997 Rules of Court), petitioner should have filed the case either in Muntinlupa City, where she resides, or in Manila, where private respondent maintains his residence. Other than filing the complaint in any of these places, petitioner proceeds with the risk of a possible dismissal of her case. Unfortunately for petitioner, private respondent forthwith raised improper venue as an affirmative defense and his stand was sustained by trial court, thus, resulting to the dismissal of the case.

Further, it is important to note that in a petition for annulment of judgment based on lack of jurisdiction, the petitioner must show not merely an abuse of jurisdictional discretion but an absolute lack of jurisdiction. The concept of lack of jurisdiction as a ground to annul a judgment does not embrace abuse of discretion. Petitioner, by claiming grave abuse of discretion on the part of the trial court, actually concedes and presupposes the jurisdiction of the court to take cognizance of the case. She only assails the manner in which the trial court formulated its judgment in the exercise of its jurisdiction. It follows that petitioner cannot use lack of jurisdiction as ground to annul the judgment by claiming grave abuse of discretion. In this case where the court refused to exercise jurisdiction due to improper venue, neither lack of jurisdiction nor grave abuse of discretion is available to challenge the assailed order of dismissal of the trial court.20 (Citations omitted)

Antonino filed a motion for reconsideration, which was denied by the CA in its Resolution dated December 5, 2008. 21

ISSUE:

The sole issue for the resolution of this Court is the propriety of Antonino’s use of the remedy of a petition for annulment of judgment as against the final and executory orders of the RTC.

RULING:

In Ramos v. Judge Combong, Jr.,22 this Court expounded that the remedy of annulment of judgment is only available under certain exceptional circumstances as this is adverse to the concept of immutability of final judgments:

Annulment of judgment is a recourse equitable in character, allowed only in exceptional cases as where there is no available or other adequate remedy. Rule 47 of the 1997 Rules of Civil Procedure, as amended, governs actions for annulment of judgments or final orders and resolutions, and Section 2 thereof explicitly provides only two grounds

for annulment of judgment, i.e., extrinsic fraud and lack of jurisdiction. The underlying reason is traceable to the notion that annulling final judgments goes against the grain of finality of judgment. Litigation must end and terminate sometime and somewhere, and it is essential to an effective administration of justice that once a judgment has become final, the issue or cause involved therein should be laid to rest. The basic rule of finality of judgment is grounded on the fundamental principle of public policy and sound practice that at the risk of occasional error, the judgment of courts and the award of quasi-judicial agencies must become final at some definite date fixed by law.23 (Citations omitted)

In Barco v. Court of Appeals,24 this Court emphasized that only void judgments, by reason of "extrinsic fraud" or the court’s lack of jurisdiction, are susceptible to being annulled.

The law sanctions the annulment of certain judgments which, though final, are ultimately void. Annulment of judgment is an equitable principle not because it allows a party-litigant another opportunity to reopen a judgment that has long lapsed into finality but because it enables him to be discharged from the burden of being bound to a judgment that is an absolute nullity to begin with.25

Apart from the requirement that the existence of "extrinsic fraud" or "lack of jurisdiction" should be amply demonstrated, one who desires to avail this remedy must convince that the ordinary and other appropriate remedies, such as an appeal, are no longer available for causes not attributable to him. This is clearly provided under Section 1, Rule 47 of the Rules of Court.

Antonino’s recourse to annulment of judgment is seriously flawed and the reasons are patent. There is therefore no reason to disturb the questioned issuances of the RTC that are already final and executory.

A petition for annulment of judgment cannot serve as a substitute for the lost remedy of an appeal.

First, Antonino cannot pursue the annulment of the various issuances of the RTC, primary of which is the Order dated December 8, 2004, in order to avoid the adverse consequences of their becoming final and executory because of her neglect in utilizing the ordinary remedies available. Antonino did not proffer any explanation for her failure to appeal the RTC’s Order dated December 8, 2004 and, thereafter, the Order dated January 6, 2005, denying her Motion for Reconsideration dated January 3, 2005. Knowledge of rudimentary remedial rules immediately indicates that an appeal was already available from the Order dated December 8, 2004, as this is a final order as contemplated under Sections 2, 3 and 5 of Rule 41 of the Rules of Court, and there was no legal compulsion for Antonino to move for reconsideration. Nonetheless, since there is no bar for her to file a motion for reconsideration so as to give the RTC opportunity to reverse itself before elevating the matter for the appellate courts’ review, appeal is the prescribed remedy from the denial of such motion and not another motion for reconsideration. While Section 1 of Rule 41 of the Rules of Court includes "an order denying a motion for new trial or reconsideration" in the enumeration of unappealable matters, this Court clarified in Quelnan v. VHF Philippines, Inc.26 that such refers to a

motion for reconsideration of an interlocutory order and the denial of a motion for reconsideration of an order of dismissal is a final order, therefore, appealable. Moreover, a second motion for reconsideration from a final judgment or order is prohibited, hence, can never interrupt the period to perfect an appeal.

The RTC may have been overly strict in the observance of the three-day notice rule under Section 4, Rule 15 of the Rules of Court contrary to liberal stance taken by this Court in cases when the purpose of such rule can be achieved by giving the opposing party sufficient time to study and controvert the motion.27 Justice and equity would thus suggest that the fifteen-day period within which Antonino can appeal should be counted from her receipt on January 7, 200528 of the Order dated January 6, 2005 denying her Motion for Reconsideration dated January 3, 2005. Unfortunately, even liberality proved to be inadequate to neutralize the adverse consequences of Antonino’s negligence as she allowed such period to lapse without filing an appeal, erroneously believing that a second motion for reconsideration is the proper remedy. While a second motion for reconsideration is not prohibited insofar as interlocutory orders are concerned,29 the Orders dated December 8, 2004 and January 6, 2005 are final orders.

In fact, even if the period to appeal would be counted from Antonino’s receipt of the Order dated February 24, 2005 denying her second motion for reconsideration, she interposed no appeal and filed a petition for annulment of judgment on April 1, 2005 instead. This, for sure, constitutes a categorical admission that the assailed issuances of the RTC had already become final and executory in view of her omission to perfect an appeal within the mandated period. By no means can her petition for annulment of judgment prosper as that would, in effect, sanction her blatant negligence or sheer obliviousness to proper procedure.

Let it be stressed at the outset that before a party can avail of the reliefs provided for by Rule 47, i.e., annulment of judgments, final orders, and resolutions, it is a condition sine qua non that one must have failed to move for new trial in, or appeal from, or file a petition for relief against said issuances or take other appropriate remedies thereon, through no fault attributable to him. If he failed to avail of those cited remedies without sufficient justification, he cannot resort to the action for annulment provided in Rule 47, for otherwise he would benefit from his own inaction or negligence.30 (Citation omitted)

"Grave abuse of discretion" is not a ground to annul a final and executory judgment.

Second, a petition for annulment of judgment can only be based on "extrinsic fraud" and "lack of jurisdiction" and cannot prosper on the basis of "grave abuse of discretion". By anchoring her petition on the alleged grave abuse of discretion that attended the dismissal of her complaint and the denial of her two (2) motions for reconsideration, Antonino, is, in effect, enlarging the concept of "lack of jurisdiction". As this Court previously clarified in Republic of the Philippines v. "G" Holdings, Inc., 31 "lack of jurisdiction" as a ground for the annulment of judgments pertains to lack of jurisdiction over the person of the defending party or over the subject matter of the claim. It does not contemplate "grave abuse of discretion" considering that "jurisdiction" is different from the exercise thereof. As ruled in Tolentino v. Judge Leviste:32

10 | P a g e

Page 11: JURISDICTION-DIGESTED-OCHOA.docx

JURISDICTION CASE DIGEST TROPANG POTCHI

Jurisdiction is not the same as the exercise of jurisdiction. As distinguished from the exercise of jurisdiction, jurisdiction is the authority to decide a cause, and not the decision rendered therein. Where there is jurisdiction over the person and the subject matter, the decision on all other questions arising in the case is but an exercise of the jurisdiction. And the errors which the court may commit in the exercise of jurisdiction are merely errors of judgment which are the proper subject of an appeal. 33 (Citation omitted)

In fact, the RTC did not gravely abuse its discretion or err in dismissing Antonino’s complaint. The RTC was correct in classifying Antonino’s cause of action as personal and in holding that it was instituted in the wrong venue. Personal action is one that is founded on privity of contracts between the parties; and in which the plaintiff usually seeks the recovery of personal property, the enforcement of a contract, or recovery of damages. Real action, on the other hand, is one anchored on the privity of real estate, where the plaintiff seeks the recovery of ownership or possession of real property or interest in it.34 Antonino’s following allegations in her amended complaint show that one of her causes of action is one for the enforcement or consummation of a contract, hence, a personal action:

XII

On July 7, 2004, plaintiff and defendant executed a document entitled "Undertaking Agreement" (copy of which is hereto attached as Annex H) wherein defendant agreed to sell said property to plaintiff "who has leased said property since March 21, 1978 up to the present" with the plaintiff paying a downpayment of $50,000.00 US dollars the following day, July 8, 2004.

XIV

Defendant also refused to accept the $50,000.00 US Dollars and was about to tear up the document they previously signed the day before when plaintiff prevented him from doing so.

XV

Consequently, plaintiff discovered that defendant was already negotiating to sell the said property to another Chinese national who incidentally is also one of plaintiff’s buyers.

Premises considered, in the interest of substantial justice, it is most respectfully prayed that after due hearing that judgment be rendered:

1. Ordering defendant to sell his property located at 1623 Cypress, Dasmariñas Village, Makati City covered by TCT No. 426900 to plaintiff in accordance with the terms and conditions stipulated in their agreement dated July 7, 2004.

Antonino’s cause of action is premised on her claim that there has already been a perfected contract of sale by virtue of their execution of the Undertaking Agreement and Su had refused to comply with his obligations as seller. However, by claiming the existence of a perfected contract of sale, it does not mean that Antonino acquired title to the subject property. She does not allege otherwise and tacitly acknowledges Su’s title to the subject property by asking for the consummation of the sale.

That there is a private document supposedly evidencing the alleged sale does not confer to Antonino title to the subject property.1âwphi1 Ownership is transferred when there is actual or constructive delivery and the thing is considered delivered when it is placed in the control or possession of the buyer or when the sale is made through a public instrument and the contrary does not appear or cannot be clearly inferred.36 In other words, Antonino’s complaint is not in the nature of a real action as ownership of the subject property is not at issue.

Moreover, that the object of the alleged sale is a real property does not make Antonino’s complaint real in nature in the absence of a contrary claim of title. After a contract of sale is perfected, the right of the parties to reciprocally demand performance, thus consummation, arises – the vendee may require the vendor to compel the transfer the title to the object of the sale37 and the vendor may require the payment of the purchase price.38 The action to cause the consummation of a sale does not involve an adverse claim of ownership as the vendor’s title is recognized and the vendor is simply being asked to perform an act, specifically, the transfer of such title by any of the recognized modes of delivery.

Considering that the filing of the complaint in a wrong venue sufficed for the dismissal thereof, it would be superfluous to discuss if Antonino’s non-payment of the correct docket fees likewise warranted it.

At any rate, even if the RTC erred in ordering the dismissal of her complaint, such had already become final and executory and will not be disturbed as it had jurisdiction and it was not alleged, much less, proved that there was extrinsic fraud. Moreover, annulment of the assailed orders of the RTC will not issue if ordinary remedies, such as an appeal, were lost and were not availed of because of Antonino’s fault. Litigation should end and terminate sometime and somewhere. It is essential to an effective and efficient administration of justice that, once a judgment has become final, the winning party should not be deprived of the fruits of the verdict.39

WHEREFORE, premises considered, the petition is DENIED for lack of merit and the Decision dated May 26, 2008 and Resolution dated December 5, 2008 of the Court of Appeals in CA-G.R. SP No. 89145 are hereby AFFIRMED.

SO ORDERED.

ABRENICA VS ABRENICAGR 180572 06.18.12

FACTS:

Petitioner and Respondent were law firm partners. R filed a caseagainst P d to return partnership fundsrepresenting profits from the sale of a parcel of land and sought to recover from petitionerretainer fees that he received from two clientsof the firm and the balance of the cash advance that he obtained.

Petitioner filed an Urgent Omnibus Motion alleging thatthe sheriff had levied on properties belongingto his children and petitioner Joena. Joena filedan Affidavit of Third Party alleging that she andher stepchildren owned a number of thepersonal properties sought to be levied and that it was under their ACP.

A Sheriff’s Certificate of Sale was issued on 3January 2008 in favor of the law firm for the P’s properties. Petitioner has been previously married to another womanbut their marriage has already been dissolved.

ISSUE:

WON Joena had the right to the claim?

HELD:

NO. Two of these stepchildren were already of legal age when Joena filed her Affidavit. As toone of the children, parental authority over himbelongs to his parents. Absent any specialpower of attorney authorizing Joena to representErlando’s children, her claim cannotbe sustained.

Art. 92, par. (3) of the Family Code

Excludesfrom the community property the propertyacquired before the marriage of a spouse whohas legitimate descendants by a formermarriage; and the fruits and the income, if any,of that property. Thus, neither these two.

WHOLE CASE:

G.R. No. 180572 June 18, 2012

SPOUSES ATTY. ERLANDO A. ABRENICA and JOENA B. ABRENICA Petitioners, vs.LAW FIRM OF ABRENICA, TUNGOL and TIBAYAN, ATTYS. ABELARDO M. TIBAYAN and DANILO N. TUNGOL, Respondents.

The present case is a continuation of G.R. No. 1694201 decided by this Court on 22 September 2006. For brevity, we quote the relevant facts narrated in that case:

11 | P a g e

Page 12: JURISDICTION-DIGESTED-OCHOA.docx

JURISDICTION CASE DIGEST TROPANG POTCHI

Petitioner Atty. Erlando A. Abrenica was a partner of individual respondents, Attys. Danilo N. Tungol and Abelardo M. Tibayan, in the Law Firm of Abrenica, Tungol and Tibayan ("the firm").

In 1998, respondents filed with the Securities and Exchange Commission (SEC) two cases against petitioner. The first was SEC Case No. 05-98-5959, for Accounting and Return and Transfer of Partnership Funds With Damages and Application for Issuance of Preliminary Attachment, where they alleged that petitioner refused to return partnership funds representing profits from the sale of a parcel of land in Lemery, Batangas. The second was SEC Case No. 10-98-6123, also for Accounting and Return and Transfer of Partnership Funds where respondents sought to recover from petitioner retainer fees that he received from two clients of the firm and the balance of the cash advance that he obtained in 1997.

The SEC initially heard the cases but they were later transferred to the Regional Trial Court of Quezon City pursuant to Republic Act No. 8799, which transferred jurisdiction over intra-corporate controversies from the SEC to the courts. In a Consolidated Decision dated November 23, 2004, the Regional Trial Court of Quezon City, Branch 226, held that:

WHEREFORE, in view of all the foregoing, judgment is hereby rendered as follows:

CIVIL CASE NO. Q01-42948

1. Ordering the respondent Atty. ErlandoAbrenica to render full accounting of the amounts he received as profits from the sale and resale of the Lemery property in the amount of P 4,524,000.00;

2. Ordering the respondent Atty. ErlandoAbrenica to remit to the law firm the said amount of P4,524,000.00 plus interest of 12% per annum from the time he received the same and converted the same to his own personal use or from September 1997 until fully paid; and

3. To pay the costs of suit.

CIVIL CASE NO. Q01-42959

1. Ordering Atty. ErlandoAbrenica to render a full accounting of the amounts he received under the retainer agreement between the law firm and Atlanta Industries Inc. and Atlanta Land Corporation in the amount of P 320,000.00.

2. Ordering Atty. ErlandoAbrenica to remit to the law firm the amount received by him under the Retainer Agreement with Atlanta Industries, Inc. and Atlanta Land Corporation in the amount of P 320,000.00 plus interests of 12% per annum from June 1998 until fully paid;

3. Ordering Atty. ErlandoAbrenica to pay the law firm his balance on his cash advance in the amount of P25,000.00 with interest of 12% per annum from the date this decision becomes final; and

4. To pay the costs of suit.

SO ORDERED.

Petitioner received a copy of the decision on December 17, 2004. On December 21, 2004, he filed a notice of appeal under Rule 41 and paid the required appeal fees.

Two days later, respondents filed a Motion for Issuance of Writ of Execution pursuant to A.M. 01-2-04-SC, which provides that decisions in intra-corporate disputes are immediately executory and not subject to appeal unless stayed by an appellate court.

On January 7, 2005, respondents filed an Opposition (To Defendant's Notice of Appeal) on the ground that it violated A.M. No. 04-9-07-SC2 prescribing appeal by certiorari under Rule 43 as the correct mode of appeal from the trial court’s decisions on intra-corporate disputes.

Petitioner thereafter filed a Reply with Manifestation (To the Opposition to Defendant's Notice of Appeal) and an Opposition to respondents’ motion for execution.

On May 11, 2005, the trial court issued an Order requiring petitioner to show cause why it should take cognizance of the notice of appeal in view of A.M. No. 04-9-07-SC. Petitioner did not comply with the said Order. Instead, on June 10, 2005, he filed with the Court of Appeals a Motion for Leave of Court to Admit Attached Petition for Review under Rule 43 of the Revised Rules of Court. Respondents opposed the motion.

The Court of Appeals denied petitioner's motion in its assailed Resolution dated June 29, 2005 x xx.

x xx x xx x xx

The Court of Appeals also denied petitioner's motion for reconsideration in its August 23, 2005 Resolution.

Given the foregoing facts, we dismissed the Petition in G.R. No. 169420 on the ground that the appeal filed by petitioner was the wrong remedy. For that reason, we held as follows:3

Time and again, this Court has upheld dismissals of incorrect appeals, even if these were timely filed. In Lanzaderas v. Amethyst Security and General Services, Inc., this Court affirmed the dismissal by the Court of Appeals of a petition for review under Rule 43 to question a decision because the proper mode of appeal should have been a petition for certiorari under Rule 65. x xx.

x xx x xx x xx

Indeed, litigations should, and do, come to an end. "Public interest demands an end to every litigation and a belated effort to reopen a case that has already attained finality will serve no purpose other than to delay the administration of justice." In the instant case, the trial court's decision became final and executory on January 3, 2005. Respondents had already acquired a vested right in the effects of the finality of the decision, which should not be disturbed any longer.

WHEREFORE, the petition is DENIED. The Court of Appeals Resolutions dated June 29, 2005 and August 23, 2005 in CA-G.R. SP No. 90076 denying admission of petitioner’s Petition for Review are AFFIRMED.

Thus, respondents sought the execution of the judgment. On 11 April 2007, G.R. No. 169420 became final and executory.4

Apparently not wanting to be bound by this Court’s Decision in G.R. No. 169420, petitioners Erlando and Joena subsequently filed with the Court of Appeals (CA) a Petition for Annulment of Judgment with prayer for the issuance of a writ of preliminary injunction and/or temporary restraining order, docketed as CA-G.R. SP No. 98679. The Petition for Annulment of Judgment assailed the merits of the RTC’s Decision in Civil Case Nos. Q-01-42948 and Q-01-42959, subject of G.R. No. 169420. In that Petition for Annulment, Petitioners raised the following grounds:

I. The lower court erred in concluding that both petitioners and respondents did not present direct documentary evidence to substantiate [their] respective claims.

II. The lower court erred in concluding that both petitioners and respondents relied mainly on testimonial evidence to prove their respective position[s].

III. The lower court erred in not ruling that the real estate transaction entered into by said petitioners and spouses Roman and AmaliaAguzar was a personal transaction and not a law partnership transaction.

IV. The lower court erred in ruling that the testimonies of the respondents are credible.

V. The lower court erred in ruling that the purchase price for the lot involved was P 3 million and not P 8 million.

VI. The lower court erred in ruling that petitioner’s retainer agreement with Atlanta Industries, Inc. was a law partnership transaction.

VII. The lower court erred when it failed to rule on said petitioners’ permissive counterclaim relative to the various personal loans secured by respondents.

12 | P a g e

Page 13: JURISDICTION-DIGESTED-OCHOA.docx

JURISDICTION CASE DIGEST TROPANG POTCHI

VIII. The lower court not only erred in the exercise of its jurisdiction but more importantly it acted without jurisdiction or with lack of jurisdiction. 5

We note that petitioners were married on 28 May 1998. The cases filed with the Securities and Exchange Commission (SEC) on 6 May 1998 and 15 October 1998 were filed against petitioner Erlando only. It was with the filing of CA-G.R. SP No. 98679 on 24 April 2007 that Joena joined Erlando as a co-petitioner.

On 26 April 2007, the CA issued a Resolution6 dismissing the Petition. First, it reasoned that the remedy of annulment of judgment under Rule 47 of the Rules of Court is available only when the ordinary remedies of new trial, appeal, petition for relief or other appropriate remedies are no longer available through no fault of petitioners.7 Considering that the dismissal of the appeal was directly attributable to them, the remedy under Rule 47 was no longer available.

Second, the CA stated that the grounds alleged in the Petition delved on the merits of the case and the appreciation by the trial court of the evidence presented to the latter. Under Rule 47, the grounds for annulment are limited only to extrinsic fraud and lack of jurisdiction.

Lastly, the CA held that the fact that the trial court was not designated as a special commercial court did not mean that the latter had no jurisdiction over the case. The appellate court stated that, in any event, petitioners could have raised this matter on appeal or through a petition for certiorari under Rule 65, but they did not do so.

Petitioners filed an Amended Petition for Annulment of Judgment dated 2 May 2007, but the CA had by then already issued the 26 April 2007 Resolution dismissing the Petition.

On 24 May 2007, the 26 April 2007 Resolution in CA-G.R. SP No. 98679 became final and executory.8

Petitioners did not give up. They once again filed a 105-page Petition for Annulment of Judgment with the CA dated 25 May 20079 docketed as CA-G.R. SP No. 99719. This time, they injected the ground of extrinsic fraud into what appeared to be substantially the same issues raised in CA-G.R. SP No. 98679. The following were the grounds raised in CA-G.R. SP No. 99719:

A. Extrinsic fraud and/or collusion attended the rendition of the Consolidated Decision x xx based on the following badges of fraud and/or glaring errors deliberately committed, to wit:

I. The lower court deliberately erred in concluding that both petitioners and respondents did not present direct documentary evidence to substantiate their respective claims, as it relied purely on the gist of what its personnel did as regards the transcript of stenographic notes the latter [sic] in collusion with the respondents.

II. The lower court deliberately erred in concluding that both petitioners and respondents relied mainly on testimonial evidence to prove their respective positions by relying totally on what was presented to it by its personnel who drafted the Consolidated Decision in collusion with the respondents.

III. The lower court deliberately erred in not ruling that the real estate transaction entered into by said petitioners and spouses Roman and AmaliaAguzar was a personal transaction and not a law partnership transaction for the same reasons as stated in Nos. 1 and II above.

IV. The lower court deliberately erred in ruling that the testimonies of the respondents are credible as against the petitioner ErlandoAbrenica and his witnesses for the same reasons as stated in Nos. I and II above.

V. The lower court deliberately erred in ruling that the purchase price for the lot involved was P 3 million and not P 8 million for the same reasons as stated in Nos. 1 and II above.

VI. The lower court deliberately erred in ruling that petitioner’s retainer agreement with Atlanta Industries, Inc. was a law partnership transaction for the same reasons as stated in Nos. 1 and II above.

VII. The lower court deliberately erred when it failed to rule on said petitioners’ permissive counterclaim relative to the various personal loans secured by respondents also for the same reasons as the above.

B. As an incident of the extrinsic fraud[,] the lower court[,] despite full knowledge of its incapacity[,] rendered/promulgated the assailed Consolidated Decision x xx without jurisdiction or with lack of jurisdiction.10(Underscoring in the original.)

On 2 August 2007, the CA issued the first assailed Resolution11 dismissing the Petition in CA-G.R. SP No. 99719, which held the Petition to be insufficient in form and substance. It noted the following:

x xx. Readily noticeable is that CA-G.R. SP No. 90076 practically contained the prayer for the annulment of the subject consolidated Decision premised on the very same allegations, grounds or issues as the present annulment of judgment case.

x xx x xx x xx

Annulment of judgment is a recourse equitable in character, allowed only in exceptional cases as where there is no available or other adequate remedy (Espinosa vs. Court of Appeals, 430 SCRA 96[2004]). Under Section 2 of Rule 47 of the Revised Rules of Court, the only grounds for an annulment of judgment are extrinsic fraud and lack of jurisdiction (Cerezo vs. Tuazon, 426 SCRA 167 [2004]). Extrinsic fraud shall not be a valid ground if it was availed of, or could have been availed of, in a motion for new trial or petition for relief.

x xx x xx x xx

x xx. In the case at bar, not only has the court a quo jurisdiction over the subject matter and over the persons of the parties, what petitioner is truly complaining [of] here is only a possible error in the exercise of jurisdiction, not on the issue of jurisdiction itself. Where there is jurisdiction over the person and the subject matter (as in this case), the decision on all other questions arising in the case is but an exercise of the jurisdiction. And the errors which the court may commit in the exercise of jurisdiction are merely errors of judgment which are the proper subject of an appeal (Republic vs. "G" Holdings, supra, citing Tolentino vs. Leviste, supra). (Emphasis supplied.)

Subsequently, petitioners filed a Humble Motion for Reconsideration12 on 28 August 2007.

While the 28 August 2007 motion was pending, on 13 September 2007, petitioner Erlando filed an Urgent Omnibus Motion13 with Branch 226, alleging that the sheriff had levied on properties belonging to his children and petitioner Joena. In addition, Erlando alleged that the trial court still had to determine the manner of distribution of the firm’s assets and the value of the levied properties. Lastly, he insisted that the RTC still had to determine the issue of whether the Rule 41 appeal was the correct remedy.

On the same day, Joena filed an Affidavit of Third Party Claim14 also with Branch 226 of the RTC of Quezon City, alleging that she15 and her stepchildren16 owned a number of the personal properties sought to be levied. She also insisted that she owned half of the two (2) motor vehicles as well as the house and lot covered by Transfer Certificate of Title (TCT) No. 216818, which formed part of the absolute community of property. She likewise alleged that the real property, being a family home, and the furniture and the utensils necessary for housekeeping having a depreciated combined value of one hundred thousand pesos (P 100,000) were exempt from execution pursuant to Rule 39, Section 13 of the Rules of Court. Thus, she sought their discharge and release and likewise the immediate remittance to her of half of the proceeds, if any.

Accordingly, the RTC scheduled17 a hearing on the motion. On 17 October 2007, however, petitioner Erlando moved to withdraw his motion on account of ongoing negotiations with respondents.18

Thereafter, petitioner Erlando and respondent AbelardoTibayan, witnessed by Sheriff Nardo de Guzman, Jr. of Branch 226 of the RTC of Quezon City, executed an agreement to postpone the auction sale of the property covered by TCT No. 216818 in anticipation of an amicable settlement of the money judgment.19

Finally, on 30 October 2007, the CA in CA-G.R. SP No. 99719 issued the second assailed Resolution20 denying petitioners’ Motion for Reconsideration for having been filed out of time, as the last day for filing was on 27 August 2007. Moreover, the CA found that the grounds stated in the motion were merely recycled and rehashed propositions, which had already been dispensed with.

13 | P a g e

Page 14: JURISDICTION-DIGESTED-OCHOA.docx

JURISDICTION CASE DIGEST TROPANG POTCHI

Petitioners are now assailing the CA Resolutions dated 2 August 2007 and 30 October 2007, respectively, in CA-G.R. SP No. 99719. They insist that there is still a pending issue that has not been resolved by the RTC. That issue arose from the Order21 given by the trial court to petitioner Erlando to explain why it should take cognizance of the Notice of Appeal when the proper remedy was a petition for review under Rule 43 of the Rules of Court.

Further, petitioners blame the trial and the appellate courts for the dismissal of their appeal despite this Court’s explanation in G.R. No. 169420 that the appeal was the wrong remedy and was thus correctly dismissed by the CA. Instead of complying with the show-cause Order issued by the RTC, petitioners went directly to the CA and insisted that the remedy they had undertaken was correct.

Petitioners also contend that there was extrinsic fraud in the appreciation of the merits of the case. They raise in the present Petition the grounds they cited in the three (3) Petitions for Annulment of Judgment (including the Amended Petition) quoted above.

Next, they assert that petitioner Joena’s right to due process was also violated when she was not made a party-in-interest to the proceedings in the lower courts, even if her half of the absolute community of property was included in the execution of the judgment rendered by Branch 226 of the RTC of Quezon City.

Finally, they insist that their Humble Motion for Reconsideration was filed on time, since 27 August 2007 was a holiday. Therefore, they had until 28 August 2007 to file their motion.

Since then, it appears that a Sheriff’s Certificate of Sale was issued on 3 January 2008 in favor of the law firm for the sum of P 5 million for the property covered by TCT No. 216818.

On 18 March 2009, while the case was pending with this Court, petitioners filed a Complaint22 with a prayer for the issuance of a writ of preliminary injunction before the RTC of Marikina City against herein respondents and Sheriff Nardo I. de Guzman, Jr. of Branch 226 of the RTC of Quezon City. The case was docketed as Civil Case No. 09-1323-MK and was raffled to Branch 273 of the RTC of Marikina City. 23 Petitioners sought the nullification of the sheriff’s sale on execution of the Decision in the consolidated cases rendered by Branch 226, as well as the payment of damages. They alleged that the process of the execution sale was conducted irregularly, unlawfully, and in violation of their right to due process.

On 2 July 2009, Branch 273 of the RTC of Marikina City issued a Writ of Preliminary Injunction enjoining respondents and/or their agents, and the Register of Deeds of Marikina City from consolidating TCT No. 216818.24

The filing of the Complaint with the RTC of Marikina City prompted respondents to file a Motion25 before us to cite for contempt petitioner spouses and their counsel, Atty. Antonio R. Bautista. This Motion was on the ground that petitioners committed forum

shopping when they filed the Complaint pending with Branch 273 of the RTC of Marikina City, while the present case was also still pending.

Meanwhile, on 22 September 2009, respondents filed before Branch 226 an Ex Parte Motion for Issuance of Writ of Possession.26 That Motion was granted by Branch 226 through a Resolution27 issued on 10 November 2011. This Resolution then became the subject of a Petition for Certiorari28 under Rule 65 filed by petitioners before the CA docketed as CA-G.R. SP No. 123164.

Soon after, on 6 March 2012, petitioners filed with the CA an Urgent Motion for Issuance of Temporary Restraining Order (T.R.O.)29 after Sheriff De Guzman, Jr. served on them a Notice to Vacate within five days from receipt or until 11 March 2012. As of the writing of this Decision, the CA has not resolved the issue raised in the Petition in CA-G.R. SP No. 123164.

Our Ruling

Petitioners elevated this case to this Court, because they were allegedly denied due process when the CA rejected their second attempt at the annulment of the Decision of the RTC and their Humble Motion for Reconsideration.

We DENY petitioners’ claims.

The rules of procedure were formulated to achieve the ends of justice, not to thwart them. Petitioners may not defy the pronouncement of this Court in G.R. No. 169420 by pursuing remedies that are no longer available to them. Twice, the CA correctly ruled that the remedy of annulment of judgment was no longer available to them, because they had already filed an appeal under Rule 41. Due to their own actions, that appeal was dismissed.

It must be emphasized that the RTC Decision became final and executory through the fault of petitioners themselves when petitioner Erlando (1) filed an appeal under Rule 41 instead of Rule 43; and (2) filed a Petition for Review directly with the CA, without waiting for the resolution by the RTC of the issues still pending before the trial court.

In Enriquez v. Court of Appeals,30 we said:

It is true that the Rules should be interpreted so as to give litigants ample opportunity to prove their respective claims and that a possible denial of substantial justice due to legal technicalities should be avoided. But it is equally true that an appeal being a purely statutory right, an appealing party must strictly comply with the requisites laid down in the Rules of Court. In other words, he who seeks to avail of the right to appeal must play by the rules. x xx. (Emphasis supplied.)

With regard to the allegation of petitioner Joena that her right to due process was violated, it must be recalled that after she filed her Affidavit of Third Party Claim on 13 September 2007 and petitioner Erlando filed his Urgent Omnibus Motion raising the

same issues contained in that third-party claim, he subsequently filed two Motions withdrawing his Urgent Omnibus Motion. Petitioner Joena, meanwhile, no longer pursued her third-party claim or any other remedy available to her. Her failure to act gives this Court the impression that she was no longer interested in her case. Thus, it was through her own fault that she was not able to ventilate her claim.

Furthermore, it appears from the records that petitioner Erlando was first married to a certain Ma. Aline Lovejoy Padua on 13 October 1983. They had three children: PatrikErlando (born on 14 April 1985), Maria Monica Erline (born on 9 September 1986), and PatrikRandel (born on 12 April 1990).

After the dissolution of the first marriage of Erlando, he and Joena got married on 28 May 1998.31 In her Affidavit, Joena alleged that she represented her stepchildren; that the levied personal properties – in particular, a piano with a chair, computer equipment and a computer table – were owned by the latter. We note that two of these stepchildren were already of legal age when Joena filed her Affidavit. As to PatrikRandel, parental authority over him belongs to his parents. Absent any special power of attorney authorizing Joena to represent Erlando’s children, her claim cannot be sustained.

Petitioner Joena also asserted that the two (2) motor vehicles purchased in 1992 and 1997, as well as the house and lot covered by TCT No. 216818 formed part of the absolute community regime.1âwphi1 However, Art. 92, par. (3) of the Family Code excludes from the community property the property acquired before the marriage of a spouse who has legitimate descendants by a former marriage; and the fruits and the income, if any, of that property. Neither these two vehicles nor the house and lot belong to the second marriage.

We now proceed to discuss the Motion for contempt filed by respondents.

Respondents claim that petitioners and their present counsel, Atty. Antonio R. Bautista, were guilty of forum shopping when the latter filed Civil Case No. 09-1323-MK with the RTC of Marikina City while the case was still pending before us. In Executive Secretary v. Gordon,32 we explained forum shopping in this wise:

Forum-shopping consists of filing multiple suits involving the same parties for the same cause of action, either simultaneously or successively, for the purpose of obtaining a favorable judgment. Thus, it has been held that there is forum-shopping —

(1) whenever as a result of an adverse decision in one forum, a party seeks a favorable decision (other than by appeal or certiorari) in another, or

(2) if, after he has filed a petition before the Supreme Court, a party files another before the Court of Appeals since in such case he deliberately splits appeals "in the hope that even as one case in which a particular remedy is sought is dismissed, another case (offering a similar remedy) would still be open," or

14 | P a g e

Page 15: JURISDICTION-DIGESTED-OCHOA.docx

JURISDICTION CASE DIGEST TROPANG POTCHI

(3) where a party attempts to obtain a preliminary injunction in another court after failing to obtain the same from the original court.

Civil Case No. 09-1323-MK was filed to question the proceedings undertaken by the sheriff in executing the judgment in Civil Case Nos. Q01-42948 and Q01-42959. On the other hand, the present case questions the merits of the Decision itself in Civil Case Nos. Q01-42948 and Q01-42959. These cases have different causes of action. Thus, it cannot be said that petitioners were clearly guilty of forum shopping when they filed the Complaint before the RTC of Marikina City.

WHEREFORE, in view of the foregoing, the Petition is hereby DENIED. The Resolutions dated 2 August 2007 and 30 October 2007 issued by the Court of Appeals in CA-G.R. SP No. 99719 are AFFIRMED.

SO ORDERED.

Spouses Manila v Spouses Manzo G.R. No. 163602

Facts:

On June 30, 1982, Ederlinda Gallardo leased two parcels of land situated along Real St., Manuyo, Las Piñas, Metro Manila, to Eulogia Manila for ten years at a monthly rental of P2,000.00 for the first two years, and an increase of 10% percent every after two years. They also agreed that the lessee shall have the option to buy the property within two years from the date of execution of the contract of lease at a fair market value of 150,000.00php.

The contract of lease expired on July 1, 1992 but the lessee continued in possession of the property despite a formal demand letter dated August 8, 1992, to vacate the same and pay the rental arrearages. In a letter reply dated August 12, 1992, herein defendant claimed that no rental fee is due because she allegedly became the owner of the property at the time she communicated to the plaintiff her desire to exercise the option to buy the said property.

They went to the Barangay to settle this but

it seems that they couldn’t come to agreement, hence the present action.

Issue: Whether or not the court of appeals committed a grave error in annulling the judgement by the regional trial court of Makati city on the ground of lack of jurisdiction when it has not been shown that the regional trial court of Makati city had no jurisdiction over the person of respondents or the subject matter of the claim.

Held: Yes the regional trial court of Makati had no jurisdiction over the person of respondent because of the lack of jurisdiction.

Lack of jurisdiction as a ground for annulment of judgment refers to either lack of jurisdiction over the person of the defending party or over the subject matter of the claim. In a petition for annulment of judgment based on lack of jurisdiction, petitioner must show not merely an abuse of jurisdictional discretion but an absolute lack of jurisdiction. Lack of jurisdiction means absence of or no jurisdiction, that is, the court should not have taken cognizance of the petition because the law does not vest it with jurisdiction over the subject matter. Jurisdiction over the nature of the action or subject matter is conferred by law

Navida v Dizon

Facts:

Beginning 1993, a number of personal injury suits were filed in different Texas state courts by citizens of twelve foreign countries, including the Philippines. The thousands of plaintiffs sought damages for injuries they allegedly sustained from their exposure to dibromochloropropane (DBCP), a chemical used to kill nematodes (worms), while working on farms in 23 foreign countries. The cases were eventually transferred to, and consolidated in, the Federal District Court for the Southern District of Texas, Houston Division. The defendants in the consolidated cases prayed for the dismissal of all the actions under the doctrine of forum non conveniens.

In a Memorandum Order, the Federal District Court conditionally granted the defendants’ motion to dismiss provided the defendants:

(1) participated in expedited discovery in the United States

(2) either waived or accepted service of process and waived any other jurisdictional defense in any action commenced by a plaintiff in these actions in his home country or the country in which his injury occurred.

(3) waived any limitations-based defense that has matured since the commencement of these actions in the courts of Texas;

(4) stipulated that any discovery conducted during the pendency of these actions may be used in any foreign proceeding to the same extent as if it had been conducted in proceedings initiated there; and

(5) submitted an agreement binding them to satisfy any final judgment rendered in favor of plaintiffs by a foreign court.

In the event that the highest court of any foreign country finally affirms the dismissal for lack of jurisdiction of an action commenced by a plaintiff in these actions in his home country or the country in which he was injured, that plaintiff may return to this court and, upon proper motion, the court will resume jurisdiction over the action as if the case had never been dismissed for.

Case 1 (125078) and 2 (125598):

336 plaintiffs from General Santos City filed a Joint Complaint in the RTC of General Santos City. Named as defendants therein were: Shell Oil Co. (SHELL); Dow Chemical Co. (DOW); Occidental Chemical Corp. (OCCIDENTAL); Dole Food Co., Inc., Dole Fresh Fruit Co., Standard Fruit Co., Standard Fruit and Steamship Co. (hereinafter collectively referred to as DOLE); Chiquita Brands, Inc. and Chiquita Brands International, Inc. (CHIQUITA); Del Monte Fresh Produce N.A. and Del Monte Tropical Fruit Co. (hereinafter collectively referred to as DEL MONTE); Dead Sea Bromine Co., Ltd.; Ameribrom, Inc.; Bromine Compounds, Ltd.; and Amvac Chemical Corp. (The aforementioned defendants are hereinafter collectively referred to as defendant companies.)

15 | P a g e

Page 16: JURISDICTION-DIGESTED-OCHOA.docx

JURISDICTION CASE DIGEST TROPANG POTCHI

NAVIDA, et al., prayed for the payment of damages in view of the illnesses and injuries to the reproductive systems which they allegedly suffered because of their exposure to DBCP. They claimed, among others, that they were exposed to this chemical during the early 1970’s up to the early 1980’s when they used the same in the banana plantations where they worked at; and/or when they resided within the agricultural area where such chemical was used. NAVIDA, et al., claimed that their illnesses and injuries were due to the fault or negligence of each of the defendant companies in that they produced, sold and/or otherwise put into the stream of commerce DBCP-containing products. According to NAVIDA, et al., they were allowed to be exposed to the said products, which the defendant companies knew, or ought to have known, were highly injurious to the former’s health and well-being.

Without resolving the motions filed by the parties, the RTC of General Santos City issued an Order dismissing the complaint. First, the trial court determined that it did not have jurisdiction to hear the case because the substance of the cause of action as stated in the complaint against the defendant foreign companies cites activity on their part which took place abroad and had occurred outside and beyond the territorial domain of the Philippines. These acts of defendants cited in the complaint included the manufacture of pesticides, their packaging in containers, their distribution through sale or other disposition, resulting in their becoming part of the stream of commerce. The subject matter stated in the complaint and which is uniquely particular to the present case, consisted of activity or course of conduct engaged in by foreign defendants outside Philippine territory, hence, outside and beyond the jurisdiction of Philippine Courts, including the present Regional Trial Court.

Second, the RTC of General Santos City adjudged that NAVIDA, et al., were coerced into submitting their case to the Philippine courts, merely to comply with the U.S. District Court’s Order and in order to keep open to the plaintiffs the opportunity to return to the U.S. District Court.

Third, the trial court ascribed little significance to the voluntary appearance of the defendant companies. Defendants have appointed their agents authorized to accept service of summons/processes in the Philippines pursuant to the agreement in the U.S. court that defendants will voluntarily submit to the jurisdiction of this court. While it is true that this court acquires jurisdiction over persons of the defendants through their voluntary appearance, it appears that such voluntary appearance of the defendants in this case is conditional. Thus in the “Defendants’ Amended Agreement Regarding Conditions of Dismissal for Forum Non Conveniens” filed with the U.S. District Court, defendants declared that “(t)he authority of each designated representative to accept service of process will become effective upon final dismissal of these actions by the

Court”. The decision of the U.S. District Court dismissing the case is not yet final and executory since both the plaintiffs and defendants appealed therefrom. Consequently, since the authority of the agent of the defendants in the Philippines is conditioned on the final adjudication of the case pending with the U.S. courts, the acquisition of jurisdiction by this court over the persons of the defendants is also conditional.

Fourth, the RTC of General Santos City ruled that the act of NAVIDA, et al., of filing the case in the Philippine courts violated the rules on forum shopping and litis pendencia. This court frowns upon the fact that the parties herein are both vigorously pursuing their appeal of the decision of the U.S. District court dismissing the case filed thereat. To allow the parties to litigate in this court when they are actively pursuing the same cases in another forum, violates the rule on ‘forum shopping’ so abhorred in this jurisdiction. Moreover, the filing of the case in the U.S. courts divested this court of its own jurisdiction. This court takes note that the U.S. District Court did not decline jurisdiction over the cause of action. The case was dismissed on the ground of forum non conveniens, which is really a matter of venue. By taking cognizance of the case, the U.S. District Court has, in essence, concurrent jurisdiction with this court over the subject matter of this case. It is settled that initial acquisition of jurisdiction divests another of its own jurisdiction.

Case 3 (126654), 4 (127856), 5(128398)

Another joint complaint for damages against SHELL, DOW, OCCIDENTAL, DOLE, DEL MONTE, and CHIQUITA was filed before Branch 16 of the RTC of Davao City by 155 plaintiffs from Davao City. They alleged that as workers in the banana plantation and/or as residents near the said plantation, they were made to use and/or were exposed to nematocides, which contained the chemical DBCP. According to ABELLA, et al., such exposure resulted in “serious and permanent injuries to their health, including, but not limited to, sterility and severe injuries to their reproductive capacities.”

The RTC of Davao City, however, junked Civil Cases. The Court however is constrained to dismiss the case at bar not solely on the basis of the above but because it shares the opinion of legal experts given in the interview made by the Inquirer in its Special report “Pesticide Cause Mass Sterility,” Former Justice Secretary Demetrio Demetria in a May 1995 opinion said: The Philippines should be an inconvenient forum to file this kind of damage suit against foreign companies since the causes of action alleged in the petition do not exist under Philippine laws. There has been no decided case in Philippine Jurisprudence awarding to those adversely affected by DBCP. This means there is no

available evidence which will prove and disprove the relation between sterility and DBCP.

Eventually, the cases reached the SC!

Present case:

The main contention of the petitioners states that the allegedly tortious acts and/or omissions of defendant companies occurred within Philippine territory. Said fact allegedly constitutes reasonable basis for our courts to assume jurisdiction over the case.

DOLE similarly maintains that the acts attributed to defendant companies constitute a quasi-delict, which falls under Article 2176 of the Civil Code. DOLE also argues that if indeed there is no positive law defining the alleged acts of defendant companies as actionable wrong, Article 9 of the Civil Code dictates that a judge may not refuse to render a decision on the ground of insufficiency of the law. The court may still resolve the case, applying the customs of the place and, in the absence thereof, the general principles of law.

CHIQUITA (another petitioner) argues that the courts a quo had jurisdiction over the subject matter of the cases filed before them. CHIQUITA avers that the pertinent matter is the place of the alleged exposure to DBCP, not the place of manufacture, packaging, distribution, sale, etc., of the said chemical. This is in consonance with the lex loci delicti commisi theory in determining the situs of a tort, which states that the law of the place where the alleged wrong was committed will govern the action. CHIQUITA and the other defendant companies also submitted themselves to the jurisdiction of the RTC by making voluntary appearances and seeking for affirmative reliefs during the course of the proceedings.

Issue:

Whether or not the RTCs have jurisdiction over the subject matter in these cases.

Held: Yes.

16 | P a g e

Page 17: JURISDICTION-DIGESTED-OCHOA.docx

JURISDICTION CASE DIGEST TROPANG POTCHI

1. The rule is settled that jurisdiction over the subject matter of a case is conferred by law and is determined by the allegations in the complaint and the character of the relief sought, irrespective of whether the plaintiffs are entitled to all or some of the claims asserted therein. Once vested by law, on a particular court or body, the jurisdiction over the subject matter or nature of the action cannot be dislodged by anybody other than by the legislature through the enactment of a law.

At the time of the filing of the complaints, the jurisdiction of the RTC in civil cases under Batas Pambansa Blg. 129, as amended by Republic Act No. 7691, was:

In all other cases in which the demand, exclusive of interest, damages of whatever kind, attorney’s fees, litigation expenses, and costs or the value of the property in controversy exceeds One hundred thousand pesos (P100,000.00) or, in such other cases in Metro Manila, where the demand, exclusive of the abovementioned items exceeds Two hundred thousand pesos (P200,000.00).

Supreme Court Administrative Circular No. 09-94, states:

The exclusion of the term “damages of whatever kind” in determining the jurisdictional amount under Section 19 (8) and Section 33 (1) of B.P. Blg. 129, as amended by R.A. No. 7691, applies to cases where the damages are merely incidental to or a consequence of the main cause of action. However, in cases where the claim for damages is the main cause of action, or one of the causes of action, the amount of such claim shall be considered in determining the jurisdiction of the court.

It is clear that the claim for damages is the main cause of action and that the total amount sought in the complaints is approximately P2.7 million for each of the plaintiff claimants. The RTCs unmistakably have jurisdiction over the cases filed in General Santos City and Davao City.

2. The jurisdiction of the court cannot be made to depend upon the defenses set up in the answer or upon the motion to dismiss, for otherwise, the question of jurisdiction would almost entirely depend upon the defendants. What determines the jurisdiction of the court is the nature of the action pleaded as appearing from the allegations in the complaint. The averments therein and the character of the relief sought are the ones to be consulted.

Clearly then, the acts and/or omissions attributed to the defendant companies constitute a quasi-delict which is the basis for the claim for damages filed by NAVIDA, et al., and ABELLA, et al., with individual claims of approximately P2.7 million for each plaintiff claimant, which obviously falls within the purview of the civil action jurisdiction of the RTCs.

3. It is, therefore, error on the part of the courts a quo when they dismissed the cases on the ground of lack of jurisdiction on the mistaken assumption that the cause of action narrated by NAVIDA, et al., and ABELLA, et al., took place abroad and had occurred outside and beyond the territorial boundaries of the Philippines, i.e., “the manufacture of the pesticides, their packaging in containers, their distribution through sale or other disposition, resulting in their becoming part of the stream of commerce,” and, hence, outside the jurisdiction of the RTCs.

Certainly, the cases below are not criminal cases where territoriality, or the situs of the act complained of, would be determinative of jurisdiction and venue for trial of cases. In personal civil actions, such as claims for payment of damages, the Rules of Court allow the action to be commenced and tried in the appropriate court, where any of the plaintiffs or defendants resides, or in the case of a non-resident defendant, where he may be found, at the election of the plaintiff.

In a very real sense, most of the evidence required to prove the claims of NAVIDA, et al., and ABELLA, et al., are available only in the Philippines. First, plaintiff claimants are all residents of the Philippines, either in General Santos City or in Davao City. Second, the specific areas where they were allegedly exposed to the chemical DBCP are within the territorial jurisdiction of the courts a quo wherein NAVIDA, et al., and ABELLA, et al., initially filed their claims for damages. Third, the testimonial and documentary evidence from important witnesses, such as doctors, co-workers, family members and other members of the community, would be easier to gather in the Philippines.

----

Re: Jurisdiction over the person

The RTC of General Santos City and the RTC of Davao City validly acquired jurisdiction over the persons of all the defendant companies. All parties voluntarily, unconditionally and knowingly appeared and submitted themselves to the jurisdiction of the courts a quo. All the defendant companies submitted themselves to the jurisdiction of the courts a quo by making several voluntary appearances, by praying for various affirmative reliefs, and by actively participating during the course of the proceedings below.

In line herewith, this Court, in Meat Packing Corporation of the Philippines v. Sandiganbayan, held that jurisdiction over the person of the defendant in civil cases is acquired either by his voluntary appearance in court and his submission to its authority or by service of summons. Furthermore, the active participation of a party in the proceedings is tantamount to an invocation of the court’s jurisdiction and a willingness to abide by the resolution of the case, and will bar said party from later on impugning the court or body’s jurisdiction.

---

Jurisdiction v Exercise of Jurisdiction

It may also be pertinently stressed that “jurisdiction” is different from the “exercise of jurisdiction.” Jurisdiction refers to the authority to decide a case, not the orders or the decision rendered therein. Accordingly, where a court has jurisdiction over the persons of the defendants and the subject matter, as in the case of the courts a quo, the decision on all questions arising therefrom is but an exercise of such jurisdiction. Any error that the court may commit in the exercise of its jurisdiction is merely an error of judgment, which does not affect its authority to decide the case, much less divest the court of the jurisdiction over the case.

----

Re: Bad faith in filing cases to procure a dismissal and to allow petitioners to return to the forum of their choice.

This Court finds such argument much too speculative to deserve any merit.

It must be remembered that this Court does not rule on allegations that are unsupported by evidence on record. This Court does not rule on allegations which are manifestly conjectural, as these may not exist at all. This Court deals with facts, not fancies; on realities, not appearances.

17 | P a g e

Page 18: JURISDICTION-DIGESTED-OCHOA.docx

JURISDICTION CASE DIGEST TROPANG POTCHI

* We REMAND the records of this case to the respective Regional Trial Courts of origin for further and appropriate proceedings in line with the ruling herein that said courts have jurisdiction over the subject matter of the amended complaints.

Republic of the Philippines v Judge Mangotara

Before the Court are seven consolidated Petitions for Review on Certiorari and a Petition for Certiorari under Rules 45 and 65 of the Rules of Court, respectively, arising from actions for quieting of title, expropriation, ejectment, and reversion, which all involve the same parcels of land.

In G.R. No. 170375, the Republic of the Philippines (Republic), by way of consolidated Petitions for Review on Certiorari and for Certiorari under Rules 45 and 65 of the Rules of Court, respectively, seeks to set aside the issuances of Judge Mamindiara P. Mangotara (Judge Mangotara) of the Regional Trial Court, Branch 1 (RTC-Branch 1) of Iligan City, Lanao del Norte, in Civil Case No. 106, particularly, the: (1) Resolution [1] dated July 12, 2005 which, in part, dismissed the Complaint for Expropriation of the Republic for the latter’s failure to implead indispensable parties and forum shopping; and (2) Resolution[2] dated October 24, 2005, which denied the Partial Motion for Reconsideration of the Republic.

G.R. Nos. 178779 and 178894 are two Petitions for Review on Certiorari under Rule 45 of the Rules of Court, where Landtrade Realty Corporation (LANDTRADE), Teofilo Cacho, and/or Atty. Godofredo Cabildo assail the Decision[3] dated January 19, 2007 and Resolution[4] dated July 4, 2007 of the Court of Appeals in CA-G.R. CV No. 00456. The Court of Appeals affirmed the Decision [5] dated July 17, 2004 of the Regional Trial Court, Branch 3 (RTC-Branch 3) of IliganCity, Lanao del Norte, in Civil Case No. 4452, granting the Petition for Quieting of Title, Injunction and Damages filed by Demetria Vidal and Azimuth International Development Corporation (AZIMUTH) against Teofilo Cacho and Atty. Godofredo Cabildo.

G.R. No. 170505 is a Petition for Review on Certiorari under Rule 45 of the Rules of Court in which LANDTRADE urges the Court to reverse and set aside theDecision [6] dated November 23, 2005 of the Court of Appeals in CA-G.R. SP Nos. 85714 and 85841. The appellate court annulled several issuances of the Regional Trial Court, Branch 5 (RTC-Branch 5) of Iligan City, Lanao del Norte, and its sheriff, in Civil Case No. 6613,

specifically, the: (1) Order[7] dated August 9, 2004 granting the Motion for Execution Pending Appeal of LANDTRADE; (2) Writ of Execution[8] dated August 10, 2004; (3) two Notices of Garnishment[9] both dated August 11, 2004, and (4) Notification[10] dated August 11, 2004. These issuances of the RTC-Branch 5 allowed and/or enabled execution pending appeal of the Decision[11] dated February 17, 2004 of the Municipal Trial Court in Cities (MTCC), Branch 2 of Iligan City, Lanao del Norte, favoring LANDTRADE in Civil Case No. 11475-AF, the ejectment case said corporation instituted against the National Power Corporation (NAPOCOR) and the National Transmission Corporation (TRANSCO).

G.R. Nos. 173355-56 and 173563-64 are two Petitions for Certiorari and Prohibition under Rule 65 of the Rules of Court with prayer for the immediate issuance of a Temporary Restraining Order (TRO) and/or Writ of Preliminary Injunction filed separately by NAPOCOR and TRANSCO. Both Petitions seek to annul theResolution[12] dated June 30, 2006 of the Court of Appeals in the consolidated cases of CA-G.R. SP Nos. 00854 and 00889, which (1) granted the Omnibus Motion of LANDTRADE for the issuance of a writ of execution and the designation of a special sheriff for the enforcement of the Decision[13] dated December 12, 2005 of the RTC-Branch 1 in Civil Case No. 6613, and (2) denied the applications of NAPOCOR and TRANSCO for a writ of preliminary injunction to enjoin the execution of the same RTC Decision. The Decision dated December 12, 2005 of RTC-Branch 1 in Civil Case No. 6613 affirmed the Decision dated February 17, 2004 of the MTCC in Civil Case No. 11475-AF, favoring LANDTRADE.

G.R. No. 173401 involves a Petition for Review on Certiorari under Rule 45 of the Rules of Court filed by the Republic, which raises pure questions of law and seeks the reversal of the following issuances of the Regional Trial Court, Branch 4 (RTC-Branch 4) of Iligan City, Lanao del Norte, in Civil Case No. 6686, an action for cancellation of titles and reversion: (1) Order[14] dated December 13, 2005 dismissing the Complaint in Civil Case No. 6686; and (2) Order[15] dated May 16, 2006, denying the Motion for Reconsideration of the Republic.

THE PRECEDING CASES

The consolidated seven cases have for their common genesis the 1914 case of Cacho v. Government of the United States[16] (1914 Cacho case).

The 1914 Cacho Case

Sometime in the early 1900s, the late Doña Demetria Cacho (Doña Demetria) applied for the registration of two parcels of land: (1) Lot 1 of Plan II-3732, the smaller parcel with an area of 3,635 square meters or 0.36 hectares (Lot 1); and (2) Lot 2 of Plan II-3732, the larger parcel with an area of 378,707 square meters or37.87 hectares (Lot 2). Both parcels are situated in what was then the Municipality of Iligan, Moro Province, which later became Sitio Nunucan, then Brgy. Suarez, inIligan City, Lanao del Norte. Doña Demetria’s applications for registration were docketed as GLRO Record Nos. 6908 and 6909.

The application in GLRO Record No. 6908 covered Lot 1, the smaller parcel of land. Doña Demetria allegedly acquired Lot 1 by purchase from Gabriel Salzos (Salzos). Salzos, in turn, bought Lot 1 from Datto Darondon and his wife Alanga, evidenced by a deed of sale in favor of Salzos signed solely by Alanga, on behalf of Datto Darondon.

The application in GLRO Record No. 6909 involved Lot 2, the bigger parcel of land. Doña Demetria purportedly purchased Lot 2 from Datto Bunglay. Datto Bunglay claimed to have inherited Lot 2 from his uncle, Datto Anandog, who died without issue.

Only the Government opposed Doña Demetria’s applications for registration on the ground that the two parcels of land were the property of the United States and formed part of a military reservation, generally known as Camp Overton.

On December 10, 1912, the land registration court (LRC) rendered its Decision in GLRO Record Nos. 6908 and 6909.

Based on the evidence, the LRC made the following findings in GLRO Record No. 6908:

6th. The court is convinced from the proofs that the small parcel of land sold by the Moro woman Alanga was the home of herself and her husband, Darondon, and was their conjugal property; and the court so finds.

18 | P a g e

Page 19: JURISDICTION-DIGESTED-OCHOA.docx

JURISDICTION CASE DIGEST TROPANG POTCHI

x x x x

As we have seen, the deed on which applicant’s title to the small parcel rests, is executed only by the Moro woman Alanga, wife of Datto Darondon, which is not permitted either by the Moro laws or the Civil Code of the Philippine Islands. It appears that the husband of Alanga, Datto Darondon, is alive yet, and before admitting this parcel to registration it is ordered that a deed from Datto Darondon, husband of Alanga, be presented, renouncing all his rights in the small parcel of land object of Case No. 6908, in favor of the applicant.[17] (Emphases supplied.)

In GLRO Record No. 6909, the LRC observed and concluded that:

A tract of land 37 hectares in area, which is the extent of the land under discussion, is larger than is cultivated ordinarily by the Christian Filipinos. In the Zamboanga cadastral case of thousands of parcels now on trial before this court, the average size of the parcels is not above 3 or 4 hectares, and the court doubts very much if a Moro with all his family could cultivate as extensive a parcel of land as the one in question. x x x

x x x x

The court is also convinced from the proofs that the small portion in the southern part of the larger parcel, where, according to the proofs, Datto Anandog had his house and where there still exist some cocos and fruit trees, was the home of the said Moro Datto Anandog; and the court so finds. As to the rest of the large parcel the court does not find the title of Datto Bunglay established. According to his own declaration his residence on this land commenced only a few days before the sale. He admitted that the coco trees he is supposed to have planted had not yet begun to bear fruit at the time of the sale,

and were very small. Datto Duroc positively denies that Bunglay lived on the land, and it clearly appears that he was not on the land when it was first occupied by the military. Nor does Datto Bunglay claim to have planted the three mango trees by the roadside near point 25 of the plan. The court believes that all the rest of this parcel, not occupied nor cultivated by Datto Anandog, was land claimed by Datto Duroc and also by Datto Anandog and possibly by other dattos as a part of their general jurisdiction, and that it is the class of land that Act No. 718 prohibits the sale of, by the dattos, without the express approval of the Government.

It is also found that Datto Bunglay is the nephew of Dato Anandog, and that the Moro woman Alanga, grantor of the small parcel, is the sister of Datto Anandog, and that he died without issue.

x x x x

It appears also that according to the provisions of the Civil Code as also the provisions of the ‘Luwaran Code’ of the Moros, the Moro woman Alanga has an interest in the portion of land left by her deceased brother, Datto Anandog. By article LXXXV, section 3, of the ‘Luwaran Code,’ it will be seen that the brothers and sisters of a deceased Moro inherit his property to the exclusion of the more distant relatives. Therefore Datto Bunglay had no legal interest whatever in the land to sell to the applicant, Doña Demetria Cacho. But the Moro woman, Alanga, having appeared as a witness for the applicant without having made any claim to the land, the court finds from this fact that she has ratified the sale made by her nephew.

The court therefore finds that the applicant Doña Demetria Cacho is owner of the portion of land occupied and planted by the deceased Datto Anandog in the southern part of the large parcel object of expediente No. 6909 only; and her application as to all the rest of the land solicited in said case is denied. And it is ordered that a new survey of the land be made and a corrected plan be presented, excluding all the land not occupied and cultivated by Datto Anandog; that said survey be made and the corrected plan presented on or before the 30th day of March, 1913, with previous notice to the commanding general of the Division of the Philippines.

On the 8th day of December, the court was at Camp Overton and had another ocular inspection of the land for the purpose of fixing the limits of the part cultivated by Datto Anandog, so often mentioned herein, with previous notice to the applicant and her husband and representative, Señor Dionisio Vidal. Having arrived late, Señor Vidal did not assist in the ocular inspection, which was fixed for 3 o’clock, p.m. of the day mentioned. But the court, nevertheless, set stakes marking the N.E., S.E., and S.W. corners of the land found to have been cultivated by the deceased Anandog. The N.E. limit of said land is a brook, and the N.W. corner is the point where the brook intersects the shore line of the sea, the other corners mentioned being marked with pine stakes. And it is ordered that the new survey be made in accordance with the points mentioned, by tracing four straight lines connecting these four points. Between the portion cultivated by Datto Anandog and the mouth of the River Agus there is a high steep hill and the court does not believe it possible to cultivate said hill, it being covered with rocks and forest.[18] (Emphases supplied.)

The LRC additionally decreed at the end of its December 10, 1912 Decision:

It is further ordered that one-half of the costs of the new survey be paid by the applicant and the other half by the Government of the United States, and that the applicant present the corresponding deed from Datto Darondon on or before the above-mentioned 30th day of March, 1913. Final decision in these cases is reserved until the presentation of the said deed and the new plan.[19]

Apparently dissatisfied with the foregoing LRC judgment, Doña Demetria appealed to this Court. In its Decision dated December 10, 1914, the Court affirmed in toto the LRC Decision of December 10, 1912, well satisfied that the findings of fact of the court below were fully sustained by the evidence adduced during trial.

19 | P a g e

Page 20: JURISDICTION-DIGESTED-OCHOA.docx

JURISDICTION CASE DIGEST TROPANG POTCHI

Eighty-three years later, in 1997, the Court was again called upon to settle a matter concerning the registration of Lots 1 and 2 in the case of Cacho v. Court of Appeals[20] (1997 Cacho case).

The 1997 Cacho Case

On June 29, 1978, Teofilo Cacho (Teofilo), claiming to be the late Doña Demetria’s son and sole heir, filed before the RTC a petition for reconstitution of two original certificates of title (OCTs), docketed under the original GLRO Record Nos. 6908 and 6909.

Teofilo’s petition was opposed by the Republic, National Steel Corporation (NSC), and the City of Iligan.

Acting on the motion for judgment on demurrer to evidence filed by the Republic and NSC, the RTC initially dismissed Teofilo’s petition for reconstitution of titles because there was inadequate evidence to show the prior existence of the titles sought to be restored. According to the RTC, the proper remedy was a petition for the reconstitution of decrees since “it is undisputed that in Cases No. 6908 and 6909, Decrees No. 10364 and 18969, respectively, were issued.” Teofilo sought leave of court for the filing and admission of his amended petition, but the RTC refused. When elevated to this Court in Cacho v. Mangotara, docketed as G.R. No. 85495, the Court resolved to remand the case to the RTC, with an order to the said trial court to accept Teofilo’s amended petition and to hear it as one for re-issuance of decrees.

In opposing Teofilo’s petition, the Republic and NSC argued that the same suffered from jurisdictional infirmities; that Teofilo was not the real party-in-interest; that Teofilo was guilty of laches; that Doña Demetria was not the registered owner of the subject parcels of land; that no decrees were ever issued in Doña Demetria’s name; and that the issuance of the decrees was dubious and irregular.

After trial, on June 9, 1993, the RTC rendered its Decision granting Teofilo’s petition and ordering the reconstitution and re-issuance of Decree Nos. 10364 and 18969. The RTC held that the issuance of Decree No. 10364 in GLRO No. 6908 on May 9,

1913 and Decree No. 18969 in GLRO Record No. 6909 on July 8, 1915 was sufficiently established by the certifications and testimonies of concerned officials. The original issuance of these decrees presupposed a prior judgment that had become final.

On appeal, the Court of Appeals reversed the RTC Decision dated June 9, 1993 and dismissed the petition for re-issuance of Decree Nos. 10364 and 18969 because: (1) re-issuance of Decree No. 18969 in GLRO Record No. 6909 could not be made in the absence of the new survey ordered by this Court in the 1914 Cacho case; (2) the heir of a registered owner may lose his right to recover possession of the property and title thereto by laches; and (3) Teofilo failed to establish his identity and existence and that he was a real party-in-interest.

Teofilo then sought recourse from this Court in the 1997 Cacho case. The Court reversed the judgment of the Court of Appeals and reinstated the decision of the RTC approving the re-issuance of Decree Nos. 10364 and 18969. The Court found that such decrees had in fact been issued and had attained finality, as certified by the Acting Commissioner, Deputy Clerk of Court III, Geodetic Engineer, and Chief of Registration of the then Land Registration Commission, now National Land Titles and Deeds Registration Administration (NALTDRA). The Court further reasoned that:

[T]o sustain the Court of Appeals ruling as regards requiring petitioners to fulfill the conditions set forth in Cacho vs. U.S. would constitute a derogation of the doctrine of res judicata. Significantly, the issuance of the subject decrees presupposes a prior final judgment because the issuance of such decrees is a mere ministerial act on part of the Land Registration Commission (now the NALTDRA), upon presentation of a final judgment. It is also worth noting that the judgment in Cacho vs. U.S. could not have acquired finality without the prior fulfillment of the conditions in GLRO Record No. 6908, the presentation of the corresponding deed of sale from Datto Dorondon on or before March 30, 1913 (upon which Decree No. 10364 was issued on May 9, 1913); and in GLRO Record No. 6909, the presentation of a new survey per decision of Judge Jorge on December 10, 1912 and affirmed by this Court on December 10, 1914 (upon which Decree No. 18969 was issued on July 8, 1915).

Requiring the submission of a new plan as a condition for the re-issuance of the decree would render the finality attained by the Cacho vs. U.S. case nugatory, thus, violating the fundamental rule regarding res judicata. It must be stressed that the judgment and the resulting decree are res judicata, and these are binding upon the whole world, the proceedings being in the nature of proceedings in rem. Besides, such a requirement is an impermissible assault upon the integrity and stability of the Torrens System of registration because it also effectively renders the decree inconclusive.[21]

As to the issue of laches, the Court referred to the settled doctrine that laches cannot bar the issuance of a decree. A final decision in land registration cases can neither be rendered inefficacious by the statute of limitations nor by laches.

Anent the issue of the identity and existence of Teofilo and he being a real party-in-interest, the Court found that these were sufficiently established by the records. The Court relied on Teofilo’s Affidavit of Adjudication as Doña Demetria’s sole heir, which he executed before the Philippine Consulate General in Chicago,United States of America (U.S.A.); as well as the publication in the Times Journal of the fact of adjudication of Doña Demetria’s estate. Teofilo also appeared personally before the Vice Consul of the Philippine Consulate General in Chicago to execute a Special Power of Attorney in favor of Atty. Godofredo Cabildo (Atty. Cabildo) who represented him in this case. The Court stressed that the execution of public documents is entitled to the presumption of regularity and proof is required to assail and controvert the same.

In the Resolution dated July 28, 1997,[22] the Court denied the Motions for Reconsideration of the Republic and NSC.

As a result of the 1997 Cacho case, the decrees of registration were re-issued bearing new numbers and OCTs were issued for the two parcels of land in Doña Demetria’s name. OCT No. 0-1200 (a.f.) was based on re-issued Decree No. N-219464 in GLRO Record No. 6908, while OCT No. 0-1201 (a.f.) was based on re-issued Decree No. N-219465 in GLRO Record No. 6909.

20 | P a g e

Page 21: JURISDICTION-DIGESTED-OCHOA.docx

JURISDICTION CASE DIGEST TROPANG POTCHI

II

THE ANTECENT FACTS

OF THE PETITIONS AT BAR

The dispute over Lots 1 and 2 did not end with the termination of the 1997 Cacho case. Another four cases involving the same parcels of land were instituted before the trial courts during and after the pendency of the 1997 Cacho case. These cases are: (1) the Expropriation Case, G.R. No. 170375; (2) the Quieting of Title Case, G.R. Nos. 178779 and 178894; (3) the Ejectment or Unlawful Detainer Case, G.R. No. 170505 (execution pending appeal before the RTC) and G.R. Nos. 173355-56 and 173563-64 (execution pending appeal before the Court of Appeals); and (4) the Cancellation of Titles and Reversion Case, G.R. No. 173401. These cases proceeded independently of each other in the courts a quo until they reached this Court via the present Petitions. In the Resolution[23] dated October 3, 2007, the Court consolidated the seven Petitions considering that they either originated from the same case or involved similar issues.

Expropriation Case

(G.R. No. 170375)

The Complaint for Expropriation was originally filed on August 15, 1983 by the Iron and Steel Authority (ISA), now the NSC, against Maria Cristina Fertilizer Corporation (MCFC), and the latter’s mortgagee, the Philippine National Bank (PNB). The Complaint was docketed as Civil Case No. 106 and raffled to RTC-Branch 1, presided over by Judge Mangotara.

ISA was created pursuant to Presidential Decree No. 2729 [24] dated August 9, 1973, to strengthen, develop, and promote the iron and steel industry in thePhilippines. Its existence was extended until October 10, 1988.

On November 16, 1982, during the existence of ISA, then President Ferdinand E. Marcos issued Presidential Proclamation No. 2239,[25] reserving in favor of

ISA a parcel of land in Iligan City, measuring 302,532 square meters or 30.25 hectares, to be devoted to the integrated steel program of the Government. MCFC occupied certain portions of this parcel of land. When negotiations with MCFC failed, ISA was compelled to file a Complaint for Expropriation.

When the statutory existence of ISA expired during the pendency of Civil Case No. 106, MCFC filed a Motion to Dismiss the case alleging the lack of capacity to sue of ISA. The RTC-Branch 1 granted the Motion to Dismiss in an Order dated November 9, 1988. ISA moved for reconsideration or, in the alternative, for the substitution of the Republic as plaintiff in Civil Case No. 106, but the motion was denied by RTC-Branch 1. The dismissal of Civil Case No. 106 was affirmed by the Court of Appeals, thus, ISA appealed to this Court. In Iron and Steel Authority v. Court of Appeals[26] (ISA case), the Court remanded the case to RTC-Branch 1, which was ordered to allow the substitution of the Republic for ISA as plaintiff. Entry of Judgment was made in the ISA case on August 31, 1998. In an Order[27]dated November 16, 2001, the RTC-Branch 1 allowed the substitution of the Republic for ISA as plaintiff in Civil Case No. 106.

Alleging that Lots 1 and 2 involved in the 1997 Cacho case encroached and overlapped the parcel of land subject of Civil Case No. 106, the Republic filed with the RTC-Branch 1 a Motion for Leave to File Supplemental Complaint dated October 7, 2004 and to Admit the Attached Supplemental Complaint dated September 28, 2004[28] seeking to implead in Civil Case No. 106 Teofilo Cacho and Demetria Vidal and their respective successors-in-interest, LANDTRADE and AZIMUTH.

MCFC opposed the Motion for leave to file and to admit the Supplemental Complaint on the ground that the Republic was without legal personality to file the same because ISA was the plaintiff in Civil Case No. 106. MCFC argued that the Republic failed to move for the execution of the decision in the ISA case within the prescriptive period of five years, hence, the only remedy left was for the Republic to file an independent action to revive the judgment. MCFC further pointed out that the unreasonable delay of more than six years of the Republic in seeking the substitution and continuation of the action for expropriation effectively barred any further proceedings therein on the ground of estoppel by laches.

In its Reply, the Republic referred to the Order dated November 16, 2001 of the RTC-Branch 1 allowing the substitution of the Republic for ISA.

In an Order dated April 4, 2005, the RTC-Branch 1 denied the Motion of the Republic for leave to file and to admit its Supplemental Complaint. The RTC-Branch 1 agreed with MCFC that the Republic did not file any motion for execution of the judgment of this Court in the ISA case. Since no such motion for execution had been filed, the RTC-Branch 1 ruled that its Order dated November 16, 2001, which effected the substitution of the Republic for ISA as plaintiff in Civil Case No. 106, was an honest mistake. The Republic filed a Motion for Reconsideration of the April 4, 2005 Order of the RTC-Branch 1.

MCFC then filed a Motion to Dismiss Civil Case No. 106 for: (1) failure of the Republic to implead indispensable parties because MCFC insisted it was not the owner of the parcels of land sought to be expropriated; and (2) forum shopping considering the institution by the Republic on October 13, 2004 of an action for the reversion of the same parcels subject of the instant case for expropriation.

Judge Mangotara of RTC-Branch 1 issued a Resolution[29] on July 12, 2005, denying for lack of merit the Motion for Reconsideration of the Order dated April 4, 2005 filed by the Republic, and granting the Motion to Dismiss Civil Case No. 106 filed by MCFC. Judge Mangotara justified the dismissal of the Expropriation Case thus:

What the Republic seeks [herein] is the expropriation of the subject parcels of land. Since the exercise of the power of eminent domain involves the taking of private lands intended for public use upon payment of just compensation to the owner x x x, then a complaint for expropriation must, of necessity, be directed against the owner of the land subject thereof. In the case at bar, the decision of the Supreme Court in Cacho v. Government of the United States x x x, decreeing the registration of the subject parcels of land in the name of the late Doña Demetria Cacho has long attained finality and is conclusive as to the question of ownership thereof. Since MCFC, the only defendant left in this case, is not a proper party defendant in this complaint for expropriation, the present case should be dismissed.

This Court notes that the Republic [has filed reversion proceedings] dated September 27, 2004, involving the same parcels of land, docketed as Case No. 6686 pending before the Regional Trial Court of Lanao del Norte, Iligan City Branch 4. [The

21 | P a g e

Page 22: JURISDICTION-DIGESTED-OCHOA.docx

JURISDICTION CASE DIGEST TROPANG POTCHI

Republic], however, did not state such fact in its “Verification and Certification of Non-Forum Shopping” attached to its Supplemental Complaint dated September 28, 2004. [It is therefore] guilty of forum shopping. Moreover, considering that in the Reversion case, [the Republic] asserts ownership over the subject parcels of land, it cannot be allowed to take an inconsistent position in this expropriation case without making a mockery of justice.[30]

The Republic filed a Motion for Reconsideration of the Resolution dated July 12, 2005, insofar as it dismissed Civil Case No. 106, but said Motion was denied by Judge Mangatora in a Resolution[31] dated October 24, 2005.

On January 16, 2006, the Republic filed with this Court the consolidated Petition for Review on Certiorari and Petition for Certiorari under Rules 45 and 65 of the Rules of Court, respectively, docketed as G.R. No. 170375.

The Quieting of Title Case

(G.R. Nos. 178779 and 178894)

Demetria Vidal (Vidal) and AZIMUTH filed on November 18, 1998, a Petition[32] for Quieting of Title against Teofilo, Atty. Cabildo, and the Register of Deeds of Iligan City, which was docketed as Civil Case No. 4452 and raffled to RTC-Branch 3.

In the Petition, Vidal claimed that she, and not Teofilo, was the late Doña Demetria’s sole surviving heir, entitled to the parcels of land covered by OCT Nos. 0-1200 (a.f.) and 0-1201 (a.f.). She averred that she is the daughter of Francisco Cacho Vidal (Francisco) and Fidela Arellano Confesor. Francisco was the only child of Don Dionisio Vidal and Doña Demetria.

AZIMUTH, for its part, filed the Petition as Vidal’s successor-in-interest with respect to a 23-hectare portion of the subject parcels of land pursuant to the

Memorandum of Agreement dated April 2, 1998 and Deed of Conditional Conveyance dated August 13, 2004, which Vidal executed in favor of AZIMUTH.

Teofilo opposed the Petition contending that it stated no cause of action because there was no title being disturbed or in danger of being lost due to the claim of a third party, and Vidal had neither legal nor beneficial ownership of the parcels of land in question; that the matter and issues raised in the Petition had already been tried, heard, and decided by the RTC of Iligan City and affirmed with finality by this Court in the 1997 Cacho case; and that the Petition was barred by the Statute of Limitations and laches.

LANDTRADE, among other parties, was allowed by the RTC-Branch 3 to intervene in Civil Case No. 4452. LANDTRADE alleged that it is the owner of a portion of the subject parcels of land, measuring 270,255 square meters or about 27.03 hectares, which it purportedly acquired through a Deed of Absolute Sale dated October 1, 1996 from Teofilo, represented by Atty. Cabildo. LANDTRADE essentially argued that Vidal's right as heir should be adjudicated upon in a separate and independent proceeding and not in the instant Quieting of Title Case.

During the pre-trial conference, the parties manifested that there was no possibility of any amicable settlement among them.

Vidal and AZIMUTH submitted testimonial and documentary evidence during the trial before the RTC-Branch 3. Teofilo and Atty. Cabildo failed to present any evidence as they did not appear at all during the trial, while LANDTRADE was declared by the RTC-Branch 3 to have waived its right to present evidence on its defense and counterclaim.

On July 17, 2004, the RTC-Branch 3 rendered its Decision [33] in Civil Case No. 4452 in favor of Vidal and AZIMUTH, the dispositive portion of which reads:

WHEREFORE, judgment is hereby rendered in favor of the petitioners and against the respondents and intervenors:

1) DECLARING:

a.) Petitioner Demetria C. Vidal the sole surviving heir of the late Doña Demetria Cacho;

b.) Petitioner Demetria C. Vidal alone has the hereditary right to and interest in the Subject Property;

c.) Petitioner Azimuth International Development Corporation is the successor-in-interest of petitioner Demetria C. Vidal to a portion of the Subject Property to the extent provided in their 2 April 1998 Memorandum of Agreement and 13 August 1998 Deed of Conditional Conveyance;

d.) Respondent Teofilo Cacho is not a son or heir of the late Dona Demetria Cacho; and

e.) Respondent Teofilo Cacho, Godofredo Cabildo and any of their transferees/assignees have no valid right to or interest in the Subject Property.

2) ORDERING:

a.) Respondent Register of Deeds of Iligan City, and any other person acting in his behalf, stop, cease and desist:

3) ORDERING respondents Teofilo Cacho and Atty. Godofredo Cabildo to pay petitioners, jointly and severally, the following:

22 | P a g e

Page 23: JURISDICTION-DIGESTED-OCHOA.docx

JURISDICTION CASE DIGEST TROPANG POTCHI

a) For temperate damages - P 80,000.00

b) For nominal damages - P 60,000.00

c) For moral damages - P500,000.00

d) For exemplary damages - P 500,000.00

e) For attorney's fees (ACCRA Law)-P1,000,000.00

f) For Attorney's fees - P500,000.00

(Atty. Voltaire Rovira)

g) For litigation expenses - P300,000.00

For lack of factual and legal basis, the counterclaim of Teofilo Cacho and Atty. Godofredo Cabildo is hereby dismissed.

Likewise, the counterclaim of intervenor IDD/Investa is dismissed for lack of basis as the petitioners succeeded in proving their cause of action.

On the cross-claim of intervenor IDD/Investa, respondents Teofilo Cacho and Atty. Godofredo Cabildo are ORDERED to pay IDD/Investa, jointly and severally, the principal sum of P5,433,036 with 15% interest per annum.

For lack of legal basis, the counterclaim of Intervenor Landtrade Realty Development Corporation is dismissed.

Likewise, Intervenor Manguera's counterclaim is dismissed for lack of legal basis.[34]

The joint appeal filed by LANDTRADE, Teofilo, and Atty. Cabildo with the Court of Appeals was docketed as CA-G.R. CV No. 00456. The Court of Appeals, in its Decision[35] of January 19, 2007, affirmed in toto the Decision dated July 17, 2004 of the RTC-Branch 3.

According to the Court of Appeals, the RTC-Branch 3 did not err in resolving the issue on Vidal’s status, filiation, and hereditary rights as it is determinative of the issue on ownership of the subject properties. It was indubitable that the RTC-Branch 3 had jurisdiction over the person of Teofilo and juridical personality of LANDTRADE as they both filed their Answers to the Petition for Quieting of Title thereby voluntarily submitting themselves to the jurisdiction of said trial court. Likewise, the Petition for Quieting of Title is in itself within the jurisdiction of the RTC-Branch 3. Hence, where there is jurisdiction over the person and subject matter, the resolution of all other questions arising in the case is but an exercise by the court of its jurisdiction. Moreover, Teofilo and LANDTRADE were guilty of estoppel by laches for failing to assail the jurisdiction of the RTC-Branch 3 at the first opportunity and even actively participating in the trial of the case and seeking affirmative reliefs.

In addition, the Court of Appeals held that the 1997 Cacho case only determined the validity and efficacy of the Affidavit of Adjudication that Teofilo executed before the Philippine Consulate General in the U.S.A. The decision of this Court in the 1997 Cacho case, which had become final and executory, did not vest upon Teofilo ownership of the parcels of land as it merely ordered the re-issuance of a lost duplicate certificate of title in its original form and condition.

The Court of Appeals agreed in the finding of the RTC-Branch 3 that the evidence on record preponderantly supports Vidal’s claim of being the granddaughter and sole heiress of the late Doña Demetria. The appellate court further adjudged that Vidal did not delay in asserting her rights over the subject parcels of land. The prescriptive period for real actions over immovables is 30 years. Vidal’s rights as Doña Demetria’s successor-in-interest accrued upon the latter’s death in 1974, and only 24 years thereafter, in 1998, Vidal already filed the present Petition for Quieting of Title. Thus, Vidal’s cause of action had not yet prescribed. And, where the action was

filed within the prescriptive period provided by law, the doctrine of laches was also inapplicable.

LANDTRADE, Teofilo, and Atty. Cabildo filed separate Motions for Reconsideration of the January 19, 2007 Decision of the Court of Appeals, which were denied in the July 4, 2007 Resolution[36] of the same court.

On August 24, 2007, LANDTRADE filed with this Court a Petition for Review on Certiorari under Rule 45 of the Rules of Court, which was docketed as G.R. No. 178779. On September 6, 2007, Teofilo and Atty. Cabildo filed their own Petition for Review on Certiorari under Rule 45 of the Rules of Court, which was docketed as G.R. No. 178894.

The Ejectment or Unlawful Detainer Case

(G.R. Nos. 170505, 173355-56, and 173563-64)

Three Petitions before this Court are rooted in the Unlawful Detainer Case instituted by LANDTRADE against NAPOCOR and TRANSCO.

On August 9, 1952, NAPOCOR took possession of two parcels of land in Sitio Nunucan, Overton, Fuentes, Iligan City, denominated as Lots 2029 and 2043, consisting of 3,588 square meters (or 0.36 hectares) and 3,177 square meters (or 0.32 hectares), respectively. On Lot 2029, NAPOCOR constructed its power sub-station, known as the Overton Sub-station, while on Lot 2043, it built a warehouse, known as the Agus 7 Warehouse, both for the use of its Agus 7 Hydro-Electric Power Plant. For more than 30 years, NAPOCOR occupied and possessed said parcels of land pursuant to its charter, Republic Act No. 6395.[37] With the enactment in 2001 of Republic Act No. 9136, otherwise known as the Electric Power Industry Reform Act (EPIRA), TRANSCO assumed the functions of NAPOCOR with regard to electrical transmissions and took over possession of the Overton Sub-station.

Claiming ownership of the parcels of land where the Overton Sub-station and Agus 7 Warehouse are located, LANDTRADE filed with the MTCC on April 9, 2003 a

23 | P a g e

Page 24: JURISDICTION-DIGESTED-OCHOA.docx

JURISDICTION CASE DIGEST TROPANG POTCHI

Complaint for Unlawful Detainer against NAPOCOR and TRANSCO, which was docketed as Civil Case No. 11475-AF.

In its Complaint, LANDTRADE alleged that it acquired from Teofilo, through Atty. Cabildo, two parcels of land at Sitio Nunucan, Overton, Fuentes, Brgy. Maria Cristina, Iligan City, with a combined area of 270,255 square meters or around 27.03 hectares, as evidenced by a Deed of Absolute Sale[38] dated October 1, 1996. Certain portions of said parcels of land were being occupied by the Overton Sub-station and Agus 7 Warehouse of NAPOCOR and TRANSCO, through the tolerance of LANDTRADE. Upon failure of NAPOCOR and TRANSCO to pay rentals or to vacate the subject properties after demands to do so, LANDTRADE filed the present Complaint for Unlawful Detainer, plus damages in the amount of P450,000.00 as yearly rental from date of the first extra-judicial demand until NAPOCOR and TRANSCO vacate the subject properties.

In their separate Answers, NAPOCOR and TRANSCO denied the material allegations in the Complaint and countered, by way of special and affirmative defenses, that the Complaint was barred by res judicata; that the MTCC has no jurisdiction over the subject matter of the action; and that LANDTRADE lacked the legal capacity to sue.

On February 17, 2004, the MTCC rendered its Decision [39] in favor of LANDTRADE. The MTCC disposed:

WHEREFORE, premises considered, judgment is hereby rendered in favor of Plaintiff Land Trade Realty Corporation represented by Atty. Max C. Tabimina and against defendant National Power Corporation represented by its President, Mr. Rogelio M. Murga and co-defendant TRANSCO represented by its President Dr. Allan T. Ortiz and Engr. Lorrymir A. Adaza, Manager, NAPOCOR-Mindanao, Regional Center, Ma. Cristina, Iligan City, ordering:

1. Defendants National Power Corporation and TRANSCO, their agents or representatives or any person/s acting on its behalf or under its authority to vacate the premises;

2. Defendants NAPOCOR and TRANSCO to pay Plaintiff jointly and solidarily:

a. Php500,000.00 a month representing fair rental value or compensation since June 29, 1978 until defendant shall have vacated the premises;

b. Php20,000.00 for and as attorney’s fees and

c. Cost of suit.

Execution shall issue immediately upon motion, unless an appeal has been perfected and the defendant to stay execution files a sufficient supersedeas bond, approved by this Court and executed in favor of the plaintiff, to pay the rents, damages, and costs accruing down to the time of judgment appealed from, and unless, during the pendency of the appeal, defendants deposit with the appellate court the amount of P500,000.00 per month, as reasonable value of the use and occupancy of the premises for the preceding month or period on or before the tenth day of each succeeding month or period.[40]

NAPOCOR and TRANSCO seasonably filed a Joint Notice of Appeal. Their appeal, docketed as Civil Case No. 6613, was initially assigned to the RTC-Branch 5, presided over by Judge Maximino Magno Libre (Judge Libre).

LANDTRADE filed on June 24, 2004 a Motion for Execution, asserting that NAPOCOR and TRANSCO had neither filed a supersedeas bond with the MTCC nor periodically deposited with the RTC the monthly rental for the properties in question, so as to stay the immediate execution pending appeal of the MTCC judgment. However, the said Motion failed to comply with the required notice of hearing under Rule 15, Section 5 of the Rules of Court. LANDTRADE then filed a Motion to Withdraw and/or Replace Notice of Hearing.

NAPOCOR and TRANSCO filed on July 13, 2004 a Joint Motion to Suspend Proceedings citing Amagan v. Marayag,[41] in which the Court ruled that if circumstances should require, the proceedings in an ejectment case may be suspended in whatever stage it may be found. Since LANDTRADE anchors its right to possession of the subject parcels of land on the Deed of Sale executed in its favor by Teofilo on October 1, 1996, the ejectment case should be held in abeyance pending the resolution of other cases in which title over the same properties are in issue, i.e., (1) Civil Case No. 6600, the action for the annulment of the Deed of Sale dated October 1, 1996 filed by Teofilo against LANDTRADE pending before the RTC-Branch 4; and (2) Civil Case No. 4452, the Quieting of Title Case filed by Vidal and AZIMUTH against Teofilo and Atty. Cabildo pending before the RTC-Branch 3.

LANDTRADE filed on July 19, 2004 another Motion for Execution, which was heard together with the Joint Motion to Suspend Proceedings of NAPOCOR and TRANSCO. After said hearing, the RTC-Branch 5 directed the parties to file their memoranda on the two pending Motions.

LANDTRADE, in its Memorandum, maintained that the pendency of Civil Case No. 4452, the Quieting of Title Case, should not preclude the execution of the MTCC judgment in the Unlawful Detainer Case because the issue involved in the latter was only the material possession or possession de facto of the parcels of land in question. LANDTRADE also reported that Civil Case No. 6600, the action for annulment of the Deed of Sale dated October 1, 1996 instituted by Teofilo, was already dismissed given that the RTC-Branch 4 had approved the Compromise Agreement executed between LANDTRADE and Teofilo.

NAPOCOR and TRANSCO likewise filed their respective Memoranda. Subsequently, NAPOCOR filed a Supplement to its Memorandum to bring to the attention of the RTC-Branch 5 the Decision rendered on July 17, 2004 by the RTC-Branch 3 in Civil Case No. 4452, the Quieting of Title Case, categorically declaring Teofilo, the predecessor-in-interest of LANDTRADE, as having no right at all to the subject parcels of land. Resultantly, the right of LANDTRADE to the two properties, which merely emanated from Teofilo, was effectively declared as non-existent too.

On August 4, 2004, the RTC-Branch 5 issued an Order [42] denying the Joint Motion to Suspend Proceedings of NAPOCOR and TRANSCO. The RTC held that the pendency of other actions involving the same parcels of land could not stay execution pending appeal of the MTCC judgment because NAPOCOR and TRANSCO failed to post the required bond and pay the monthly rentals.

24 | P a g e

Page 25: JURISDICTION-DIGESTED-OCHOA.docx

JURISDICTION CASE DIGEST TROPANG POTCHI

Five days later, on August 9, 2004, the RTC-Branch 5 issued another Order[43] granting the Motion of LANDTRADE for execution of the MTCC judgment pending appeal.

The next day, on August 10, 2004, the Acting Clerk of Court, Atty. Joel M. Macaraya, Jr., issued a Writ of Execution Pending Appeal[44] which directed Sheriff IV Alberto O. Borres (Sheriff Borres) to execute the MTCC Decision dated February 17, 2004.

A day later, on August 11, 2004, Sheriff Borres issued two Notices of Garnishment[45] addressed to PNB and Land Bank of the Philippines in Iligan City, garnishing all the goods, effects, stocks, interests in stocks and shares, and any other personal properties belonging to NAPOCOR and TRANSCO which were being held by and under the possession and control of said banks. On even date, Sheriff Borres also issued a Notification[46] to NAPOCOR and TRANSCO for them to vacate the subject parcels of land; and to pay LANDTRADE the sums of (a) P156,000,000.00, representing the total fair rental value for the said properties, computed atP500,000.00 per month, beginning June 29, 1978 until June 29, 2004, or for a period of 26 years, and (b) P20,000.00 as attorney's fees.

Thereafter, NAPOCOR and TRANSCO each filed before the Court of Appeals in Cagayan de Oro City a Petition for Certiorari, under Rule 65 of the Rules of Court, with prayer for the issuance of a TRO and writ of preliminary injunction. The Petitions, docketed as CA-G.R. SP Nos. 85174 and 85841, were eventually consolidated.

The Court of Appeals issued on August 18, 2004 a TRO [47] enjoining the enforcement and implementation of the Order of Execution and Writ of Execution Pending Appeal of the RTC-Branch 5 and Notices of Garnishment and Notification of Sheriff Borres.

The Court of Appeals, in its Decision[48] dated November 23, 2005, determined that public respondents did commit grave abuse of discretion in allowing and/or effecting the execution of the MTCC judgment pending appeal, since NAPOCOR and TRANSCO were legally excused from complying with the requirements for a stay of

execution specified in Rule 70, Section 19 of the Rules of Court, particularly, the posting of a supersedeas bond and periodic deposits of rental payments. The decretal portion of said appellate court Decision states:

ACCORDINGLY, the two petitions at bench are GRANTED; the Order dated 9 August 2004, the Writ of Execution Pending Appeal dated 10 August 2004, the two Notices of Garnishment dated 11 August 2004, and the Notification dated 11 August 2004, are ANNULLED and SET ASIDE.[49]

Displeased, LANDTRADE elevated the case to this Court on January 10, 2006 via a Petition for Review on Certiorari under Rule 45 of the Rules of Court, which was docketed as G.R. No. 170505.

In the meantime, with the retirement of Judge Libre and the inhibition[50] of Judge Oscar Badelles, the new presiding judge of RTC-Branch 5, Civil Case No. 6613 was re-raffled to the RTC-Branch 1, presided over by Judge Mangotara. The RTC-Branch 1 promulgated on December 12, 2005 a Decision[51] in Civil Case No. 6613 which affirmed in toto the February 17, 2004 Decision of the MTCC in Civil Case No. 11475-AF favoring LANDTRADE.

NAPOCOR and TRANSCO filed with the RTC-Branch 1 twin Motions, namely: (1) Motion for Reconsideration of the Decision dated December 12, 2005; and (2) Motion for Inhibition of Judge Mangotara. The RTC-Branch 1 denied both Motions in a Resolution dated January 30, 2006.

NAPOCOR and TRANSCO filed with the Court of Appeals separate Petitions for Review with prayer for TRO and/or a writ of preliminary injunction, which were docketed as CA-G.R. SP Nos. 00854 and 00889, respectively. In a Resolution dated March 24, 2006, the Court of Appeals granted the prayer for TRO of NAPOCOR and TRANSCO.

With the impending lapse of the effectivity of the TRO on May 23, 2006, NAPOCOR filed on May 15, 2006 with the Court of Appeals a Manifestation and Motion praying for the resolution of its application for preliminary injunction.

On May 23, 2006, the same day the TRO lapsed, the Court of Appeals granted the motions for extension of time to file a consolidated comment of LANDTRADE. Two days later, LANDTRADE filed an Omnibus Motion seeking the issuance of (1) a writ of execution pending appeal, and (2) the designation of a special sheriff in accordance with Rule 70, Section 21 of the Rules of Court.

In a Resolution[52] dated June 30, 2006, the Court of Appeals granted the Omnibus Motion of LANDTRADE and denied the applications for the issuance of a writ of preliminary injunction of NAPOCOR and TRANSCO. In effect, the appellate court authorized the execution pending appeal of the judgment of the MTCC, affirmed by the RTC-Branch 1, thus:

IN LIGHT OF THE ABOVE DISQUISITIONS, this Court resolves to grant the [LANDRADE]’s omnibus motion for execution pending appeal of the decision rendered in its favor which is being assailed in these consolidated petitions for review. Accordingly, the [NAPOCOR and TRANSCO’s] respective applications for issuance of writ of preliminary injunction are both denied for lack of factual and legal bases. The Municipal Trial Court in Cities, Branch 2, Iligan City, which at present has the custody of the records of the case a quo, is hereby ordered to cause the immediate issuance of a writ of execution relative to its decision dated 17 February 2004 in Civil Case No. 11475-AF.[53]

On July 20, 2006, NAPOCOR filed with this Court a Petition for Certiorari and Prohibition under Rule 65 of the Rules of Court with an urgent plea for a TRO, docketed as G.R. No. 173355-56. On August 2, 2006, TRANSCO filed with this Court its own Petition for Certiorari, docketed as G.R. No. 173563-64.

25 | P a g e

Page 26: JURISDICTION-DIGESTED-OCHOA.docx

JURISDICTION CASE DIGEST TROPANG POTCHI

On July 21, 2006, NAPOCOR filed an Urgent Motion for the Issuance of a TRO in G.R. No. 173355-56. In a Resolution[54] dated July 26, 2006, the Court granted the Motion of NAPOCOR and issued a TRO,[55] effective immediately, which enjoined public and private respondents from implementing the Resolution dated June 30, 2006 of the Court of Appeals in CA-G.R. SP Nos. 00854 and 00889 and the Decision dated February 17, 2004 of the MTCC in Civil Case No. 11475-AF.

On July 31, 2006, Vidal and AZIMUTH filed a Motion for Leave to Intervene and to Admit Attached Comment-in-Intervention, contending therein that Vidal was the lawful owner of the parcels of land subject of the Unlawful Detainer Case as confirmed in the Decision dated July 17, 2004 of the RTC-Branch 3 in Civil Case No. 4452. In a Resolution dated September 30, 2006, the Court required the parties to comment on the Motion of Vidal and AZIMUTH, and deferred action on the said Motion pending the submission of such comments.

The Cancellation of Titles and Reversion Case

(G.R. No. 173401)

On October 13, 2004, the Republic filed a Complaint for the Cancellation of OCT Nos. 0-1200 (a.f.) and 0-1201 (a.f.) and Reversion against the late Doña Demetria, represented by her alleged heirs, Vidal and/or Teofilo, together with AZIMUTH and LANDTRADE. The Complaint, docketed as Civil Case No. 6686, was raffled to the RTC-Branch 4.

The Republic sought the cancellation of OCT Nos. 0-1200 (a.f.) and 0-1201 (a.f.) and the reversion of the parcels of land covered thereby to the Government based on the following allegations in its Complaint, under the heading “Cause of Action”:

5. On October 15, 1998, Original Certificates of Title (OCTs) Nos. 0-1200 (a.f.) and 0-1201 (a.f.) were issued in the name of “Demetria Cacho, widow, now deceased…” consisting of a total area of Three Hundred Seventy-Eight Thousand Seven Hundred and Seven (378,707) square meters and Three Thousand

Seven Hundred Thirty-Five (3,635) square meters, respectively, situated in Iligan City, x x x

x x x x

6. The afore-stated titles were issued in implementation of a decision rendered in LRC (GLRO) Record Nos. 6908 and 6909 dated December 10, 1912, as affirmed by the Honorable Supreme Court in Cacho v. Government of the United States, 28 Phil. 616 (December 10, 1914),

7. The decision in LRC (GLRO) Record Nos. 6908 and 6909, upon which the titles were issued, did not grant the entire area applied for therein. x x x

x x x x

9. As events turned out, the titles issued in connection with LRC (GLRO) Record Nos. 6908 and 6909 – i.e. OCT Nos. 0-1200 (a.f.) and 0-1201 (a.f.) – cover property MUCH LARGER in area than that granted by the land registration court in its corresponding decision, supra.

10. While the LRC Decision, as affirmed by the Honorable Supreme Court, granted only the southern part of the 37.87 hectare land subject of LRC (GLRO) Record Case No. 6909, the ENTIRE 37.87 hectares is indicated as the property covered by OCT 0-1200 (a.f.). Worse, OCT No. 0-1200 (a.f.) made reference to Case No. 6908 as basis thereof, yet, the decision in said case is clear:

(i) The parcel “object of Case No. 6908 is small” (Cacho vs. Government of the United States, 28 Phil. 616, p. 619)

(ii) “The parcel of land claimed by the applicant in Case No. 6909 is the bigger of two parcels and contains 37.87 hectares…”

11. More significantly, the technical description in Original Certificate of Title No. 0-1200 (a.f.) specifies the date of survey as “August 31 to September 1, 1910,” which is EARLIER than the date the Supreme Court, in Cacho supra, resolved LRC (GLRO) Record No. 6909 (involving 37.87 hectares). In resolving the application involving the 37.87 hectares, the Honorable Supreme Court declared that only the southern part of the 37.87 hectare property applied for is granted and that a new survey specifying the “southern part” thereof should be submitted. Accordingly, any survey involving the “granted southern part” should bear a date subsequent to the December 10, 1914 Supreme Court decision. x x x

x x x x

12. The Honorable Supreme Court further declared that the Decision in LRC (GLRO) Record No. 6909 was reserved:

“Final decision in these case is reserved until the presentation of the … new plan.” (28 Phil. 616, p. 631; Underscoring supplied)

In other words, as of December 10, 1914, when the Honorable Supreme Court rendered its Decision on appeal in LRC (GLRO) Record No. 6909, “final decision” of the case was still reserved

26 | P a g e

Page 27: JURISDICTION-DIGESTED-OCHOA.docx

JURISDICTION CASE DIGEST TROPANG POTCHI

until the presentation of a new plan. The metes and bounds of OCT No. 0-1200 (a.f.) could not have been the technical description of the property granted by the court – described as “the southern part of the large parcel object of expediente 6909 only” (Cacho vs. Government of the United States, 28 Phil. 617, 629). As earlier stated, the technical description appearing in said title was the result of a survey conducted in 1910 or before the Supreme Court decision was rendered in 1914.

13. In the same vein, Original Certificate of Title No. 0-1201 (a.f.) specifies LRC (GLRO) Record No. 6909 as the basis thereof (see front page of OCT No. 0-1201 (a.f.)). Yet, the technical description makes, as its reference, Lot 1, Plan II-3732, LR Case No. 047, LRC (GLRO) Record No. 6908 (see page 2 of said title). A title issued pursuant to a decision may only cover the property subject of the case. A title cannot properly be issued pursuant to a decision in Case 6909, but whose technical description is based on Case 6908.

14. The decision in LRC (GLRO) Record Nos. 6908 and 6909 has become final and executory, and it cannot be modified, much less result in an increased area of the property decreed therein.

x x x x

16. In sum, Original Certificates of Title Nos. 0-1200 (a.f.) and 0-1201 (a.f.), as issued, are null and void since the technical descriptions vis-à-vis the areas of the parcels of land covered therein went beyond the areas granted by the land registration court in LRC (GLRO) Record Nos. 6908 and 6909.[56]

Vidal and AZIMUTH filed a Motion to Dismiss dated December 23, 2004 on the grounds that (1) the Republic has no cause of action; (2)

assuming arguendothat the Republic has a cause of action, its Complaint failed to state a cause of action; (3) assuming arguendo that the Republic has a cause of action, the same is barred by prior judgment; (4) assuming further that the Republic has a cause of action, the same was extinguished by prescription; and (4) the Republic is guilty of forum shopping.

Upon motion of the Republic, the RTC-Branch 4 issued an Order [57] dated October 4, 2005, declaring LANDTRADE and Teofilo, as represented by Atty. Cabildo, in default since they failed to submit their respective answers to the Complaint despite the proper service of summons upon them.

LANDTRADE subsequently filed its Answer with Compulsory Counterclaim dated September 28, 2005. It also moved for the setting aside and reconsideration of the Order of Default issued against it by the RTC-Branch 4 on October 20, 2005.

On December 13, 2005, the RTC-Branch 4 issued an Order[58] dismissing the Complaint of the Republic in Civil Case No. 6686, completely agreeing with Vidal and AZIMUTH.

The RTC-Branch 4 reasoned that the Republic had no cause of action because there was no showing that the late Doña Demetria committed any wrongful act or omission in violation of any right of the Republic. Doña Demetria had sufficiently proven her ownership over the parcels of land as borne in the ruling of the LRC in GLRO Record Nos. 6908 and 6909. On the other hand, the Republic had no more right to the said parcels of land. The Regalian doctrine does not apply in this case because the titles were already issued to Doña Demetria and segregated from the mass of the public domain.

The RTC-Branch 4 likewise held that the Republic failed to state a cause of action in its Complaint. The arguments of the Republic – i.e., the absence of a new survey plan and deed, the titles covered properties with much larger area than that granted by the LRC – had been answered squarely in the 1997 Cacho case. Also, the Complaint failed to allege that fraud had been committed in having the titles registered and that the Director of Lands requested the reversion of the subject parcels of land.

The RTC-Branch 4 was convinced that the Complaint was barred by res judicata because the 1914 Cacho case already decreed the registration of the parcels of land in the late Doña Demetria’s name and the 1997 Cacho case settled that there was no merit in the argument that the conditions imposed in the first case have not been complied with.

The RTC-Branch 4 was likewise persuaded that the cause of action or remedy of the Republic was lost or extinguished by prescription pursuant to Article 1106 of the Civil Code and Section 32 of Presidential Decree No. 1529, otherwise known as the Land Registration Decree, which prescribes a one-year period within which to file an action for the review of a decree of registration.

Finally, the RTC-Branch 4 found the Republic guilty of forum shopping because there is between this case, on one hand, and the 1914 and 1997 Cacho cases, on the other, identity of parties, as well as rights asserted and reliefs prayed for, as the contending parties are claiming rights of ownership over the same parcels of land.

The Republic filed a Motion for Reconsideration of the dismissal of its Complaint but the same was denied by the RTC-Branch 4 in its Order [59] dated May 16, 2006.

Assailing the Orders dated December 13, 2005 and May 16, 2006 of the RTC-Branch 4, the Republic filed on August 11, 2006 a Petition for Review onCertiorari under Rule 45 of the Rules of Court, which was docketed as G.R. No. 173401.

III

ISSUES AND DISCUSSIONS

Expropriation Case

(G.R. No. 170375)

27 | P a g e

Page 28: JURISDICTION-DIGESTED-OCHOA.docx

JURISDICTION CASE DIGEST TROPANG POTCHI

The Republic, in its consolidated Petitions challenging the Resolutions dated July 12, 2005 and October 24, 2005 of the RTC-Branch 1 in Civil Case No. 106, made the following assignment of errors:

RESPONDENT JUDGE GRAVELY ERRED IN ORDERING THE DISMISSAL OF THE EXPROPRIATION COMPLAINT IN CIVIL CASE NO. 106 CONSIDERING THAT:

(a) THE NON-JOINDER OF PARTIES IS NOT A GROUND FOR THE DISMISSAL OF AN ACTION PURSUANT TO SECTION 11, RULE 3 OF THE 1997 RULES OF CIVIL PROCEDURE;

(b) AN EXPROPRIATION PROCEEDING IS AN ACTION QUASI IN REM WHEREIN THE FACT THAT THE OWNER OF THE PROPERTY IS MADE A PARTY TO THE ACTION IS NOT ESSENTIALLY INDISPENSABLE;

(c) PETITIONER DID NOT COMMIT ANY FORUM SHOPPING WITH THE FILING OF THE REVERSION COMPLAINT DOCKETED AS CIVIL CASE NO. 6686 WHICH IS PENDING BEFORE BRANCH 4 OF THE REGIONAL TRIAL COURT OF ILIGAN CITY.

At the outset, the Court notes that the Republic filed a pleading with the caption Consolidated Petitions for Review on Certiorari (Under Rule 45) and Certiorari (Under Rule 65) of the Rules of Court. The Republic explains that it filed the Consolidated Petitions pursuant to Metropolitan Waterworks and Sewerage System (MWSS) v. Court of Appeals[61] (MWSS case).

The reliance of the Republic on the MWSS case to justify its mode of appeal is misplaced, taking the pronouncements of this Court in said case out of context.

The issue in the MWSS case was whether a possessor in good faith has the right to remove useful improvements, and not whether consolidated petitions under both Rules 45 and 65 of the Rules of Court can be filed. Therein petitioner MWSS simply filed an appeal by certiorari under Rule 45 of the Rules of Court, but named the Court of Appeals as a respondent. The Court clarified that the only parties in an appeal by certiorari under Rule 45 of the Rules of Court are the appellant as petitioner and the appellee as respondent. The court which rendered the judgment appealed from is not a party in said appeal. It is in the special civil action ofcertiorari under Rule 65 of the Rules of Court where the court or judge is required to be joined as party defendant or respondent. The Court, however, also acknowledged that there may be an instance when in an appeal by certiorari under Rule 45, the petitioner-appellant would also claim that the court that rendered the appealed judgment acted without or in excess of its jurisdiction or with grave abuse of discretion, in which case, such court should be joined as a party-defendant or respondent. While the Court may have stated that in such an instance, “the petition for review on certiorari under Rule 45 of the Rules of Court is at the same time a petition for certiorari under Rule 65,” the Court did not hold that consolidated petitions under both Rules 45 and 65 could or should be filed.

The Court, in more recent cases, had been stricter and clearer on the distinction between these two modes of appeal. In Nunez v. GSIS Family Bank,[62] the Court elucidated:

In Ligon v. Court of Appeals where the therein petitioner described her petition as “an appeal under Rule 45 and at the same time as a special civil action of certiorari under Rule 65 of the Rules of Court,” this Court, in frowning over what it described as a “chimera,” reiterated that the remedies of appeal and certiorari are mutually exclusive and not alternative nor successive.

To be sure, the distinctions between Rules 45 and 65 are far and wide. However, the most apparent is that errors of jurisdiction are best reviewed in a special civil action for certiorari under Rule 65 while errors of judgment can only be corrected by appeal in a petition for review under Rule 45.

But in the same case, the Court also held that:

This Court, x x x, in accordance with the liberal spirit which pervades the Rules of Court and in the interest of justice may treat a petition for certiorari as having been filed under Rule 45, more so if the same was filed within the reglementary period for filing a petition for review.[63]

It is apparent in the case at bar that the Republic availed itself of the wrong mode of appeal by filing Consolidated Petitions for Review under Rule 45 and forCertiorari under Rule 65, when these are two separate remedies that are mutually exclusive and neither alternative nor successive. Nevertheless, the Court shall treat the Consolidated Petitions as a Petition for Review on Certiorari under Rule 45 and the allegations therein as errors of judgment. As the records show, the Petition was filed on time under Rules 45. Before the lapse of the 15-day reglementary period to appeal under Rule 45, the Republic filed with the Court a motion for extension of time to file its petition. The Court, in a Resolution[64] dated January 23, 2006, granted the Republic a 30-day extension, which was to expire on December 29, 2005. The Republic was able to file its Petition on the last day of the extension period.

Hierarchy of courts

The direct filing of the instant Petition with this Court did not violate the doctrine of hierarchy of courts.

According to Rule 41, Section 2(c)[65] of the Rules of Court, a decision or order of the RTC may be appealed to the Supreme Court by petition for review oncertiorari under Rule 45, provided that such petition raises only questions of law.[66]

A question of law exists when the doubt or controversy concerns the correct application of law or jurisprudence to a certain set of facts; or when the issue does not

28 | P a g e

Page 29: JURISDICTION-DIGESTED-OCHOA.docx

JURISDICTION CASE DIGEST TROPANG POTCHI

call for an examination of the probative value of the evidence presented, the truth or falsehood of facts being admitted.[67] A question of fact exists when the doubt or difference arises as to the truth or falsehood of facts or when the query invites calibration of the whole evidence considering mainly the credibility of the witnesses, the existence and relevancy of specific surrounding circumstances, as well as their relation to each other and to the whole, and the probability of the situation.[68]

Here, the Petition of the Republic raises pure questions of law, i.e., whether Civil Case No. 106 should have been dismissed for failure to implead indispensable parties and for forum shopping. Thus, the direct resort by the Republic to this Court is proper.

The Court shall now consider the propriety of the dismissal by the RTC-Branch 1 of the Complaint for Expropriation of the Republic.

The right of the Republic to be substituted for ISA as plaintiff in Civil Case No. 106 had long been affirmed by no less than this Court in the ISA case. The dispositive portion of the ISA case reads:

WHEREFORE, for all the foregoing, the Decision of the Court of Appeals dated 8 October 1991 to the extent that it affirmed the trial court’s order dismissing the expropriation proceedings, is hereby REVERSED and SET ASIDE and the case is REMANDED to the court a quo which shall allow the substitution of the Republic of the Philippines for petitioner Iron Steel Authority for further proceedings consistent with this Decision. No pronouncement as to costs.[69]

The ISA case had already become final and executory, and entry of judgment was made in said case on August 31, 1998. The RTC-Branch 1, in an Order dated November 16, 2001, effected the substitution of the Republic for ISA.

The failure of the Republic to actually file a motion for execution does not render the substitution void. A writ of execution requires the sheriff or other proper officer to whom it is directed to enforce the terms of the writ. [70] The November 16, 2001 Order of the RTC-Branch 1 should be deemed as voluntary compliance with a final and executory judgment of this Court, already rendering a motion for and issuance of a writ of execution superfluous.

Besides, no substantive right was violated by the voluntary compliance by the RTC-Branch 1 with the directive in the ISA case even without a motion for execution having been filed. To the contrary, the RTC-Branch 1 merely enforced the judicially determined right of the Republic to the substitution. While it is desirable that the Rules of Court be faithfully and even meticulously observed, courts should not be so strict about procedural lapses that do not really impair the administration of justice. If the rules are intended to insure the orderly conduct of litigation it is because of the higher objective they seek which is the protection of the substantive rights of the parties.[71]

The Court also observes that MCFC did not seek any remedy from the Order dated November 16, 2001 of the RTC-Branch 1. Consequently, the said Order already became final, which even the RTC-Branch 1 itself cannot reverse and set aside on the ground of “honest mistake.”

The RTC-Branch 1 dismissed the Complaint in Civil Case No. 106 on another ground: that MCFC is not a proper party to the expropriation proceedings, not being the owner of the parcels of land sought to be expropriated. The RTC-Branch 1 ratiocinated that since the exercise of the power of eminent domain involves the taking of private land intended for public use upon payment of just compensation to the owner, then a complaint for expropriation must be directed against the owner of the land sought to be expropriated.

The Republic insists, however, that MCFC is a real party-in-interest, impleaded as a defendant in the Complaint for Expropriation because of its possessory

or occupancy rights over the subject parcels of land, and not by reason of its ownership of the said properties. In addition, the Republic maintains that non-joinder of parties is not a ground for the dismissal of an action.

Rule 67, Section 1 of the then Rules of Court[72] described how expropriation proceedings should be instituted:

Section 1. The complaint. – The right of eminent domain shall be exercised by the filing of a complaint which shall state with certainty the right and purpose of condemnation, describe the real or personal property sought to be condemned, and join as defendants all persons owning or claiming to own, or occupying, any part thereof or interest therein, showing, so far as practicable, the interest of each defendant separately. If the title to any property sought to be condemned appears to be in the Republic of the Philippines, although occupied by private individuals, or if the title is otherwise obscure or doubtful so that the plaintiff cannot with accuracy or certainty specify who are the real owners, averment to that effect may be made in the complaint. [73] (Emphases supplied.)

For sure, defendants in an expropriation case are not limited to the owners of the property to be expropriated, and just compensation is not due to the property owner alone. As this Court held in De Knecht v. Court of Appeals[74]:

The defendants in an expropriation case are not limited to the owners of the property condemned. They include all other persons owning, occupying or claiming to own the property. When [property] is taken by eminent domain, the owner x x x is not necessarily the only person who is entitled to compensation. In the American jurisdiction, the term ‘owner’ when employed in statutes relating to eminent domain to designate the persons who are to be made parties to the proceeding, refer, as is the rule in respect of those entitled to compensation, to all those who have lawful interest in the property to be condemned, including a mortgagee, a lessee and a vendee in possession under an executory contract. Every person

29 | P a g e

Page 30: JURISDICTION-DIGESTED-OCHOA.docx

JURISDICTION CASE DIGEST TROPANG POTCHI

having an estate or interest at law or in equity in the land taken is entitled to share in the award. If a person claiming an interest in the land sought to be condemned is not made a party, he is given the right to intervene and lay claim to the compensation. (Emphasis supplied.)

At the time of the filing of the Complaint for Expropriation in 1983, possessory/occupancy rights of MCFC over the parcels of land sought to be expropriated were undisputed. In fact, Letter of Instructions No. 1277[75] dated November 16, 1982 expressly recognized that portions of the lands reserved by Presidential Proclamation No. 2239, also dated November 16, 1982, for the use and immediate occupation by the NSC, were then occupied by an idle fertilizer plant/factory and related facilities of MCFC. It was ordered in the same Letter of Instruction that:

(1) NSC shall negotiate with the owners of MCFC, for and on behalf of the Government, for the compensation of MCFC's present occupancy rights on the subject lands at an amount of Thirty (P30.00) Pesos per square meter or equivalent to the assessed value thereof (as determined by the City Assessor of Iligan), whichever is higher. NSC shall give MCFC the option to either remove its aforesaid plant, structures, equipment, machinery and other facilities from the lands or to sell or cede ownership thereof to NSC at a price equivalent to the fair market value thereof as appraised by the Asian Appraisal Inc. as may be mutually agreed upon by NSC and MCFC.

(2) In the event that NSC and MCFC fail to agree on the foregoing within sixty (60) days from the date hereof, the Iron and Steel Authority (ISA) shall exercise its authority under Presidential Decree (PD) No. 272, as amended, to initiate the expropriation of the aforementioned occupancy rights of MCFC on the subject lands as well as the plant, structures, equipment, machinery and related facilities, for and on behalf of NSC, and thereafter cede the same to NSC. During the pendency of the expropriation proceedings, NSC shall take possession of the properties, subject to bonding and other requirements of P.D. 1533. (Emphasis supplied.)

Being the occupant of the parcel of land sought to be expropriated, MCFC could very well be named a defendant in Civil Case No. 106. The RTC-Branch 1 evidently erred in dismissing the Complaint for Expropriation against MCFC for not being a proper party.

Also erroneous was the dismissal by the RTC-Branch 1 of the original Complaint for Expropriation for having been filed only against MCFC, the occupant of the subject land, but not the owner/s of the said property.

Dismissal is not the remedy for misjoinder or non-joinder of parties. According to Rule 3, Section 11 of the Rules of Court:

SEC. 11. Misjoinder and non-joinder of parties. – Neither misjoinder nor non-joinder of parties is ground for dismissal of an action. Parties may be dropped or added by order of the court on motion of any party or on its own initiative at any stage of the action and on such terms as are just. Any claim against a misjoined party may be severed and proceeded with separately. (Emphasis supplied.)

MCFC contends that the aforequoted rule does not apply in this case where the party not joined, i.e., the owner of the property to be expropriated, is an indispensable party.

An indispensable party is a party-in-interest without whom no final determination can be had of an action.[76]

Now, is the owner of the property an indispensable party in an action for expropriation? Not necessarily. Going back to Rule 67, Section 1 of the Rules of Court,

expropriation proceedings may be instituted even when “title to the property sought to be condemned appears to be in the Republic of the Philippines, although occupied by private individuals.” The same rule provides that a complaint for expropriation shall name as defendants “all persons owning or claiming to own, or occupying, any part thereof or interest” in the property sought to be condemned. Clearly, when the property already appears to belong to the Republic, there is no sense in the Republic instituting expropriation proceedings against itself. It can still, however, file a complaint for expropriation against the private persons occupying the property. In such an expropriation case, the owner of the property is not an indispensable party.

To recall, Presidential Proclamation No. 2239 explicitly states that the parcels of land reserved to NSC are part of the public domain, hence, owned by the Republic. Letter of Instructions No. 1277 recognized only the occupancy rights of MCFC and directed NSC to institute expropriation proceedings to determine the just compensation for said occupancy rights. Therefore, the owner of the property is not an indispensable party in the original Complaint for Expropriation in Civil Case No. 106.

Assuming for the sake of argument that the owner of the property is an indispensable party in the expropriation proceedings, the non-joinder of said party would still not warrant immediate dismissal of the complaint for expropriation. In Vda. De Manguerra v. Risos,[77] the Court applied Rule 3, Section 11 of the Rules of Court even in case of non-joinder of an indispensable party, viz:

[F]ailure to implead an indispensable party is not a ground for the dismissal of an action. In such a case, the remedy is to implead the non-party claimed to be indispensable. Parties may be added by order of the court, on motion of the party or on its own initiative at any stage of the action and/or such times as are just. If the petitioner/plaintiff refuses to implead an indispensable party despite the order of the court, the latter may dismiss the complaint/petition for the petitioner's/plaintiff's failure to comply. (Emphasis supplied.)

In this case, the RTC-Branch 1 did not first require the Republic to implead the alleged owner/s of the parcel of land sought to be expropriated. Despite the absence of any order from the Court, the Republic – upon becoming aware that the

30 | P a g e

Page 31: JURISDICTION-DIGESTED-OCHOA.docx

JURISDICTION CASE DIGEST TROPANG POTCHI

parcels of land involved in the 1914 Cacho case and 1997 Cacho case, claimed by Teofilo and LANDTRADE, and Vidal and AZIMUTH, encroached into and overlapped with the parcel of land subject of Civil Case No. 106 – sought leave of court to file a Supplemental Complaint to implead these four parties. The RTC-Branch 1 did not take the Supplemental Complaint of the Republic into consideration. Instead, it dismissed outright the original Complaint for Expropriation against MCFC.

Forum shopping

The RTC-Branch 1 further erred in finding that the Republic committed forum shopping by (1) simultaneously instituting the actions for expropriation (Civil Case No. 106) and reversion (Civil Case No. 6686) for the same parcels of land; and (2) taking inconsistent positions when it conceded lack of ownership over the parcels of land in the expropriation case but asserted ownership of the same properties in the reversion case.

There is no dispute that the Republic instituted reversion proceedings (Civil Case No. 6686) for the same parcels of land subject of the instant Expropriation Case (Civil Case No. 106). The Complaint for Cancellation of Titles and Reversion[78] dated September 27, 2004 was filed by the Republic with the RTC on October 13, 2004. The records, however, do not show when the Supplemental Complaint for Expropriation[79] dated September 28, 2004 was filed with the RTC. Apparently, the Supplemental Complaint for Expropriation was filed after the Complaint for Cancellation of Titles and Reversion since the Republic mentioned in the former the fact of filing of the latter.[80] Even then, the Verification and Certification of Non-Forum Shopping[81] attached to the Supplemental Complaint for Expropriation did not disclose the filing of the Complaint for Cancellation of Titles and Reversion. Notwithstanding such non-disclosure, the Court finds that the Republic did not commit forum shopping for filing both Complaints.

In NBI-Microsoft Corporation v Hwang,[82] the Court laid down the circumstances when forum shopping exists:

Forum-shopping takes place when a litigant files multiple suits involving the same parties, either simultaneously or successively, to secure a favorable

judgment. Thus, itexists where the elements of litis pendentia are present, namely: (a) identity of parties, or at least such parties who represent the same interests in both actions; (b) identity of rights asserted and relief prayed for, the relief being founded on the same facts; and (c) the identity with respect to the two preceding particulars in the two cases is such that any judgment that may be rendered in the pending case, regardless of which party is successful, would amount to res judicata in the other case. Forum-shopping is an act of malpractice because it abuses court processes. x x x.

Here, the elements of litis pendencia are wanting. There is no identity of rights asserted and reliefs prayed for in Civil Case No. 106 and Civil Case No. 6686.

Civil Case No. 106 was instituted against MCFC to acquire, for a public purpose, its possessory/occupancy rights over 322,532 square meters or 32.25 hectares of land which, at the time of the filing of the original Complaint in 1983, was not yet covered by any certificate of title. On the other hand, Civil Case No. 6686 sought the cancellation of OCT Nos. 0-1200 (a.f.) and 0-1201 (a.f.), which was entered into registration on December 4, 1998 in Doña Demetria’s name, on the argument that the parcels of land covered by said certificates exceeded the areas granted by the LRC to Doña Demetria in GLRO Record Nos. 6908 and 6909, as affirmed by this Court in the 1914 Cacho case.

Expropriation vis-à-vis reversion

The Republic is not engaging in contradictions when it instituted both expropriation and reversion proceedings for the same parcels of land. The expropriation and reversion proceedings are distinct remedies that are not necessarily exclusionary of each other.

The filing of a complaint for reversion does not preclude the institution of an action for expropriation. Even if the land is reverted back to the State, the same may still be subject to expropriation as against the occupants thereof.

Also, Rule 67, Section 1 of the Rules of Court allows the filing of a complaint for expropriation even when “the title to any property sought to be condemned appears to be in the Republic of the Philippines, although occupied by private individuals, or if the title is otherwise obscure or doubtful so that the plaintiff cannot with accuracy or certainty specify who are the real owners.” Rule 67, Section 9 of the Rules of Court further provides:

SEC. 9. Uncertain ownership; conflicting claims. – If the ownership of the property taken is uncertain, or there are conflicting claims to any part thereof, the court may order any sum or sums awarded as compensation for the property to be paid to the court for the benefit of the person adjudged in the same proceeding to be entitled thereto. But the judgment shall require the payment of the sum or sums awarded to either the defendant or the court before the plaintiff can enter upon the property, or retain it for the public use or purpose if entry has already been made. (Emphasis supplied.)

Hence, the filing by the Republic of the Supplemental Complaint for Expropriation impleading Teofilo, Vidal, LANDTRADE, and AZIMUTH, is not necessarily an admission that the parcels of land sought to be expropriated are privately owned. At most, the Republic merely acknowledged in its Supplemental Complaint that there are private persons also claiming ownership of the parcels of land. The Republic can still consistently assert, in both actions for expropriation and reversion, that the subject parcels of land are part of the public domain.

In sum, the RTC-Branch 1 erred in dismissing the original Complaint and disallowing the Supplemental Complaint in Civil Case No. 106. The Court reverses and sets aside the Resolutions dated July 12, 2005 and October 24, 2005 of the RTC-Branch 1 in Civil Case 106, and reinstates the Complaint for Reversion of the Republic.

Dolot v Paje, et al.

31 | P a g e

Page 32: JURISDICTION-DIGESTED-OCHOA.docx

JURISDICTION CASE DIGEST TROPANG POTCHI

Facts: Maricris Dolot filed a petition that sometime in 2009 they protested that an ore mining company in the municipality of Matnog violated a number of laws including not having a permit and Republic Act no. 7076. The petitioners find it their right to protect the environment. The case was referred by the Executive Judge to the RTC of Sorsogon, Branch 53 being the designated environmental court. In the Order dated September 16, 2011, the case was summarily dismissed for lack of jurisdiction.

Issue: whether or not the RTC-Branch 53 has jurisdiction to resolve Civil Case No. 2011-8338.

Held: Yes it has and RTC cannot solely rely on SC.A.O. No. 7 and Admin Circular No. 23-2008 whether it has jurisdiction over the case. None is more well-settled than the rule that jurisdiction, which is the power and authority of the court to hear, try and decide a case, is conferred by law.It may either be over the nature of the action, over the subject matter, over the person of the defendants or over the issues framed in the pleadings. By virtue of Batas Pambansa Blg. 129 or the Judiciary Reorganization Act of 1980, jurisdiction over special civil actions for certiorari, prohibition and mandamus is vested in the RTC. Particularly, Section 21 thereof provides that the RTCs shall exercise original jurisdiction –

in the issuance of writs of certiorari, prohibition, mandamus, quo warranto, habeas corpus and injunction which may be enforced in any part of their respective regions.

Ang v Spouse Ang

Facts: On September 2, 1992, spouses Alan and Em Ang (respondents)obtained a loan of US$300,000.00 from Theodore and Nancy Ang (petitioners). On evendate, the respondents executed a promissory note in favor of the petitionerswherein they promised to pay the latter the said amount, with interest at 10% per annum, upon demand. However, despiterepeated demands, the respondents failed to pay the petitioners.Thus, on August 28, 2006, the petitioners sent the respondents a demand letter asking them to pay their outstanding debt which, at that time, already amounted to US$719,671.23 inclusive of the 10%) annual interest that had accumulated overthe years. Notwithstanding the receipt of the said demand letter, therespondents still failed to settle their loan obligation.On August 6, 2006, the petitioners, who were then residing in LosAngeles, California, United States of America (USA), executed theirrespective Special Powers of Attorney in favor of Atty. Aceron for the purpose of filing an action in court againstthe respondents. On September 15, 2006, Atty. Aceron, in behalf of the petitioners, filed a Complaint for collection of sum of money with the RTCof Quezon City against the respondents.

Issue: Whether or not the CA committed reversible error of law when it ruled that the complaint must be dismissed on the ground that venue was not properly laid.

Held: The CA did not commit any reversible error when it annulledand set aside the orders of the RTC of Quezon City and consequentlydismissed the petitioners’ complaint against the respondents on the groundof improper venue.It is a legal truism that the rules on the venue of personal actions are fixed for the convenience of the plaintiffs and their witnesses. Equallysettled, however, is the principle that choosing the venue of an action is notleft to a plaintiff’s caprice; the matter is regulated by the Rules of Court. The petitioners’ complaint for collection of sum of money against therespondents is a personal action as it primarily seeks the enforcement of acontract. The Rules give the plaintiff the option of choosing where to filehis complaint. He can file it in the place where he himself or any of them resides, or where the defendant or any of the defendants resides or maybe found. The plaintiff or the defendant must be residents of the place wherethe action has been instituted at the time the action is commenced. However, if the plaintiff does not reside in the Philippines, thecomplaint in such case may only be filed in the court of the place where thedefendant resides.

Republic of the PhilippinesSUPREME COURT

Manila

FIRST DIVISION

G.R. No. 159746 July 18, 2012

SPOUSES RAMON MENDIOLA and ARACELI N. MENDIOLA, Petitioners, vs.THE HON. COURT OF APPEALS, PILIPINAS SHELL PETROLEUM CORPORATION, and TABANGAO REALTY, INC., Respondents.

D E C I S I O N

BERSAMIN, J.:

Through their petition for certiorari, mandamus and prohibition, petitioners assail the resolutions promulgated on November 22, 20021 and July 31, 2002,2 whereby the Court of Appeals (CA) respectively denied petitioners' motion to dismiss the appeal and motion for reconsideration. They allege that the CA thereby committed grave abuse of discretion amounting to lack or excess of jurisdiction.

Antecedents

On July 31, 1985, Pilipinas Shell Petroleum Corporation (Shell) entered into an agreement for the distribution of Shell petroleum products (such as fuels, lubricants and allied items) by Pacific Management & Development (Pacific), a single proprietorship belonging to petitioner Ramon G. Mendiola (Ramon). To secure Pacific’s performance of its obligations under the agreement, petitioners executed on August 1, 1985 a real

estate mortgage in favor of Shell3 covering their real estate and its improvements, located in the then Municipality of Parañaque, Rizal, and registered under Transfer Certificate of Title No. S-59807 of the Registry of Deeds of Rizal (in the name of "Ramon Mendiola, married to Araceli Mendoza").4

Pacific ultimately defaulted on its obligations, impelling Shell to commence extrajudicial foreclosure proceedings in April 1987. Having received a notice of the extrajudicial foreclosure scheduled to be held at the main entrance of the Parañaque Municipal Hall on May 14, 1987,5 petitioners proceeded to the announced venue on the scheduled date and time but did not witness any auction being conducted and did not meet the sheriff supposed to conduct the auction despite their being at the lobby from 9:00 am until 11:30 am of May 14, 1987.6 They later learned that the auction had been held as scheduled by Deputy Sheriff Bernardo San Juan of the Regional Trial Court (RTC) in Makati, and that their mortgaged realty had been sold to Tabangao Realty, Inc. (Tabangao), as the corresponding certificate of sale bears out.7 They further learned that Tabangao’s winning bidder bid of P670,000.00 had topped Shell’s bid of P660,000.00.8

After application of the proceeds of the sale to the obligation of Pacific, a deficiency of P170,228.00 (representing the foreclosure expenses equivalent of 25% of the amount claimed plus interest) remained. The deficiency was not paid by Ramon. Thus, on September 2, 1987, Shell sued in the RTC in Manila to recover the deficiency, docketed as Civil Case No. 87-41852 entitled Pilipinas Shell Petroleum Corporation v. Ramon G. Mendiola, doing business under the name and style Pacific Management & Development (Manila case).9

In his answer with counterclaim filed on October 28, 1987, Ramon asserted that the extra-judicial foreclosure of the mortgage had been devoid of basis in fact and in law; and that the foreclosure and the filing of the action were made in bad faith, with malice, fraudulently and in gross and wanton violation of his rights.

On March 22, 1988, petitioners commenced in the RTC in Makati an action to annul the extrajudicial foreclosure docketed as Civil Case No. 88-398 entitled Ramon G. Mendiola and Araceli N. Mendiola v. Pilipinas Shell Petroleum Corporation, Tabangao Realty, Inc., and Maximo C. Contreras, as Clerk of Court and Ex Oficio Sheriff of Rizal,10 which was assigned to Branch 134 (Makati case).

As defendants in the Makati case, Shell and Tabangao separately moved for dismissal,11 stating similar grounds, namely: (a) that the Makati RTC had no jurisdiction due to the pendency of the Manila case; (b) that the complaint stated no cause of action, the Makati case having been filed more than a year after the registration of the certificate of sale; (c) that another action (Manila case) involving the same subject matter was pending; (d) that the venue was improperly laid; and (e) that the Makati case was already barred by petitioners’ failure to raise its cause of action as a compulsory counterclaim in the Manila case.

After the Makati RTC denied both motions on September 23, 1988,12 Shell filed its answer ad cautelam,13whereby it denied petitioners’ allegation that no auction had been held; insisted that there had been proper accounting of the deliveries made to Pacific

32 | P a g e

Page 33: JURISDICTION-DIGESTED-OCHOA.docx

JURISDICTION CASE DIGEST TROPANG POTCHI

and its clients; and averred that petitioners’ failure to file their compulsory counterclaim in the Manila case already barred the action.

Pending the trial of the Makati case, the Manila RTC rendered its judgment in favor of Shell on May 31, 1990, viz:

WHEREFORE, IN VIEW OF THE FOREGOING, defendants (sic) is ordered to pay plaintiffs as follows:

1. On the First Cause of Action –

a) P167,585.50 representing the deficiency as of the date of the foreclosure sale;

b) P2,643.26 representing the interest due on the unpaid principal as of 30 June 1987; and

c) The sum corresponding to the interest due on the unpaid principal from 30 June 1987 to date.

2. On the Second Cause of Action – attorney’s fees and expenses of litigation to (sic) the amount of P15,000.00; and finally,

3. Costs of suit.

SO ORDERED.14

As sole defendant in the Manila case, Ramon appealed (C.A.-G.R. No. CV-28056), but his appeal was decided adversely to him on July 22, 1994,15 with the CA affirming the Manila RTC’s decision and finding that he was guilty of forum shopping for instituting the Makati case.

Undaunted, he next appealed to the Court (G.R. No. 122795), which denied his petition for review on February 26, 1996,16 and upheld the foreclosure of the mortgage. The decision of the Court became final and executory, as borne out by the entry of judgment issued on June 10, 1996.17

Nonetheless, on February 3, 1998, the Makati RTC resolved the Makati case,18 finding that there had been no auction actually conducted on the scheduled date; that had such auction taken place, petitioners could have actively participated and enabled to raise their objections against the amount of their supposed obligation; and that they had been consequently deprived of notice and hearing as to their liability. The Makati RTC disposed as follows:

WHEREFORE, premises considered, plaintiffs having duly established their case that the SHERIFF’s Certificate of Sale of May 14, 1987, is void for lack of actual auction sale and lack of valid consideration as the amount utilized by the SHERIFF was based on an

invalid amount as a basis of an Extra-Judicial Foreclosure of Mortgage where the amount of the mortgage is based on a future obligation unilaterally adjudicated by SHELL alone in violation of MENDIOLA’s right of due process, and judgment is hereby rendered as follows:

1. Declaring as NULL and VOID the Extra-Judicial Foreclosure of Mortgage of plaintiff’s house and lot under TCT No. T-59807 issued by the Register of Deeds of Rizal;

2. Declaring as NULL and VOID the Certificate of Sale issued by Maximo C. Contreras on May 14, 1987 in favor of TABANGAO REALTY, INC.;

3. Ordering defendant PILIPINAS SHELL PETROLEUM CORPORATION to make a full accounting of the extent of the future obligation of plaintiff MENDIOLA in the Mortgage Contract before any foreclosure proceedings are initiated;

4. Ordering defendants PILIPINAS SHELL PETROLEUM CORPORATION and TABANGAO REALTY INC. to pay the amount of P20,000.00 as and by way of attorney’s fees; and

5. To pay the costs.

SO ORDERED.

Shell sought the reconsideration of the decision,19 maintaining that the issues raised on the validity of the foreclosure sale and on the amount of the outstanding obligation of Pacific had been settled in the Manila case; and that the Makati RTC became bereft of jurisdiction to render judgment on the same issues pursuant to the principle of res judicata.

Tabangao adopted Shell’s motion for reconsideration.

On October 5, 1999, however, the Makati RTC denied Shell’s motion for reconsideration,20 to wit:

WHEREFORE, premises considered, there is NO RES JUDICATA to speak of in this case. Consequently, the "Motion for Reconsideration" filed by defendant Pilipinas Shell Petroleum Corporation, which was later adopted by defendant Tabangao Realty, Inc., is hereby DENIED. Plaintiff’s "Motion for Execution" is likewise DENIED for reasons as stated above.

SO ORDERED.21

Aggrieved by the decision of the Makati RTC, Shell and Tabangao filed a joint notice of appeal.22 The appeal was docketed in the CA as C.A.-G.R. No. 65764.

In their appellants’ brief filed in C.A.-G.R. No. 65764,23 Shell and Tabangao assigned the following errors, namely:

I

THE COURT A QUO COMMITTED GRAVE ERROR IN NOT DISMISSING THE CASE ON THE GROUND OF LITIS PENDENTIA AND, SUBSEQUENTLY, ON THE GROUND OF RES JUDICATA.

II

THE COURT A QUO COMMITTED MANIFEST ERROR IN DISREGARDING THAT THE LEGAL REQUIREMENTS FOR A VALID EXTRAJUDICIAL FORECLOSURE WERE SATISFIED.

III

THE COURT A QUO COMMITTED SERIOUS ERROR IN RENDERING THE ASSAILED DECISION AND ASSAILED RESOLUTION IN CONTRAVENTION OF THE RULINGS OF A CO-EQUAL COURT AND SUPERIOR COURTS.

Instead of filing their appellees’ brief, petitioners submitted a motion to dismiss appeal,24 mainly positing that Section 1, Rule 41 of the Rules of Court prohibited an appeal of the order denying a motion for reconsideration.

On November 22, 2002, the CA denied petitioners’ motion to dismiss appeal through the first assailed resolution, stating:25

For consideration is the Motion to Dismiss Appeal dated August 6, 2002 filed by counsel for plaintiffs-appellees praying for the dismissal of the appeal on the grounds that the Notice of Appeal filed by defendants-appellants was specifically interposed solely against the Resolution of the trial court dated October 20, 1999 which merely denied defendant-appellants’ Motion for Reconsideration of the trial court’s decision, dated February 3, 1998.

Upon perusal of the records of the case, it seems apparent that herein defendants-appellants intended to appeal not only the Resolution dated October 2, 1999 but also the Decision dated February 3, 1998. Assuming arguendo that defendants-appellants indeed committed a technical error, it is best that the parties be given every chance to fight their case fairly and in the open without resort to technicality to afford petitioners their day in court (Zenith Insurance vs. Purisima, 114 SCRA 62).

The Motion to Dismiss Appeal must not be granted if only to stress that the rules of procedure may not be misused as instruments for the denial of substantial justice. We must not forget the plain injunction of Section 2 of (now Sec. 6 of Rule 1, 1997 Revised Rules of Civil Procedure) Rule 1 that the "rules shall be liberally construed in order to promote their object and to assist the parties in obtaining not only speedy, but more imperatively just and inexpensive determination of justice in every action and proceeding" (Lim Tanhu vs. Ramolete 66 SCRA 425).

33 | P a g e

Page 34: JURISDICTION-DIGESTED-OCHOA.docx

JURISDICTION CASE DIGEST TROPANG POTCHI

WHEREFORE, in view of the foregoing, the Motion to Dismiss Appeal is hereby DENIED.

SO ORDERED.

On July 31, 2002, the CA denied petitioners’ motion for reconsideration through the second assailed resolution.26

Hence, petitioners brought these special civil actions for certiorari, mandamus and prohibition, insisting that the CA committed grave abuse of discretion amounting to lack or excess of jurisdiction in denying their motion to dismiss appeal and their motion for reconsideration.

Issue

Petitioners contend that the CA committed grave abuse of discretion in entertaining the appeal of Shell and Tabangao in contravention of Section 1, Rule 41 of the Rules of Court, which proscribes an appeal of the denial of a motion for reconsideration.

Shell and Tabangao counter that their appeal was not proscribed because the action could be said to be completely disposed of only upon the rendition on October 5, 1999 of the assailed resolution denying their motion for reconsideration; that, as such, the decision of February 3, 1998 and the denial of their motion for reconsideration formed one integrated disposition of the merits of the action; and that the CA justifiably applied the rules of procedure liberally.

Two issues have to be determined. The first is whether or not an appeal may be taken from the denial of a motion for reconsideration of the decision of February 3, 1998. The determination of this issue necessarily decides whether the petitions for certiorari, prohibition and mandamus were warranted. The second is whether the Makati case could prosper independently of the Manila case. The Court has to pass upon and resolve the second issue without waiting for the CA to decide the appeal on its merits in view of the urging by Shell and Tabangao that the Makati case was barred due to litispendentia or res judicata.

Ruling

The petition for certiorari, mandamus and prohibition lacks merit.

1.

Appeal by Shell and Tabangao of the denial of their motion for reconsideration was not proscribed

Petitioners’ contention that the appeal by Shell and Tabangao should be rejected on the ground that an appeal of the denial of their motion for reconsideration was prohibited cannot be sustained.

It is true that the original text of Section 1, Rule 41 of the 1997 Rules of Civil Procedure expressly limited an appeal to a judgment or final order, and proscribed the taking of an appeal from an order denying a motion for new trial or reconsideration, among others, viz:

Section 1.Subject of appeal. — An appeal may be taken from a judgment or final order that completely disposes of the case, or of a particular matter therein when declared by these Rules to be appealable.

No appeal may be taken from:

(a) An order denying a motion for new trial or reconsideration;

(b) An order denying a petition for relief or any similar motion seeking relief from judgment;

(c) An interlocutory order;

(d) An order disallowing or dismissing an appeal;

(e) An order denying a motion to set aside a judgment by consent, confession or compromise on the ground of fraud, mistake or duress, or any other ground vitiating consent;

(f) An order of execution;

(g) A judgment or final order for or against one or more of several parties or in separate claims, counterclaims, cross-claims and third-party complaints, while the main case is pending, unless the court allows an appeal therefrom; and

(h) An order dismissing an action without prejudice.

In all the above instances where the judgment or final order is not appealable, the aggrieved party may file an appropriate special civil action under Rule 65. (n)

The inclusion of the order denying a motion for new trial or a motion for reconsideration in the list of issuances of a trial court not subject to appeal was by reason of such order not being the final order terminating the proceedings in the trial court. This nature of the order is reflected in Section 9 of Rule 37 of the 1997 Rules of Civil Procedure, which declares that such order denying a motion for new trial or reconsideration is not appealable, "the remedy being an appeal from the judgment or final order."

In Heirs of Spouses Teofilo M. Reterta and Elisa Reterta v. Spouses Lorenzo Mores and Virginia Lopez,27 the Court further expounded:

The restriction against an appeal of a denial of a motion for reconsideration independently of a judgment or final order is logical and reasonable. A motion for reconsideration is not putting forward a new issue, or presenting new evidence, or changing the theory of the case, but is only seeking a reconsideration of the judgment or final order based on the same issues, contentions, and evidence either because: (a) the damages awarded are excessive; or (b) the evidence is insufficient to justify the decision or final order; or (c) the decision or final order is contrary to law.

By denying a motion for reconsideration, or by granting it only partially, therefore, a trial court finds no reason either to reverse or to modify its judgment or final order, and leaves the judgment or final order to stand. The remedy from the denial is to assail the denial in the course of an appeal of the judgment or final order itself.

In Quelnan v. VHF Philippines, Inc.,28 however, the Court has interpreted the proscription against appealing the order denying a motion for reconsideration to refer only to a motion for reconsideration filed against an interlocutory order, not to a motion for reconsideration filed against a judgment or final order, to wit:

This Court finds that the proscription against appealing from an order denying a motion for reconsideration refers to an interlocutory order, and not to a final order or judgment. That that was the intention of the above-quoted rules is gathered from Pagtakhan v. CIR, 39 SCRA 455 (1971), cited in above-quoted portion of the decision in Republic, in which this Court held that an order denying a motion to dismiss an action is interlocutory, hence, not appealable.

The rationale behind the rule proscribing the remedy of appeal from an interlocutory order is to prevent undue delay, useless appeals and undue inconvenience to the appealing party by having to assail orders as they are promulgated by the court, when they can be contested in a single appeal. The appropriate remedy is thus for the party to wait for the final judgment or order and assign such interlocutory order as an error of the court on appeal.

The denial of the motion for reconsideration of an order of dismissal of a complaint is not an interlocutory order, however, but a final order as it puts an end to the particular matter resolved, or settles definitely the matter therein disposed of, and nothing is left for the trial court to do other than to execute the order.

Not being an interlocutory order, an order denying a motion for reconsideration of an order of dismissal of a complaint is effectively an appeal of the order of dismissal itself.

The reference by petitioner, in his notice of appeal, to the March 12, 1999 Order denying his Omnibus Motion—Motion for Reconsideration should thus be deemed to refer to the January 17, 1999 Order which declared him non-suited and accordingly dismissed his complaint.

If the proscription against appealing an order denying a motion for reconsideration is applied to any order, then there would have been no need to specifically mention in both above-quoted sections of the Rules "final orders or judgments" as subject of

34 | P a g e

Page 35: JURISDICTION-DIGESTED-OCHOA.docx

JURISDICTION CASE DIGEST TROPANG POTCHI

appeal. In other words, from the entire provisions of Rule 39 and 41, there can be no mistaking that what is proscribed is to appeal from a denial of a motion for reconsideration of an interlocutory order.29

In Apuyan v. Haldeman,30 too, the Court categorized an order denying the motion for reconsideration as the final resolution of the issues a trial court earlier passed upon and decided, and accordingly held that the notice of appeal filed against the order of denial was deemed to refer to the decision subject of the motion for reconsideration.31

Subsequently, in Neypes v. Court of Appeals,32 where the decisive issue was whether or not the appeal was taken within the reglementary period, with petitioners contending that they had timely filed their notice of appeal based on their submission that the period of appeal should be reckoned from July 22, 1998, the day they had received the final order of the trial court denying their motion for reconsideration (of the order dismissing their complaint), instead of on March 3, 1998, the day they had received the February 12, 1998 order dismissing their complaint, the Court, citing Quelnan v. VHF Philippines, Inc. and Apuyan v. Haldeman, ruled that the receipt by petitioners of the denial of their motion for reconsideration filed against the dismissal of their complaint, which was a final order, started the reckoning point for the filing of their appeal, to wit:

Rule 41, Section 3 of the 1997 Rules of Civil Procedure states:

SEC. 3.Period of ordinary appeal. ― The appeal shall be taken wi thin fifteen (15) days from the notice of the judgment or final order appealed from. Where a record on appeal is required, the appellant shall file a notice of appeal and a record on appeal within thirty (30) days from the notice of judgment or final order.

The period to appeal shall be interrupted by a timely motion for new trial or reconsideration. No motion for extension of time to file a motion for new trial or reconsideration shall be allowed. (emphasis supplied)

Based on the foregoing, an appeal should be taken within 15 days from the notice of judgment or final order appealed from. A final judgment or order is one that finally disposes of a case, leaving nothing more for the court to do with respect to it. It is an adjudication on the merits which, considering the evidence presented at the trial, declares categorically what the rights and obligations of the parties are; or it may be an order or judgment that dismisses an action.

As already mentioned, petitioners argue that the order of July 1, 1998 denying their motion for reconsideration should be construed as the "final order," not the February 12, 1998 order which dismissed their complaint. Since they received their copy of the denial of their motion for reconsideration only on July 22, 1998, the 15-day reglementary period to appeal had not yet lapsed when they filed their notice of appeal on July 27, 1998.

What therefore should be deemed as the "final order," receipt of which triggers the start of the 15-day reglementary period to appeal – the February 12, 1998 order dismissing the complaint or the July 1, 1998 order dismissing the MR?

In the recent case of Quelnan v. VHF Philippines, Inc., the trial court declared petitioner Quelnan non-suited and accordingly dismissed his complaint. Upon receipt of the order of dismissal, he filed an omnibus motion to set it aside. When the omnibus motion was filed, 12 days of the 15-day period to appeal the order had lapsed. He later on received another order, this time dismissing his omnibus motion. He then filed his notice of appeal. But this was likewise dismissed ― for having been filed out of time.

The court a quo ruled that petitioner should have appealed within 15 days after the dismissal of his complaint since this was the final order that was appealable under the Rules. We reversed the trial court and declared that it was the denial of the motion for reconsideration of an order of dismissal of a complaint which constituted the final order as it was what ended the issues raised there.

This pronouncement was reiterated in the more recent case of Apuyan v. Haldeman et al. where we again considered the order denying petitioner Apuyan’s motion for reconsideration as the final order which finally disposed of the issues involved in the case.

Based on the aforementioned cases, we sustain petitioners’ view that the order dated July 1, 1998 denying their motion for reconsideration was the final order contemplated in the Rules.33

As the aftermath of these rulings, the Court issued its resolution in A.M. No. 07-7-12-SC to approve certain amendments to Rules 41, 45, 58 and 65 of the Rules of Court effective on December 27, 2007. Among the amendments was the delisting of an order denying a motion for new trial or motion for reconsideration from the enumeration found in Section 1, Rule 41 of the 1997 Rules of Civil Procedure of what are not appealable. The amended rule now reads:

Section 1. Subject of appeal.— An appeal may be taken from a judgment or final order that completely disposes of the case, or of a particular matter therein when declared by these Rules to be appealable.

No appeal may be taken from:

(a) An order denying a petition for relief or any similar motion seeking relief from judgment;

(b) An interlocutory order;

(c) An order disallowing or dismissing an appeal;

(d) An order denying a motion to set aside a judgment by consent, confession or compromise on the ground of fraud, mistake or duress, or any other ground vitiating consent;

(e) An order of execution;

(f) A judgment or final order for or against one or more of several parties or in separate claims, counterclaims, cross-claims and third-party complaints, while the main case is pending, unless the court allows an appeal therefrom; and

(g) An order dismissing an action without prejudice.

In any of the foregoing circumstances, the aggrieved party may file an appropriate special civil action as provided in Rule 65.

Based on the foregoing developments, Shell and Tabangao’s appeal, albeit seemingly directed only at the October 5, 1999 denial of their motion for reconsideration, was proper. Thus, we sustain the CA’s denial for being in accord with the rules and pertinent precedents. We further point out that for petitioners to insist that the appeal was limited only to the assailed resolution of October 5, 1999 was objectively erroneous, because Shell and Tabangao expressly indicated in their appellant’s brief that their appeal was directed at both the February 3, 1998 decision and the October 5, 1999 resolution.34

The petition cannot prosper if the CA acted in accordance with law and jurisprudence. Certiorari, prohibition and mandamus are extraordinary remedies intended to correct errors of jurisdiction and to check grave abuse of discretion. The term grave abuse of discretion connotes capricious and whimsical exercise of judgment as is equivalent to excess, or a lack of jurisdiction.35 The abuse must be so patent and gross as to amount to an evasion of a positive duty or a virtual refusal to perform a duty enjoined by law, or to act at all in contemplation of law as where the power is exercised in an arbitrary and despotic manner by reason of passion or hostility.36 Yet, here, petitioners utterly failed to establish that the CA abused its discretion, least of all gravely.

2. Makati case is barred and should be dismissed on ground of res judicata and waiver

The dismissal of the petition should ordinarily permit the CA to resume its proceedings in order to enable it to resolve the appeal of Shell and Tabangao. But the Court deems itself bound to first determine whether the Makati case could still proceed by virtue of their insistence that the cause of action for annulment of the foreclosure sale in the Makati case, which was intimately intertwined with the cause of action for collection of the deficiency amount in the Manila case, could not proceed independently of the Manila case.

Shell and Tabangao’s insistence has merit. The Makati case should have been earlier disallowed to proceed on the ground of litispendentia, or, once the decision in the Manila case became final, should have been dismissed on the ground of being barred by res judicata.

In the Manila case, Ramon averred a compulsory counterclaim asserting that the extrajudicial foreclosure of the mortgage had been devoid of basis in fact and in law; and that the foreclosure and the filing of the action had been made in bad faith, with malice, fraudulently and in gross and wanton violation of his rights. His pleading thereby showed that the cause of action he later pleaded in the Makati case - that of annulment

35 | P a g e

Page 36: JURISDICTION-DIGESTED-OCHOA.docx

JURISDICTION CASE DIGEST TROPANG POTCHI

of the foreclosure sale - was identical to the compulsory counterclaim he had set up in the Manila case.

Rule 6 of the 1997 Rules of Civil Procedure defines a compulsory counterclaim as follows:

Section 7.Compulsory counterclaim. — A compulsory counterclaim is one which, being cognizable by the regular courts of justice, arises out of or is connected with the transaction or occurrence constituting the subject matter of the opposing party’s claim and does not require for its adjudication the presence of third parties of whom the court cannot acquire jurisdiction. Such a counterclaim must be within the jurisdiction of the court both as to the amount and the nature thereof, except that in an original action before the Regional Trial Court, the counterclaim may be considered compulsory regardless of the amount. (n)

Accordingly, a counterclaim is compulsory if: (a) it arises out of or is necessarily connected with the transaction or occurrence which is the subject matter of the opposing party’s claim; (b) it does not require for its adjudication the presence of third parties of whom the court cannot acquire jurisdiction; and (c) the court has jurisdiction to entertain the claim both as to its amount and nature, except that in an original action before the RTC, the counterclaim may be considered compulsory regardless of the amount.

A compulsory counterclaim that a defending party has at the time he files his answer shall be contained therein.37Pursuant to Section 2, Rule 9 of the 1997 Rules of Civil Procedure, a compulsory counterclaim not set up shall be barred.

The four tests to determine whether a counterclaim is compulsory or not are the following, to wit: (a) Are the issues of fact or law raised by the claim and the counterclaim largely the same? (b) Would res judicata bar a subsequent suit on defendant’s claims, absent the compulsory counterclaim rule? (c) Will substantially the same evidence support or refute plaintiff’s claim as well as the defendant’s counterclaim? and (d) Is there any logical relation between the claim and the counterclaim, such that the conduct of separate trials of the respective claims of the parties would entail a substantial duplication of effort and time by the parties and the court?38 Of the four, the one compelling test of compulsoriness is the logical relation between the claim alleged in the complaint and that in the counterclaim. Such relationship exists when conducting separate trials of the respective claims of the parties would entail substantial duplication of time and effort by the parties and the court; when the multiple claims involve the same factual and legal issues; or when the claims are offshoots of the same basic controversy between the parties.39 If these tests result in affirmative answers, the counterclaim is compulsory.

The four tests are affirmatively met as far as the Makati case was concerned. The Makati case had the logical relation to the Manila case because both arose out of the extrajudicial foreclosure of the real estate mortgage constituted to secure the payment of petitioners’ credit purchases under the distributorship agreement with Shell. Specifically, the right of Shell to demand the deficiency was predicated on the validity of the extrajudicial foreclosure, such that there would not have been a deficiency to be

claimed in the Manila case had Shell not validly foreclosed the mortgage. As earlier shown, Ramon’s cause of action for annulment of the extrajudicial foreclosure was a true compulsory counterclaim in the Manila case. Thus, the Makati RTC could not have missed the logical relation between the two actions.

We hold, therefore, that the Makati case was already barred by res judicata. Hence, its immediate dismissal is warranted.

Bar by res judicata avails if the following elements are present, to wit: (a) the former judgment or order must be final; (b) the judgment or order must be on the merits; (c) it must have been rendered by a court having jurisdiction over the subject matter and the parties; (d) there must be, between the first and the second action, identity of parties, of subject matter and cause of action.40

The Manila RTC had jurisdiction to hear and decide on the merits Shell’s complaint to recover the deficiency, and its decision rendered on May 31, 1990 on the merits already became final and executory. Hence, the first, second and third elements were present.

Anent the fourth element, the Makati RTC concluded that the Manila case and the Makati case had no identity as to their causes of action, explaining that the former was a personal action involving the collection of a sum of money, but the latter was a real action affecting the validity of the foreclosure sale, stating in its order of October 5, 1999 denying Shell’s motion for reconsideration as follows:

Finally, as to whether there is identity of causes of action between the two (2) cases, this Court finds in negative.

xxxx

True, the test of identity of causes of action lies not in the form of an action but on whether the same evidence would support and establish the former and the present causes of action. The difference of actions in the aforesaid cases is of no moment. It has been held that a party cannot by varying the form of action or adopting a different method of presenting his case, escape the operation of the principle that one and the same cause of action shall not be twice litigated between the same parties and their privies. (Sangalang vs. Caparas, 151 SCRA 53; Gutierrez vs. Court of Appeals, 193 SCRA 437. This ruling however does not fall squarely on the present controversy.

Civil Case No. 42852 is for collection of sum of money, a personal action where what is at issue is whether spouses Mendiola have indebtedness to Pilipinas Shell. There is no concrete findings on questions regarding the validity of sale affecting the mortgaged property, otherwise, there would be a determination of transferring of title over the property which is already a real action. In the latter action, Manila courts has no jurisdiction considering that the property is located in Paranaque, then sitting under Makati RTC. At any rate, this Court is not unmindful of series of cases which state that from an otherwise rigid rule outlining jurisdiction of courts being limited in character, deviations have been sanctioned where the (1) parties agreed or have acquiesced in submitting the issues for determination by the court; (2) the parties were accorded full

opportunity in presenting their respective arguments of the issues litigated and of the evidence in support thereof; and (3) the court has already considered the evidence on record and is convinced that the same is sufficient and adequate for rendering a decision upon the issues controverted. xxx. While there is a semblance of substantial compliance with the aforesaid criteria, primarily because the issue of validity of foreclosure proceedings was submitted for determination of RTC Manila when this was stated as an affirmative defense by spouses Mendiola in their Answer to the complaint in Civil Case No. 42852, however it appears from the Decision rendered in said case that the issue on validity of foreclosure sale was not fully ventilated before the RTC Manila because spouses Mendiola’s right to present evidence in its behalf was declared waived. Naturally, where this issue was not fully litigated upon, no resolution or declaration could be made therein.

On the other hand, Civil Case No. 88-398 is an action for declaration of nullity or annulment of foreclosure sale, a real action where the location of property controls the venue where it should properly be filed. This Court undoubtedly has jurisdiction to adjudicate this case. Plaintiff spouses Mendiola merely claimed that no actual foreclosure sale was conducted, and if there was, the same was premature for lack of notice and hearing. Take note that plaintiffs do not deny their indebtedness to Pilipinas Shell although the amount being claimed is disputed. They are simply asserting their rights as owners of the mortgaged property, contending that they were not afforded due process in the course of foreclosure proceedings. And based mainly on the testimonial and documentary evidence presented, as well as the postulations, expositions and arguments raised by all parties in this case, it is the Court’s considered view that spouses Mendiola have established the material allegations in their complaint and have convincingly shown to the satisfaction of the Court that they are entitled to the reliefs prayed for. With these findings and adjudications, the Court does not find inconsistency with those held in Civil Case No. 42852. As to whether spouses Mendiola is still indebted to Pilipinas Shell is not in issue here, and not even a single discussion touched that matter as this would tantamount to encroaching upon the subject matter litigated in Civil Case No. 42852.41

The foregoing conclusion of the Makati RTC on lack of identity between the causes of action was patently unsound. The identity of causes of action does not mean absolute identity; otherwise, a party may easily escape the operation of res judicata by changing the form of the action or the relief sought. The test to determine whether the causes of action are identical is to ascertain whether the same evidence will sustain the actions, or whether there is an identity in the facts essential to the maintenance of the actions. If the same facts or evidence will sustain the actions, then they are considered identical, and a judgment in the first case is a bar to the subsequent action.42 Petitioners’ Makati case and Shell’s Manila case undeniably required the production of the same evidence. In fact, Shell’s counsel faced a dilemma upon being required by the Makati RTC to present the original copies of certain documents because the documents had been made part of the records of the Manila case elevated to the CA in connection with the appeal of the Manila RTC’s judgment.43 Also, both cases arose from the same transaction (i.e., the foreclosure of the mortgage), such that the success of Ramon in invalidating the extrajudicial foreclosure would have necessarily negated Shell’s right to recover the deficiency.

Apparently, the Makati RTC had the erroneous impression that the Manila RTC did not have jurisdiction over the complaint of petitioners because the property involved was

36 | P a g e

Page 37: JURISDICTION-DIGESTED-OCHOA.docx

JURISDICTION CASE DIGEST TROPANG POTCHI

situated within the jurisdiction of the Makati RTC. Thereby, the Makati RTC confused venue of a real action with jurisdiction. Its confusion was puzzling, considering that it was well aware of the distinction between venue and jurisdiction, and certainly knew that venue in civil actions was not jurisdictional and might even be waived by the parties.44 To be clear, venue related only to the place of trial or the geographical location in which an action or proceeding should be brought and does not equate to the jurisdiction of the court. It is intended to accord convenience to the parties, as it relates to the place of trial, and does not restrict their access to the courts.45 In contrast, jurisdiction refers to the power to hear and determine a cause,46 and is conferred by law and not by the parties.47

By virtue of the concurrence of the elements of res judicata, the immediate dismissal of the Makati case would have been authorized under Section 1, Rule 9 of the 1997 Rules of Civil Procedure, which provides:

Section 1. Defenses and objections not pleaded. — Defenses and objections not pleaded either in a motion to dismiss or in the answer are deemed waived. However, when it appears from the pleadings or the evidence on record that the court has no jurisdiction over the subject matter, that there is another action pending between the same parties for the same cause, or that the action is barred by a prior judgment or by statute of limitations, the court shall dismiss the claim. (2a)

The rule expressly mandated the Makati RTC to dismiss the case motuproprio once the pleadings or the evidence on record indicated the pendency of the Manila case, or, later on, disclosed that the judgment in the Manila case had meanwhile become final and executory.

Yet, we are appalled by the Makati RTC's flagrant disregard of the mandate.1âwphi1 Its reason for the disregard was not well-founded. We stress that its disregard cannot be easily ignored because it needlessly contributed to the clogging of the dockets of the Judiciary. Thus, we deem it to be imperative to again remind all judges to consciously heed any clear mandate under the Rules of Court designed to expedite the disposition of cases as well as to declog the court dockets.

WHEREFORE, we DISMISS the petition for certiorari, prohibition and mandamus for lack of merit; CONSIDER Civil Case No. 88-398 dismissed with prejudice on the. ground of res judicata; and ORDER petitioners to pay the costs of suit to respondents.

The Office of the Court Administrator is DIRECTED to disseminate this decision to all trial courts for their guidance. SO ORDERED.

Republic of the PhilippinesSupreme Court

Baguio City

EN BANC

THE CIVIL SERVICE G.R. No. 168766COMMISSION, Petitioner, - versus - HENRY A. SOJOR, Respondent.

x - - - - - - - - - - - - - - - - - - - - - - - - - - - - - - - - xD E C I S I O N

REYES, R.T., J.:

IS the president of a state university outside the reach of the disciplinary jurisdiction constitutionally granted to the Civil Service Commission (CSC) over all civil servants and officials?

Does the assumption by the CSC of jurisdiction over a president of a state university violate academic freedom?The twin questions, among others, are posed in this petition for review on certiorari of the Decision[1] of the Court of Appeals (CA) which annulled two (2) CSCResolutions[2] against respondent Henry A. Sojor.

The FactsThe uncontroverted facts that led to the controversy, as found by the CSC and the CA, are as follows:On August 1, 1991, respondent Sojor was appointed by then President Corazon Aquino as president of the Central Visayas Polytechnic College (CVPC) in DumagueteCity. In June 1997, Republic Act (R.A.) No. 8292, or the “Higher Education Modernization Act of 1997,” was enacted. This law mandated that a Board of Trustees (BOT) be formed to act as the governing body in state colleges. The BOT of CVPC appointed respondent as president, with a four-year term beginning September 1998 up to September 2002.[3] Upon the expiration of his first term of office in 2002, he was appointed president of the institution for a second four-year term, expiring on September 24, 2006.[4]On June 25, 2004, CVPC was converted into the Negros Oriental State University (NORSU).[5] A Board of Regents (BOR) succeeded the BOT as its governing body.Meanwhile, three (3) separate administrative cases against respondent were filed by CVPC faculty members before the CSC Regional Office (CSC-RO) No. VII in CebuCity, to wit:

1. ADMC DC No. 02-20(A) – Complaint for dishonesty, grave misconduct and conduct prejudicial to the best interest of the service filed on June 26, 2002 by Jose Rene A. Cepe and Narciso P. Ragay. It was alleged that respondent approved the release of salary differentials despite the absence of the required Plantilla and Salary Adjustment Form and valid appointments.[6]

2.2. ADM DC No. 02-20 – Complaint for dishonesty, misconduct and falsification of official documents filed on July 10, 2002 by Jocelyn Juanon and Carolina Fe Santos. The complaint averred that respondent maliciously allowed the antedating and falsification of the reclassification differential payroll, to the prejudice of instructors and professors who have pending request for adjustment of their academic ranks.[7]3. ADM DC No. 02-21 – Complaint for nepotism filed on August 15, 2002 by Rose Marie Palomar, a former part-time instructor of CVPC. It was alleged that respondent appointed his half-sister, EstrellasSojor-Managuilas, as casual clerk, in violation of the provisions against nepotism under the Administrative Code.[8]

Before filing his counter-affidavits, respondent moved to dismiss the first two complaints on grounds of lack of jurisdiction, bar by prior judgment and forum shopping.He claimed that the CSC had no jurisdiction over him as a presidential appointee. Being part of the non-competitive or unclassified service of the government, he was exclusively under the disciplinary jurisdiction of the Office of the President (OP). He argued that CSC had no authority to entertain, investigate and resolve charges against him; that the Civil Service Law contained no provisions on the investigation, discipline, and removal of presidential appointees. He also pointed out that the subject matter of the complaints had already been resolved by the Office of the Ombudsman.[9]Finding no sufficient basis to sustain respondent’s arguments, the CSC-RO denied his motion to dismiss in its Resolution dated September 4, 2002.[10] His motion for reconsideration[11] was likewise denied. Thus, respondent was formally charged with three administrative cases, namely: (1) Dishonesty, Misconduct, and Falsification of Official Document; (2) Dishonesty, Grave Misconduct, and Conduct Prejudicial to the Best Interest of the Service; and (3) Nepotism.[12]Respondent appealed the actions of the regional office to the Commission proper (CSC), raising the same arguments in his motion to dismiss.[13] He argued that since the BOT is headed by the Committee on Higher Education Chairperson who was under the OP, the BOT was also under the OP. Since the president of CVPC was appointed by the BOT, then he was a presidential appointee. On the matter of the jurisdiction granted to CSC by virtue of Presidential Decree (P.D.) No. 807[14] enacted in October 1975, respondent contended that this was superseded by the provisions of R.A. No. 8292,[15] a later law which granted to the BOT the power to remove university officials.CSC DispositionIn a Resolution dated March 30, 2004,[16] the CSC dismissed respondent’s appeal and authorized its regional office to proceed with the investigation. He was also preventively suspended for 90 days. The fallo of the said resolution states:

WHEREFORE, the appeal of Henry A. Sojor, President of Central Visayas Polytechnic College, is hereby DISMISSED. The Civil Service Commission Regional Office No. VII,

37 | P a g e

Page 38: JURISDICTION-DIGESTED-OCHOA.docx

JURISDICTION CASE DIGEST TROPANG POTCHI

CebuCity, is authorized to proceed with the formal investigation of the cases against Sojor and submit the investigation reports to the Commission within one hundred five (105) days from receipt hereof. Finally, Sojor is preventively suspended for ninety (90) days.[17]In decreeing that it had jurisdiction over the disciplinary case against respondent, the CSC opined that his claim that he was a presidential appointee had no basis in fact or in law. CSC maintained that it had concurrent jurisdiction with the BOT of the CVPC. We quote:His appointment dated September 23, 2002 was signed by then Commission on Higher Education (CHED) Chairman Ester A. Garcia. Moreover, the said appointment expressly stated that it was approved and adopted by the Central Visayas Polytechnic College Board of Trustees on August 13, 2002 in accordance with Section 6 of Republic Act No. 8292 (Higher education Modernization Act of 1997), which explicitly provides that, “He (the president of a state college) shall be appointed by the Board of Regents/Trustees, upon recommendation of a duly constituted search committee.” Since the President of a state college is appointed by the Board of Regents/Trustees of the college concerned, it is crystal clear that he is not a presidential appointee. Therefore, it is without doubt that Sojor, being the President of a state college (Central Visayas Polytechnic College), is within the disciplinary jurisdiction of the Commission.

The allegation of appellant Sojor that the Commission is bereft of disciplinary jurisdiction over him since the same is exclusively lodged in the CVPC Board of Trustees, being the appointing authority, cannot be considered. The Commission and the CVPC Board of Trustees have concurrent jurisdiction over cases against officials and employees of the said agency. Since the three (3) complaints against Sojor were filed with the Commission and not with the CVPC, then the former already acquired disciplinary jurisdiction over the appellant to the exclusion of the latter agency.[18] (Emphasis supplied)The CSC categorized respondent as a third level official, as defined under its rules, who are under the jurisdiction of the Commission proper. Nevertheless, it adopted the formal charges issued by its regional office and ordered it to proceed with the investigation:Pursuant to the Uniform Rules on Administrative Cases in the Civil Service, Sojor, being a third level official, is within the disciplinary jurisdiction of the Commission Proper. Thus, strictly speaking, the Commission has the sole jurisdiction to issue the formal charge against Sojor. x xx However, since the CSC RO No. VII already issued the formal charges against him and found merit in the said formal charges, the same is adopted. The CSC RO No. VII is authorized to proceed with the formal investigation of the case against Sojor in accordance with the procedure outlined in the aforestated Uniform Rules.[19] (Emphasis supplied) No merit was found by the CSC in respondent’s motion for reconsideration and, accordingly, denied it with finality on July 6, 2004.[20]

Respondent appealed the CSC resolutions to the CA via a petition for certiorari and prohibition. He alleged that the CSC acted without or in excess of its jurisdiction, or with grave abuse of discretion amounting to lack or excess of jurisdiction when it issued the assailed resolutions; that CSC encroached upon the academic freedom of CVPC; and that the power to remove, suspend, and discipline the president of CVPC was exclusively lodged in the BOT of CVPC.

CA DispositionOn September 29, 2004, the CA issued a writ of preliminary injunction directing the CSC to cease and desist from enforcing its Resolution dated March 30, 2004 and Resolution dated July 6, 2004.[21] Thus, the formal investigation of the administrative charges against Sojor before the CSC-RO was suspended.On June 27, 2005, after giving both parties an opportunity to air their sides, the CA resolved in favor of respondent. It annulled the questioned CSC resolutions and permanently enjoined the CSC from proceeding with the administrative investigation. The dispositive part of the CA decision reads:

WHEREFORE, in view of all the foregoing, and finding that the respondent Civil Service Commission acted without jurisdiction in issuing the assailed Resolution Nos. 040321 and 040766 dated March 20, 2004 and July 6, 2004, respectively, the same are hereby ANNULLED and SET ASIDE. The preliminary injunction issued by this Court on September 29, 2004 is hereby made permanent.SO ORDERED.[22]The CA ruled that the power to appoint carries with it the power to remove or to discipline. It declared that the enactment of R.A. No. 9299[23] in 2004, which converted CVPC into NORSU, did not divest the BOT of the power to discipline and remove its faculty members, administrative officials, and employees. Respondent was appointed as president of CVPC by the BOT by virtue of the authority granted to it under Section 6 of R.A. No. 8292.[24] The power of the BOT to remove and discipline erring employees, faculty members, and administrative officials as expressly provided for under Section 4 of R.A. No. 8292 is also granted to the BOR of NORSU under Section 7 of R.A. No. 9299. The said provision reads:

Power and Duties of Governing Boards. – The governing board shall have the following specific powers and duties in addition to its general powers of administration and exercise of all the powers granted to the board of directors of a corporation under Section 36 of Batas PambansaBlg. 68, otherwise known as the Corporation Code of the Philippines:

x xxx

to fix and adjust salaries of faculty members and administrative officials and employees x xx; and to remove them for cause in accordance with the requirements of due process of law. (Emphasis added)The CA added that Executive Order (E.O.) No. 292,[25] which grants disciplinary jurisdiction to the CSC over all branches, subdivisions, instrumentalities, and agencies of the government, including government-owned or controlled corporations with original charters, is a general law. According to the appellate court, E.O. No. 292 does not prevail over R.A. No. 9299,[26] a special law.IssuesPetitioner CSC comes to Us, seeking to reverse the decision of the CA on the ground that THE COURT OF APPEALS GRAVELY ERRED IN HOLDING THAT PETITIONER ACTED WITHOUT JURISDICTION IN ISSUING RESOLUTION NO. 040321 DATED MARCH 30, 2004 AND RESOLUTION NO. 04766 DATED JULY 6, 2004.[27]Our RulingThe petition is meritorious.I. Jurisdiction of the CSCThe Constitution grants to the CSC administration over the entire civil service.[28] As defined, the civil service embraces every branch, agency, subdivision, and instrumentality of the government, including every government-owned or controlled corporation.[29] It is further classified into career and non-career service positions. Career service positions are those where: (1) entrance is based on merit and fitness or highly technical qualifications; (2) there is opportunity for advancement to higher career positions; and (3) there is security of tenure. These include:

(1) Open Career positions for appointment to which prior qualification in an appropriate examination is required;

(2)(3) Closed Career positions which are scientific, or highly technical in nature;

these include the faculty and academic staff of state colleges and universities, and scientific and technical positions in scientific or research institutions which shall establish and maintain their own merit systems;

(4) Positions in the Career Executive Service; namely, Undersecretary, Assistant Secretary, Bureau Director, Assistant Bureau Director, Regional Director, Assistant Regional Director, Chief of Department Service and other officers of equivalent rank as may be identified by the Career Executive Service Board, all of whom are appointed by the President;

(5) Career officers, other than those in the Career Executive Service, who are appointed by the President, such as the Foreign Service Officers in the Department of Foreign Affairs;

(6) Commissioned officers and enlisted men of the Armed Forces which shall maintain a separate merit system;

38 | P a g e

Page 39: JURISDICTION-DIGESTED-OCHOA.docx

JURISDICTION CASE DIGEST TROPANG POTCHI

(7) Personnel of government-owned or controlled corporations, whether performing governmental or proprietary functions, who do not fall under the non-career service; and

(8) Permanent laborers, whether skilled, semi-skilled, or unskilled.[30]

Career positions are further grouped into three levels. Entrance to the first two levels is determined through competitive examinations, while entrance to the third level is prescribed by the Career Executive Service Board.[31] The positions covered by each level are:(a) The first level shall include clerical, trades, crafts, and custodial service positions which involve non-professional or subprofessional work in a non-supervisory or supervisory capacity requiring less than four years of collegiate studies;(b) The second level shall include professional, technical, and scientific positions which involve professional, technical, or scientific work in a non-supervisory or supervisory capacity requiring at least four years of college work up to Division Chief level; and(c) The third level shall cover positions in the Career Executive Service.[32]On the other hand, non-career service positions are characterized by: (1) entrance not by the usual tests of merit and fitness; and (2) tenure which is limited to a period specified by law, coterminous with the appointing authority or subject to his pleasure, or limited to the duration of a particular project for which purpose employment was made.[33] The law states:The Non-Career Service shall include:(1) Elective officials and their personal or confidential staff;(2) Secretaries and other officials of Cabinet rank who hold their positions at the pleasure of the President and their personal or confidential staff(s);(3) Chairman and members of commissions and boards with fixed terms of office and their personal or confidential staff;(4) Contractual personnel or those whose employment in the government is in accordance with a special contract to undertake a specific work or job, requiring special or technical skills not available in the employing agency, to be accomplished within a specific period, which in no case shall exceed one year, and performs or accomplishes the specific work or job, under his own responsibility with a minimum of direction and supervision from the hiring agency; and(5) Emergency and seasonal personnel.[34]It is evident that CSC has been granted by the Constitution and the Administrative Code jurisdiction over all civil service positions in the government service, whether career or non-career. From this grant of general jurisdiction, the CSC promulgated the Revised Uniform Rules on Administrative Cases in the Civil Service.[35] We find that the specific jurisdiction, as spelled out in the CSC rules, did not depart from the general jurisdiction granted to it by law. The jurisdiction of the Regional Office of the CSC and the Commission central office (Commission Proper) is specified in the CSC rules as:

Section 4.Jurisdiction of the Civil Service Commission. – The Civil Service Commission shall hear and decide administrative cases instituted by, or brought before it, directly or on appeal, including contested appointments, and shall review decisions and actions of its offices and of the agencies attached to it.Except as otherwise provided by the Constitution or by law, the Civil Service Commission shall have the final authority to pass upon the removal, separation and suspension of all officers and employees in the civil service and upon all matters relating to the conduct, discipline and efficiency of such officers and employees.Section 5.Jurisdiction of the Civil Service Commission Proper. – The Civil Service Commission Proper shall have jurisdiction over the following cases:A. Disciplinary1. Decisions of Civil Service Regional Offices brought before it on petition for review;2. Decisions of heads of departments, agencies, provinces, cities, municipalities and other instrumentalities, imposing penalties exceeding thirty days suspension or fine in an amount exceeding thirty days salary brought before it on appeal;3. Complaints brought against Civil Service Commission Proper personnel;4. Complaints against third level officials who are not presidential appointees;5. Complaints against Civil Service officials and employees which are not acted upon by the agencies and such other complaints requiring direct or immediate action, in the interest of justice;6. Requests for transfer of venue of hearing on cases being heard by Civil Service Regional Offices;7. Appeals from the Order of Preventive Suspension; and8. Such other actions or requests involving issues arising out of or in connection with the foregoing enumerations.B. Non-Disciplinary1. Decisions of Civil Service Commission Regional Offices brought before it;2. Requests for favorable recommendation on petition for executive clemency;3. Protests against the appointment, or other personnel actions, involving third level officials; and4. Such other analogous actions or petitions arising out of or in relation with the foregoing enumerations.Section 6.Jurisdiction of Civil Service Regional Offices. – The Civil Service Commission Regional Offices shall have jurisdiction over the following cases:A. Disciplinary1. Complaints initiated by, or brought before, the Civil Service Commission Regional Offices provided that the alleged acts or omissions were committed within the jurisdiction of the Regional Office, including Civil Service examination anomalies or irregularities and the persons complained of are employees of agencies, local or national, within said geographical areas;2. Complaints involving Civil Service Commission Regional Office personnel who are appointees of said office; and

3. Petitions to place respondent under Preventive Suspension.B. Non-Disciplinary1. Disapproval of appointments brought before it on appeal;2. Protests against the appointments of first and second level employees brought before it directly or on appeal. (Emphasis supplied)Respondent, a state university president with a fixed term of office appointed by the governing board of trustees of the university, is a non-career civil service officer. He was appointed by the chairman and members of the governing board of CVPC. By clear provision of law, respondent is a non-career civil servant who is under the jurisdiction of the CSC.II. The power of the BOR to discipline officials and employees is not exclusive. CSC has concurrent jurisdiction over a president of a state university.Section 4 of R.A. No. 8292, or the Higher Education Modernization Act of 1997, under which law respondent was appointed during the time material to the present case, provides that the school’s governing board shall have the general powers of administration granted to a corporation. In addition, Section 4 of the law grants to the board the power to remove school faculty members, administrative officials, and employees for cause:Section 4.Powers and Duties of Governing Boards. – The governing board shall have the following specific powers and duties in addition to its general powers of administration and the exercise of all the powers granted to the board of directors of a corporation under Section 36 of Batas PambansaBlg. 68, otherwise known as the Corporation Code of the Philippines:

x xxxh) to fix and adjust salaries of faculty members and administrative officials and employees subject to the provisions of the revised compensation and classification system and other pertinent budget and compensation laws governing hours of service, and such other duties and conditions as it may deem proper; to grant them, at its discretion, leaves of absence under such regulations as it may promulgate, any provisions of existing law to the contrary not withstanding; and to remove them for cause in accordance with the requirements of due process of law. (Emphasis supplied)The above section was subsequently reproduced as Section 7(i) of the succeeding law that converted CVPC into NORSU, R.A. No. 9299. Notably, and in contrast with the earlier law, R.A. No. 9299 now provides that the administration of the university and exercise of corporate powers of the board of the school shall be exclusive:Sec. 4.Administration. – The University shall have the general powers of a corporation set forth in Batas PambansaBlg. 68, as amended, otherwise known as “The Corporation Code of the Philippines.” The administration of the University and the exercise of its corporate powers shall be vested exclusively in the Board of Regents and the president of the University insofar as authorized by the Board.

39 | P a g e

Page 40: JURISDICTION-DIGESTED-OCHOA.docx

JURISDICTION CASE DIGEST TROPANG POTCHI

Measured by the foregoing yardstick, there is no question that administrative power over the school exclusively belongs to its BOR. But does this exclusive administrative power extend to the power to remove its erring employees and officials?In light of the other provisions of R.A. No. 9299, respondent’s argument that the BOR has exclusive power to remove its university officials must fail. Section 7 of R.A. No. 9299 states that the power to remove faculty members, employees, and officials of the university is granted to the BOR “in addition to its general powers of administration.” This provision is essentially a reproduction of Section 4 of its predecessor, R.A. No. 8292, demonstrating that the intent of the lawmakers did not change even with the enactment of the new law. For clarity, the text of the said section is reproduced below:Sec. 7.Powers and Duties of the Board of Regents. – The Board shall have the following specific powers and duties in addition to its general powers of administration and the exercise of all the powers granted to the Board of Directors of a corporation under existing laws:

x xxx

i. To fix and adjust salaries of faculty members and administrative officials and employees, subject to the provisions of the Revised Compensation and Position Classification System and other pertinent budget and compensation laws governing hours of service and such other duties and conditions as it may deem proper; to grant them, at its discretion, leaves of absence under such regulations as it may promulgate, any provision of existing law to the contrary notwithstanding; and to remove them for cause in accordance with the requirements of due process of law.[36] (Emphasis supplied)

ii.Verily, the BOR of NORSU has the sole power of administration over the university. But this power is not exclusive in the matter of disciplining and removing its employees and officials.Although the BOR of NORSU is given the specific power under R.A. No. 9299 to discipline its employees and officials, there is no showing that such power is exclusive. When the law bestows upon a government body the jurisdiction to hear and decide cases involving specific matters, it is to be presumed that such jurisdiction is exclusive unless it be proved that another body is likewise vested with the same jurisdiction, in which case, both bodies have concurrent jurisdiction over the matter.[37]All members of the civil service are under the jurisdiction of the CSC, unless otherwise provided by law. Being a non-career civil servant does not remove respondent from the ambit of the CSC. Career or non-career, a civil service official or employee is within the jurisdiction of the CSC.

This is not a case of first impression.

In University of the Philippines v. Regino,[38] this Court struck down the claim of exclusive jurisdiction of the UP BOR to discipline its employees. The Court held then:The Civil Service Law (PD 807) expressly vests in the Commission appellate jurisdiction in administrative disciplinary cases involving members of the Civil Service. Section 9(j) mandates that the Commission shall have the power to “hear and decide administrative disciplinary cases instituted directly with it in accordance with Section 37 or brought to it on appeal.” And Section 37(a) provides that, “The Commission shall decide upon appeal all administrative disciplinary cases involving the imposition of a penalty of suspension for more than thirty (30) days, or fine in an amount exceeding thirty days’ salary, demotion in rank or salary or transfer, removal or dismissal from office.” (Emphasis supplied)Under the 1972 Constitution, all government-owned or controlled corporations, regardless of the manner of their creation, were considered part of the Civil Service. Under the 1987 Constitution, only government-owned or controlled corporations with original charters fall within the scope of the Civil Service pursuant to Article IX-B, Section 2(1), which states:“The Civil Service embraces all branches, subdivisions, instrumentalities, and agencies of the government, including government-owned or controlled corporations with original charters.”

As a mere government-owned or controlled corporation, UP was clearly a part of the Civil Service under the 1973 Constitution and now continues to be so because it was created by a special law and has an original charter. As a component of the Civil Service, UP is therefore governed by PD 807 and administrative cases involving the discipline of its employees come under the appellate jurisdiction of the Civil Service Commission.[39] (Emphasis supplied)In the more recent case of Camacho v. Gloria,[40] this Court lent credence to the concurrent jurisdiction of the CSC when it affirmed that a case against a university official may be filed either with the university’s BOR or directly with the CSC. We quote:

Further, petitioner contends that the creation of the committee by the respondent Secretary, as Chairman of the USP Board of Regents, was contrary to the Civil Service Rules. However, he cites no specific provision of the Civil Service Law which was violated by the respondents in forming the investigating committee. The Civil Service Rules embodied in Executive Order 292 recognize the power of the Secretary and the university, through its governing board, to investigate and decide matters involving disciplinary action against officers and employees under their jurisdiction. Of course under EO 292, a complaint against a state university official may be filed either with the university’s Board of Regents or directly with the Civil Service Commission, although the CSC may delegate the investigation of a complaint and for that purpose, may deputize any department, agency, official or group of officials to conduct such investigation.[41] (Emphasis supplied)

Thus, CSC validly took cognizance of the administrative complaints directly filed before the regional office, concerning violations of civil service rules against respondent.III. Academic freedom may not be invoked when there are alleged violations of civil service laws and rules.Certainly, academic institutions and personnel are granted wide latitude of action under the principle of academic freedom. Academic freedom encompasses the freedom to determine who may teach, who may be taught, how it shall be taught, and who may be admitted to study.[42] Following that doctrine, this Court has recognized that institutions of higher learning has the freedom to decide for itself the best methods to achieve their aims and objectives, free from outside coercion, except when the welfare of the general public so requires.[43] They have the independence to determine who to accept to study in their school and they cannot be compelled by mandamus to enroll a student.[44]That principle, however, finds no application to the facts of the present case. Contrary to the matters traditionally held to be justified to be within the bounds of academic freedom, the administrative complaints filed against Sojor involve violations of civil service rules. He is facing charges of nepotism, dishonesty, falsification of official documents, grave misconduct, and conduct prejudicial to the best interest of the service. These are classified as grave offenses under civil service rules, punishable with suspension or even dismissal.[45]This Court has held that the guaranteed academic freedom does not give an institution the unbridled authority to perform acts without any statutory basis.[46] For that reason, a school official, who is a member of the civil service, may not be permitted to commit violations of civil service rules under the justification that he was free to do so under the principle of academic freedom.Lastly, We do not agree with respondent’s contention that his appointment to the position of president of NORSU, despite the pending administrative cases against him, served as a condonation by the BOR of the alleged acts imputed to him. The doctrine this Court laid down in Salalima v. Guingona, Jr.[47] and Aguinaldo v. Santos[48] are inapplicable to the present circumstances. Respondents in the mentioned cases are elective officials, unlike respondent here who is an appointed official. Indeed, election expresses the sovereign will of the people.[49] Under the principle of voxpopuliestsupremalex, the re-election of a public official may, indeed, supersede a pending administrative case. The same cannot be said of a re-appointment to a non-career position. There is no sovereign will of the people to speak of when the BOR re-appointed respondent Sojor to the post of university president.

WHEREFORE, the petition is GRANTED. The Decision of the Court of Appeals is REVERSED and SET ASIDE. The assailed Resolutions of the Civil Service Commission are REINSTATED.

SO ORDERED.

40 | P a g e

Page 41: JURISDICTION-DIGESTED-OCHOA.docx

JURISDICTION CASE DIGEST TROPANG POTCHI

Republic of the PhilippinesSUPREME COURT

Baguio City

EN BANC

G.R. No. 168766 May 22, 2008

THE CIVIL SERVICE COMMISSION, petitioner, vs.HENRY A. SOJOR, respondent.

D E C I S I O N

REYES, R.T., J.:

IS the president of a state university outside the reach of the disciplinary jurisdiction constitutionally granted to the Civil Service Commission (CSC) over all civil servants and officials?

Does the assumption by the CSC of jurisdiction over a president of a state university violate academic freedom?

The twin questions, among others, are posed in this petition for review on certiorari of the Decision1 of the Court of Appeals (CA) which annulled two (2) CSC Resolutions2 against respondent Henry A. Sojor.

The Facts

The uncontroverted facts that led to the controversy, as found by the CSC and the CA, are as follows:

On August 1, 1991, respondent Sojor was appointed by then President Corazon Aquino as president of the Central Visayas Polytechnic College (CVPC) in Dumaguete City. In June 1997, Republic Act (R.A.) No. 8292, or the "Higher Education Modernization Act of 1997," was enacted. This law mandated that a Board of Trustees (BOT) be formed to act as the governing body in state colleges. The BOT of CVPC appointed respondent as president, with a four-year term beginning September 1998 up to September 2002.3 Upon the expiration of his first term of office in 2002, he was appointed president of the institution for a second four-year term, expiring on September 24, 2006.4

On June 25, 2004, CVPC was converted into the Negros Oriental State University (NORSU).5 A Board of Regents (BOR) succeeded the BOT as its governing body.

Meanwhile, three (3) separate administrative cases against respondent were filed by CVPC faculty members before the CSC Regional Office (CSC-RO) No. VII in Cebu City, to wit:

1. ADMC DC No. 02-20(A) – Complaint for dishonesty, grave misconduct and conduct prejudicial to the best interest of the service filed on June 26, 2002 by Jose Rene A. Cepe and Narciso P. Ragay. It was alleged that respondent approved the release of salary differentials despite the absence of the required Plantilla and Salary Adjustment Form and valid appointments.6

2. ADM DC No. 02-20 – Complaint for dishonesty, misconduct and falsification of official documents filed on July 10, 2002 by Jocelyn Juanon and Carolina Fe Santos. The complaint averred that respondent maliciously allowed the antedating and falsification of the reclassification differential payroll, to the prejudice of instructors and professors who have pending request for adjustment of their academic ranks.7

3. ADM DC No. 02-21 – Complaint for nepotism filed on August 15, 2002 by Rose Marie Palomar, a former part-time instructor of CVPC. It was alleged that respondent appointed his half-sister, EstrellasSojor-Managuilas, as casual clerk, in violation of the provisions against nepotism under the Administrative Code.8

Before filing his counter-affidavits, respondent moved to dismiss the first two complaints on grounds of lack of jurisdiction, bar by prior judgment and forum shopping.

He claimed that the CSC had no jurisdiction over him as a presidential appointee. Being part of the non-competitive or unclassified service of the government, he was exclusively under the disciplinary jurisdiction of the Office of the President (OP). He argued that CSC had no authority to entertain, investigate and resolve charges against him; that the Civil Service Law contained no provisions on the investigation, discipline, and removal of presidential appointees. He also pointed out that the subject matter of the complaints had already been resolved by the Office of the Ombudsman.9

Finding no sufficient basis to sustain respondent’s arguments, the CSC-RO denied his motion to dismiss in its Resolution dated September 4, 2002.10 His motion for reconsideration11 was likewise denied. Thus, respondent was formally charged with three administrative cases, namely: (1) Dishonesty, Misconduct, and Falsification of Official Document; (2) Dishonesty, Grave Misconduct, and Conduct Prejudicial to the Best Interest of the Service; and (3) Nepotism.12

Respondent appealed the actions of the regional office to the Commission proper (CSC), raising the same arguments in his motion to dismiss.13 He argued that since the BOT is headed by the Committee on Higher Education Chairperson who was under the OP, the BOT was also under the OP. Since the president of CVPC was appointed by the BOT, then he was a presidential appointee. On the matter of the jurisdiction granted to

CSC by virtue of Presidential Decree (P.D.) No. 80714 enacted in October 1975, respondent contended that this was superseded by the provisions of R.A. No. 8292,15 a later law which granted to the BOT the power to remove university officials.

CSC Disposition

In a Resolution dated March 30, 2004,16 the CSC dismissed respondent’s appeal and authorized its regional office to proceed with the investigation. He was also preventively suspended for 90 days. The fallo of the said resolution states:

WHEREFORE, the appeal of Henry A. Sojor, President of Central Visayas Polytechnic College, is hereby DISMISSED. The Civil Service Commission Regional Office No. VII, Cebu City, is authorized to proceed with the formal investigation of the cases against Sojor and submit the investigation reports to the Commission within one hundred five (105) days from receipt hereof. Finally, Sojor is preventively suspended for ninety (90) days.17

In decreeing that it had jurisdiction over the disciplinary case against respondent, the CSC opined that his claim that he was a presidential appointee had no basis in fact or in law. CSC maintained that it had concurrent jurisdiction with the BOT of the CVPC. We quote:

His appointment dated September 23, 2002 was signed by then Commission on Higher Education (CHED) Chairman Ester A. Garcia. Moreover, the said appointment expressly stated that it was approved and adopted by the Central Visayas Polytechnic College Board of Trustees on August 13, 2002 in accordance with Section 6 of Republic Act No. 8292 (Higher education Modernization Act of 1997), which explicitly provides that, "He (the president of a state college) shall be appointed by the Board of Regents/Trustees, upon recommendation of a duly constituted search committee." Since the President of a state college is appointed by the Board of Regents/Trustees of the college concerned, it is crystal clear that he is not a presidential appointee. Therefore, it is without doubt that Sojor, being the President of a state college (Central Visayas Polytechnic College), is within the disciplinary jurisdiction of the Commission.

The allegation of appellant Sojor that the Commission is bereft of disciplinary jurisdiction over him since the same is exclusively lodged in the CVPC Board of Trustees, being the appointing authority, cannot be considered. The Commission and the CVPC Board of Trustees have concurrent jurisdiction over cases against officials and employees of the said agency. Since the three (3) complaints against Sojor were filed with the Commission and not with the CVPC, then the former already acquired disciplinary jurisdiction over the appellant to the exclusion of the latter agency.18 (Emphasis supplied)

The CSC categorized respondent as a third level official, as defined under its rules, who are under the jurisdiction of the Commission proper. Nevertheless, it adopted the formal charges issued by its regional office and ordered it to proceed with the investigation:

41 | P a g e

Page 42: JURISDICTION-DIGESTED-OCHOA.docx

JURISDICTION CASE DIGEST TROPANG POTCHI

Pursuant to the Uniform Rules on Administrative Cases in the Civil Service, Sojor, being a third level official, is within the disciplinary jurisdiction of the Commission Proper. Thus, strictly speaking, the Commission has the sole jurisdiction to issue the formal charge against Sojor. x xx However, since the CSC RO No. VII already issued the formal charges against him and found merit in the said formal charges, the same is adopted. The CSC RO No. VII is authorized to proceed with the formal investigation of the case against Sojor in accordance with the procedure outlined in the aforestated Uniform Rules.19 (Emphasis supplied)

No merit was found by the CSC in respondent’s motion for reconsideration and, accordingly, denied it with finality on July 6, 2004.20

Respondent appealed the CSC resolutions to the CA via a petition for certiorari and prohibition. He alleged that the CSC acted without or in excess of its jurisdiction, or with grave abuse of discretion amounting to lack or excess of jurisdiction when it issued the assailed resolutions; that CSC encroached upon the academic freedom of CVPC; and that the power to remove, suspend, and discipline the president of CVPC was exclusively lodged in the BOT of CVPC.

CA Disposition

On September 29, 2004, the CA issued a writ of preliminary injunction directing the CSC to cease and desist from enforcing its Resolution dated March 30, 2004 and Resolution dated July 6, 2004.21 Thus, the formal investigation of the administrative charges against Sojor before the CSC-RO was suspended.

On June 27, 2005, after giving both parties an opportunity to air their sides, the CA resolved in favor of respondent. It annulled the questioned CSC resolutions and permanently enjoined the CSC from proceeding with the administrative investigation. The dispositive part of the CA decision reads:

WHEREFORE, in view of all the foregoing, and finding that the respondent Civil Service Commission acted without jurisdiction in issuing the assailed Resolution Nos. 040321 and 040766 dated March 20, 2004 and July 6, 2004, respectively, the same are hereby ANNULLED and SET ASIDE. The preliminary injunction issued by this Court on September 29, 2004 is hereby made permanent.

SO ORDERED.22

The CA ruled that the power to appoint carries with it the power to remove or to discipline. It declared that the enactment of R.A. No. 929923 in 2004, which converted CVPC into NORSU, did not divest the BOT of the power to discipline and remove its faculty members, administrative officials, and employees. Respondent was appointed as president of CVPC by the BOT by virtue of the authority granted to it under Section 6 of R.A. No. 8292.24 The power of the BOT to remove and discipline erring employees, faculty members, and administrative officials as expressly provided for under Section 4 of R.A. No. 8292 is also granted to the BOR of NORSU under Section 7 of R.A. No. 9299. The said provision reads:

Power and Duties of Governing Boards. – The governing board shall have the following specific powers and duties in addition to its general powers of administration and exercise of all the powers granted to the board of directors of a corporation under Section 36 of Batas PambansaBlg. 68, otherwise known as the Corporation Code of the Philippines:

x xxx

to fix and adjust salaries of faculty members and administrative officials and employees x xx; and to remove them for cause in accordance with the requirements of due process of law. (Emphasis added)

The CA added that Executive Order (E.O.) No. 292,25 which grants disciplinary jurisdiction to the CSC over all branches, subdivisions, instrumentalities, and agencies of the government, including government-owned or controlled corporations with original charters, is a general law. According to the appellate court, E.O. No. 292 does not prevail over R.A. No. 9299,26 a special law.

Issues

Petitioner CSC comes to Us, seeking to reverse the decision of the CA on the ground that THE COURT OF APPEALS GRAVELY ERRED IN HOLDING THAT PETITIONER ACTED WITHOUT JURISDICTION IN ISSUING RESOLUTION NO. 040321 DATED MARCH 30, 2004 AND RESOLUTION NO. 04766 DATED JULY 6, 2004.27

Our Ruling

The petition is meritorious.

I. Jurisdiction of the CSC

The Constitution grants to the CSC administration over the entire civil service.28 As defined, the civil service embraces every branch, agency, subdivision, and instrumentality of the government, including every government-owned or controlled corporation.29 It is further classified into career and non-career service positions. Career service positions are those where: (1) entrance is based on merit and fitness or highly technical qualifications; (2) there is opportunity for advancement to higher career positions; and (3) there is security of tenure. These include:

(1) Open Career positions for appointment to which prior qualification in an appropriate examination is required;

(2) Closed Career positions which are scientific, or highly technical in nature; these include the faculty and academic staff of state colleges and universities, and scientific and technical positions in scientific or research institutions which shall establish and maintain their own merit systems;

(3) Positions in the Career Executive Service; namely, Undersecretary, Assistant Secretary, Bureau Director, Assistant Bureau Director, Regional Director, Assistant Regional Director, Chief of Department Service and other officers of equivalent rank as may be identified by the Career Executive Service Board, all of whom are appointed by the President;

(4) Career officers, other than those in the Career Executive Service, who are appointed by the President, such as the Foreign Service Officers in the Department of Foreign Affairs;

(5) Commissioned officers and enlisted men of the Armed Forces which shall maintain a separate merit system;

(6) Personnel of government-owned or controlled corporations, whether performing governmental or proprietary functions, who do not fall under the non-career service; and

(7) Permanent laborers, whether skilled, semi-skilled, or unskilled.30

Career positions are further grouped into three levels. Entrance to the first two levels is determined through competitive examinations, while entrance to the third level is prescribed by the Career Executive Service Board.31 The positions covered by each level are:

(a) The first level shall include clerical, trades, crafts, and custodial service positions which involve non-professional or subprofessional work in a non-supervisory or supervisory capacity requiring less than four years of collegiate studies;

(b) The second level shall include professional, technical, and scientific positions which involve professional, technical, or scientific work in a non-supervisory or supervisory capacity requiring at least four years of college work up to Division Chief level; and

(c) The third level shall cover positions in the Career Executive Service.32

On the other hand, non-career service positions are characterized by: (1) entrance not by the usual tests of merit and fitness; and (2) tenure which is limited to a period specified by law, coterminous with the appointing authority or subject to his pleasure, or limited to the duration of a particular project for which purpose employment was made.33 The law states:

The Non-Career Service shall include:

(1) Elective officials and their personal or confidential staff;

(2) Secretaries and other officials of Cabinet rank who hold their positions at the pleasure of the President and their personal or confidential staff(s);

42 | P a g e

Page 43: JURISDICTION-DIGESTED-OCHOA.docx

JURISDICTION CASE DIGEST TROPANG POTCHI

(3) Chairman and members of commissions and boards with fixed terms of office and their personal or confidential staff;

(4) Contractual personnel or those whose employment in the government is in accordance with a special contract to undertake a specific work or job, requiring special or technical skills not available in the employing agency, to be accomplished within a specific period, which in no case shall exceed one year, and performs or accomplishes the specific work or job, under his own responsibility with a minimum of direction and supervision from the hiring agency; and

(5) Emergency and seasonal personnel.34

It is evident that CSC has been granted by the Constitution and the Administrative Code jurisdiction over all civil service positions in the government service, whether career or non-career. From this grant of general jurisdiction, the CSC promulgated the Revised Uniform Rules on Administrative Cases in the Civil Service.35 We find that the specific jurisdiction, as spelled out in the CSC rules, did not depart from the general jurisdiction granted to it by law. The jurisdiction of the Regional Office of the CSC and the Commission central office (Commission Proper) is specified in the CSC rules as:

Section 4. Jurisdiction of the Civil Service Commission. – The Civil Service Commission shall hear and decide administrative cases instituted by, or brought before it, directly or on appeal, including contested appointments, and shall review decisions and actions of its offices and of the agencies attached to it.

Except as otherwise provided by the Constitution or by law, the Civil Service Commission shall have the final authority to pass upon the removal, separation and suspension of all officers and employees in the civil service and upon all matters relating to the conduct, discipline and efficiency of such officers and employees.

Section 5. Jurisdiction of the Civil Service Commission Proper. – The Civil Service Commission Proper shall have jurisdiction over the following cases:

A. Disciplinary

1. Decisions of Civil Service Regional Offices brought before it on petition for review;

2. Decisions of heads of departments, agencies, provinces, cities, municipalities and other instrumentalities, imposing penalties exceeding thirty days suspension or fine in an amount exceeding thirty days salary brought before it on appeal;

3. Complaints brought against Civil Service Commission Proper personnel;

4. Complaints against third level officials who are not presidential appointees;

5. Complaints against Civil Service officials and employees which are not acted upon by the agencies and such other complaints requiring direct or immediate action, in the interest of justice;

6. Requests for transfer of venue of hearing on cases being heard by Civil Service Regional Offices;

7. Appeals from the Order of Preventive Suspension; and

8. Such other actions or requests involving issues arising out of or in connection with the foregoing enumerations.

B. Non-Disciplinary

1. Decisions of Civil Service Commission Regional Offices brought before it;

2. Requests for favorable recommendation on petition for executive clemency;

3. Protests against the appointment, or other personnel actions, involving third level officials; and

4. Such other analogous actions or petitions arising out of or in relation with the foregoing enumerations.

Section 6. Jurisdiction of Civil Service Regional Offices. – The Civil Service Commission Regional Offices shall have jurisdiction over the following cases:

A. Disciplinary

1. Complaints initiated by, or brought before, the Civil Service Commission Regional Offices provided that the alleged acts or omissions were committed within the jurisdiction of the Regional Office, including Civil Service examination anomalies or irregularities and the persons complained of are employees of agencies, local or national, within said geographical areas;

2. Complaints involving Civil Service Commission Regional Office personnel who are appointees of said office; and

3. Petitions to place respondent under Preventive Suspension.

B. Non-Disciplinary

1. Disapproval of appointments brought before it on appeal;

2. Protests against the appointments of first and second level employees brought before it directly or on appeal. (Emphasis supplied)

Respondent, a state university president with a fixed term of office appointed by the governing board of trustees of the university, is a non-career civil service officer. He was appointed by the chairman and members of the governing board of CVPC. By clear provision of law, respondent is a non-career civil servant who is under the jurisdiction of the CSC.

II. The power of the BOR to discipline officials and employees is not exclusive. CSC has concurrent jurisdiction over a president of a state university.

Section 4 of R.A. No. 8292, or the Higher Education Modernization Act of 1997, under which law respondent was appointed during the time material to the present case, provides that the school’s governing board shall have the general powers of administration granted to a corporation. In addition, Section 4 of the law grants to the board the power to remove school faculty members, administrative officials, and employees for cause:

Section 4. Powers and Duties of Governing Boards. – The governing board shall have the following specific powers and duties in addition to its general powers of administration and the exercise of all the powers granted to the board of directors of a corporation under Section 36 of Batas PambansaBlg. 68, otherwise known as the Corporation Code of the Philippines:

x xxx

h) to fix and adjust salaries of faculty members and administrative officials and employees subject to the provisions of the revised compensation and classification system and other pertinent budget and compensation laws governing hours of service, and such other duties and conditions as it may deem proper; to grant them, at its discretion, leaves of absence under such regulations as it may promulgate, any provisions of existing law to the contrary not withstanding; and to remove them for cause in accordance with the requirements of due process of law. (Emphasis supplied)

The above section was subsequently reproduced as Section 7(i) of the succeeding law that converted CVPC into NORSU, R.A. No. 9299. Notably, and in contrast with the earlier law, R.A. No. 9299 now provides that the administration of the university and exercise of corporate powers of the board of the school shall be exclusive:

Sec. 4. Administration. – The University shall have the general powers of a corporation set forth in Batas PambansaBlg. 68, as amended, otherwise known as "The Corporation Code of the Philippines." The administration of the University and the exercise of its corporate powers shall be vested exclusively in the Board of Regents and the president of the University insofar as authorized by the Board.

43 | P a g e

Page 44: JURISDICTION-DIGESTED-OCHOA.docx

JURISDICTION CASE DIGEST TROPANG POTCHI

Measured by the foregoing yardstick, there is no question that administrative power over the school exclusively belongs to its BOR. But does this exclusive administrative power extend to the power to remove its erring employees and officials?

In light of the other provisions of R.A. No. 9299, respondent’s argument that the BOR has exclusive power to remove its university officials must fail. Section 7 of R.A. No. 9299 states that the power to remove faculty members, employees, and officials of the university is granted to the BOR "in addition to its general powers of administration." This provision is essentially a reproduction of Section 4 of its predecessor, R.A. No. 8292, demonstrating that the intent of the lawmakers did not change even with the enactment of the new law. For clarity, the text of the said section is reproduced below:

Sec. 7. Powers and Duties of the Board of Regents. – The Board shall have the following specific powers and duties in addition to its general powers of administration and the exercise of all the powers granted to the Board of Directors of a corporation under existing laws:

x xxx

i. To fix and adjust salaries of faculty members and administrative officials and employees, subject to the provisions of the Revised Compensation and Position Classification System and other pertinent budget and compensation laws governing hours of service and such other duties and conditions as it may deem proper; to grant them, at its discretion, leaves of absence under such regulations as it may promulgate, any provision of existing law to the contrary notwithstanding; and to remove them for cause in accordance with the requirements of due process of law.36 (Emphasis supplied)

Verily, the BOR of NORSU has the sole power of administration over the university. But this power is not exclusive in the matter of disciplining and removing its employees and officials.

Although the BOR of NORSU is given the specific power under R.A. No. 9299 to discipline its employees and officials, there is no showing that such power is exclusive. When the law bestows upon a government body the jurisdiction to hear and decide cases involving specific matters, it is to be presumed that such jurisdiction is exclusive unless it be proved that another body is likewise vested with the same jurisdiction, in which case, both bodies have concurrent jurisdiction over the matter.37

All members of the civil service are under the jurisdiction of the CSC, unless otherwise provided by law. Being a non-career civil servant does not remove respondent from the ambit of the CSC. Career or non-career, a civil service official or employee is within the jurisdiction of the CSC.

This is not a case of first impression.

In University of the Philippines v. Regino,38 this Court struck down the claim of exclusive jurisdiction of the UP BOR to discipline its employees. The Court held then:

The Civil Service Law (PD 807) expressly vests in the Commission appellate jurisdiction in administrative disciplinary cases involving members of the Civil Service. Section 9(j) mandates that the Commission shall have the power to "hear and decide administrative disciplinary cases instituted directly with it in accordance with Section 37 or brought to it on appeal." And Section 37(a) provides that, "The Commission shall decide upon appeal all administrative disciplinary cases involving the imposition of a penalty of suspension for more than thirty (30) days, or fine in an amount exceeding thirty days’ salary, demotion in rank or salary or transfer, removal or dismissal from office." (Emphasis supplied)

Under the 1972 Constitution, all government-owned or controlled corporations, regardless of the manner of their creation, were considered part of the Civil Service. Under the 1987 Constitution, only government-owned or controlled corporations with original charters fall within the scope of the Civil Service pursuant to Article IX-B, Section 2(1), which states:

"The Civil Service embraces all branches, subdivisions, instrumentalities, and agencies of the government, including government-owned or controlled corporations with original charters."

As a mere government-owned or controlled corporation, UP was clearly a part of the Civil Service under the 1973 Constitution and now continues to be so because it was created by a special law and has an original charter. As a component of the Civil Service, UP is therefore governed by PD 807 and administrative cases involving the discipline of its employees come under the appellate jurisdiction of the Civil Service Commission.39 (Emphasis supplied)

In the more recent case of Camacho v. Gloria,40 this Court lent credence to the concurrent jurisdiction of the CSC when it affirmed that a case against a university official may be filed either with the university’s BOR or directly with the CSC. We quote:

Further, petitioner contends that the creation of the committee by the respondent Secretary, as Chairman of the USP Board of Regents, was contrary to the Civil Service Rules. However, he cites no specific provision of the Civil Service Law which was violated by the respondents in forming the investigating committee. The Civil Service Rules embodied in Executive Order 292 recognize the power of the Secretary and the university, through its governing board, to investigate and decide matters involving disciplinary action against officers and employees under their jurisdiction. Of course under EO 292, a complaint against a state university official may be filed either with the university’s Board of Regents or directly with the Civil Service Commission, although the CSC may delegate the investigation of a complaint and for that purpose, may deputize any department, agency, official or group of officials to conduct such investigation.41 (Emphasis supplied)

Thus, CSC validly took cognizance of the administrative complaints directly filed before the regional office, concerning violations of civil service rules against respondent.

III. Academic freedom may not be invoked when there are alleged violations of civil service laws and rules.

Certainly, academic institutions and personnel are granted wide latitude of action under the principle of academic freedom. Academic freedom encompasses the freedom to determine who may teach, who may be taught, how it shall be taught, and who may be admitted to study.42 Following that doctrine, this Court has recognized that institutions of higher learning has the freedom to decide for itself the best methods to achieve their aims and objectives, free from outside coercion, except when the welfare of the general public so requires.43 They have the independence to determine who to accept to study in their school and they cannot be compelled by mandamus to enroll a student.44

That principle, however, finds no application to the facts of the present case. Contrary to the matters traditionally held to be justified to be within the bounds of academic freedom, the administrative complaints filed against Sojor involve violations of civil service rules. He is facing charges of nepotism, dishonesty, falsification of official documents, grave misconduct, and conduct prejudicial to the best interest of the service. These are classified as grave offenses under civil service rules, punishable with suspension or even dismissal.45

This Court has held that the guaranteed academic freedom does not give an institution the unbridled authority to perform acts without any statutory basis.46 For that reason, a school official, who is a member of the civil service, may not be permitted to commit violations of civil service rules under the justification that he was free to do so under the principle of academic freedom.

Lastly, We do not agree with respondent’s contention that his appointment to the position of president of NORSU, despite the pending administrative cases against him, served as a condonation by the BOR of the alleged acts imputed to him. The doctrine this Court laid down in Salalima v. Guingona, Jr.47 and Aguinaldo v. Santos48 are inapplicable to the present circumstances. Respondents in the mentioned cases are elective officials, unlike respondent here who is an appointed official. Indeed, election expresses the sovereign will of the people.49Under the principle of voxpopuliestsupremalex, the re-election of a public official may, indeed, supersede a pending administrative case. The same cannot be said of a re-appointment to a non-career position. There is no sovereign will of the people to speak of when the BOR re-appointed respondent Sojor to the post of university president.

WHEREFORE, the petition is GRANTED. The Decision of the Court of Appeals is REVERSED and SET ASIDE. The assailed Resolutions of the Civil Service Commission are REINSTATED.

SO ORDERED.

Republic of the PhilippinesSUPREME COURT

Manila

EN BANC

44 | P a g e

Page 45: JURISDICTION-DIGESTED-OCHOA.docx

JURISDICTION CASE DIGEST TROPANG POTCHI

G.R. No. L-23537 March 31, 1965

ASSOCIATED LABOR UNION, DEMOCRITO T. MENDOZA and CECILIO T. SENO, petitioners, vs.THE HON. JUDGE MODESTO R. RAMOLETE of the Court of First Instance of Cebu, KATIPUNAN LUMBER CO., INC., and ROQUE ABELLAR, respondents.

PAREDES, J.:

Respondent Katipunan Lumber Co., Inc. (Katipunan for short), is engaged in the Lumber business, and maintains a regular and permanent staff of office employees, drivers and laborers, who perform the routinary phases of its operations, and who are affiliated with the Cebu Industrial Labor Organization, a duly accredited labor union. Regarding the other phases of its business, specially those which are occasional, it engages, as all other firms dealing in the same kind of business, independent labor contractors, the compensation of which was dependent upon the work done, so much per thousand board feet of lumber hauled, piled, transferred and/or classified. For the handling and hauling, loading and unloading of lumber cargoes from the pier to the lumberyard of Katipunan, Dionisio Antioquia was engaged; the City Trucking Service, Inc., took care of handling the hauling, loading and unloading of plywood cargoes from pier to bodega or from bodega to pier and other places; Luis Amores was in charge of the classification, piling and transfer of stocks from one place to another inside the lumberyard; and CiriloCabasa, other manual work not included in the routinary exigencies of work within the lumberyard. Since the case at bar arose out of the contract of CiriloCabasa with Katipunan, We will refrain from discussing the participation of the other independent contractors.

CiriloCabasa had an existing contract with Katipunan to supply all the laborers occasionally needed by it on the jobs not assigned to other independent contractors and not embraced in the regular routinary conduct of business. On August 18, 1964, Cabasa asked for the termination of his contract. RoqueAbellar who also had similar contracts, with other lumber companies in Cebu City, entered into a written contract with respondent Katipunan, to furnish all the labor needed by the latter, in connection with its business phases, before performed by Cabasa.

On September 3, 1964, Katipunan and RoqueAbelar, filed with the CFI of Cebu, presided over by the Honorable Modesto Ramolete, a complaint for Injunction and Damages, with a Preliminary Injunction (Civil Case No. R-8564), against Democrito T. Mendoza, Cecilio T. Seno and the Associated Labor Union. In the complaint, besides stating the jurisdictional facts and the harassing and coercive tactics, threats, cajoleries and other overt acts which Katipunan claimed to be an illegal interference by the respondents therein, in the contractual obligations of Katipunan and Abellar, it was also alleged that there was absolutely no employer-employee relationship between the Katipunan and the laborers of the independent contractor Abellar, the latter being strictly responsible in matters of control and supervision.

On September 8, 1964, respondent Judge Ramolete handed down an Order, the pertinent portions of which recite:

The allegations of facts of the plaintiffs and the further fact that the said plaintiffs will suffer great and irreparable damage unless the acts complained of by the defendants will be restrained by a writ of preliminary injunction. The Court believes that the writ may be issued upon the plaintiffs' filing a bond of P50,000.00 to guarantee the damages that may be suffered by the defendants in case the issuance of the writ is not justified under the authority of Section 3 of Rule 58 of the Rules of Court.

WHEREFORE, considering the request for the issuance in the meantime of the writ of preliminary injunction justified and authorized under the provisions of the said Rules of Court, the Court hereby grants the request and let a writ of preliminary injunction be issued upon plaintiffs' filing of a bond of P50,000.00 to answer for any damage that may be caused on the defendants by the issuance of the writ, restraining the said defendants, their agents, laborers, officials and representatives from (a) blocking and disturbing the passage of trucks used by co-plaintiff RoqueAbellar in the course of the performance of the duties assumed by him under his contract with plaintiff Katipunan Lumber Co., Inc., especially in the entry and exit of the same to and from the premises of latter plaintiff's lumberyard; (b) disturbing and molesting the laborers of co-plaintiff RoqueAbellar in the piers, wharf and other places of loading and unloading of the lumber, plywood and other construction materials, including the premises of plaintiff Katipunan Lumber Co., Inc.; (c) persuading, cajoling and/or coercing shipping companies into refusing to accept the shipment of incoming and outgoing cargoes of plaintiff Katipunan Lumber Co., Inc., and (d) persuading and/or threatening plaintiff Katipunan Lumber Co., Inc.'s customers and dealers into refusing to purchase or deal with.

Defendants presented a Motion for Reconsideration and for Lifting of the Writ of Preliminary Injunction, claiming that —

(1) the Court did not have jurisdiction to issue the restraining order in cases of the same nature as the one at bar;

(2) that peaceful picketing cannot be lawfully enjoined by any court in this jurisdiction;

(3) that the requisites enumerated in Section 9 (d) of Republic Act 875 should have been followed and not those provided in Section 3, Rule 58 of the Rules of Court in issuing the restraining Order in cases of the same nature;

(4) that there is already pending an Unfair Labor practice case involving the labor dispute presented with the CIR, by the ALU against the Katipunan Lumber Co., Inc., where all the incidents relative to the parties should be ventilated.

Plaintiffs therein, now private-parties respondents, on September 12, 1964, opposed the motion for reconsideration, arguing that the Court had jurisdiction for the allegations in the complaint are the basis in determining jurisdiction; that what respondent Court had enjoined was not a peaceful picketing, but interference in the performance of the contractual obligations of the plaintiffs; that Section 9 (d) of Republic Act 875 does not apply; and that there was no unfair labor practice case actually filed against Katipunan, although there was a charge, which had not yet been investigated by the CIR.

Without waiting for resolution of their Motion for Reconsideration and to lift the injunction, petitioners came to this Court on a Petition for Certiorari and Prohibition with Preliminary Injunction, raising the same issues alleged in their motion for reconsideration in the lower court. In effect, their petition centers on their assumption that the matter between the Katipunan Lumber and Abellar on one hand, and petitioners on the other, is a labor dispute of which the CFI cannot validly take cognizance, and that in entertaining the same and in issuing the Writ of Preliminary Injunction, respondent Judge acted without or in excess of jurisdiction and/or with grave abuse of discretion and there is no appeal, nor any plain, speedy and adequate remedy in the ordinary course of law.

We gave Due Course to the petition and directed the issuance of a Writ of Preliminary Injunction as prayed for upon the posting of a P1,000.00 bond, and ordered respondents to answer. Before the Writ could be issued, however, private-parties respondents presented a Manifestation, inviting the attention of this Court to the fact that the petition under consideration was filed prematurely, since there was, at the time, pending resolution by the respondent Court, the Motion for Reconsideration and Lifting of the Writ. This Court suspended the issuance of the Injunctive Writ, and ordered petitioners to answer the manifestation.

In a Very Urgent Reply to Respondents' Manifestation, petitioners alleged that since the case involves a labor dispute, the respondent court cannot validly take cognizance thereof, and while they did not wait for the resolution of their motion for reconsideration, their case falls within the exception, for there existed special reasons for not exhausting all the remedies in the lower court, such as the enforcement of the ex-parte writ, which resulted in the paralyzation of their right to self-organization, to picket and to otherwise promote the collective welfare of petitioners and the members of the ALU. Thereafter, several incidents had taken place, which need not be considered, since they are not necessary in the determination of the issues presently involved.

Under date of September 25, 1964, private-parties respondents filed an Urgent Motion for Dismissal, claiming that the petition was manifestly premature, and that the determination of whether the lower court had jurisdiction over the case was dependent upon facts which must be shown before respondent Court. Attached to said urgent motion was the Order of the respondent Court, dated September 17, 1964, the pertinent portions of which read:

... Under these circumstances based on the opposing mere allegations of the parties in which no evidence so far has been formally presented, the Court finds itself unable to determine whether the present case involves labor dispute or unfair labor practice, or it is merely a simple civil case involving the restraining of some unlawful and illegal acts committed and continued to be committed by the defendants.

For these reasons the Court instead of resolving to lift or to maintain the writ already issued defers the resolution of the defendants' motion until evidence is presented by the parties to prove their respective allegations and contentions, and, in order to expedite matters, the hearing and reception of evidence of the parties on the main case shall be set by the Clerk of Court as soon as the issues are joined; and the defendants are required to answer the complaint within the legal period counted from the time they are served copies of this order embodying in their answer all the grounds they

45 | P a g e

Page 46: JURISDICTION-DIGESTED-OCHOA.docx

JURISDICTION CASE DIGEST TROPANG POTCHI

alleged in their motion so that the issues may be put squarely before the Court for its decision.1äwphï1.ñët

WHEREFORE, the consideration and resolution of the defendants' motion for reconsideration and lifting of the writ of preliminary injunction are hereby deferred until evidence is presented by the parties to establish their respective allegations and contentions and the defendants are required to answer within the reglementary period the plaintiffs' complaint; and, as soon as the issues are joined the Clerk of Court shall immediately calendar the hearing of the main case for the reception of the evidence of the parties with due notice to them. (Emphasis supplied.)

Both parties, in the interim, filed various pleadings. On October 26, 1964, respondents presented a Manifestation inviting the attention of this Court to the fact that aside from the petitioners, filing their Answer to the Complaint,they have also availed and/or actually commenced to take advantage of the remedies available to them before the respondent Court. So much so, that on October 23, 1964, respondent Judge entered a Pre-Trial Order, which contained matters agreed upon to be proved by the parties and those which they did not dispute. On November 17, 1964, private-parties respondents, filed their Answer to the Petition for Certiorari and Prohibition, and invoked the following defenses, to wit:

1. The instant petition is premature;

2. Petitioners have speedy and adequate remedy before the lower court which they failed to exhaust before presenting the instant petition;

3. The instant proceedings have been rendered moot and academic, because petitioners have actually taken advantage of the remedies available to them in the court below;

4. That whether the case involves a labor dispute could only be determined after a hearing, which the respondent court has already ordered;

5. That the principal cause of action as alleged in the complaint is for the recovery of P50,000.00 as damages, a matter of which the respondent Court has jurisdiction, and all orders issued in connection therewith being within its power and authority;

6. That the nature of an action as appearing in the complaint, cannot be changed nor varied by the impleading by the defendants of defenses tending to change the cause of action;

7. The alleged failure of respondent judge to observe the procedural requirements of section 9(a) of Act 875, merely amounted to an error of judgment, not reviewable by certiorari;

8. That the acts enjoined by respondent judge are not the acts insulated from injunction under section 9(a) of Act 875;

9. That there is no pending Unfair Labor Practice Case against the Katipunan Lumber; and

10. That petitioner are amply protected of any damage that may be caused them with the P50,000.00 injunction bond posted by respondents.

A panoramic appraisal of the facts of the case will show that the issue primarily involved is — whether the respondent Court had jurisdiction over the case, as the allegation of the complaint indicates, and/or there was grave abuse of discretion on the part of the respondent Court in issuing the injunctive writ, so that certiorari or prohibition would lie.

No plausible argument could be offered to dispute the proposition that what confers jurisdiction are the allegations of the complaint. In the case at bar, the plaintiffs sought the amount of P50,000.00 by way of damages on overt acts, which they considered illegal, and which had caused them losses. They also asserted that there existed no employer-employee relationship whatsoever between them. Generally, therefore, upon such allegations, the CFI had jurisdiction over the case and it was authorized under the Rules of Court to issue an injunctive writ, even ex parte, upon a valid showing of the necessity thereof. It is true that petitioners herein, in their motion for reconsideration and to lift the writ of injunction, they alleged that there is a labor dispute. This mere allegation did not serve to automatically deprive the Court of its jurisdiction duly conferred by the allegations of the complaint. In the wake of the assertions in the motion for reconsideration that there was, in the opinion of the petitioners, a labor dispute, the respondent Court was duty bound to find out if such a circumstance really existed. In order to intelligently form an opinion regarding the matter, respondent Court ordered the presentation of evidence by both parties. Unfortunately, however, petitioners, without waiting for the resolution of the court a quo on their motion for reconsideration and to lift the injunction, they filed with this Court the instant petition. In the case of Villa-Rey Transit, vs. Hon. E. Bello, G.R. No. L-18957. April 23, 1963, We said:

True, that petitioner had filed a motion to lift order of default, and a motion for new trial and to set aside the default judgment, but before they could be resolved, petitioner had already brought the matter to this Court, on a petition for Certiorari and Injunction, without giving the respondent Court an opportunity to pass upon the said motions, which act renders the filing of the present petition premature.

Obviously, therefore, the petition at bar is premature.

There is manifestly, a need for evidence towards proving the allegations pertaining to petitioners' claim of a labor dispute. This act is properly within the power and prerogative of respondent Court. Even petitioners are aware of this fact, for they have already presented their Answer to the complainant and have entered into a pre-trial. When there are other remedies, proper and adequate in the premises, certiorari or prohibition will not lie. Should the lower court ultimately hold that it has jurisdiction, appeal in due time would perhaps be the proper remedy.

Petitioner further ascribe excess of jurisdiction and/or grave abuse of discretion to respondent Court, because it issued the Writ of Preliminary Injunction ex parte and in violation of the provisions of Act No. 875. The Rules properly gives the Court the authority to issue injunctive writs ex parte. Since the cause of action was for damages, arising from what plaintiffs, now respondents, pointed as interference in the performance of contractual obligations, there was no occasion, at that time, for respondent Judge to consider the applicability of Act No. 875. It was only when the matter of an apparent labor dispute was injected by petitioners in their motion for reconsideration, that said act could be considered Judge wanted to hear evidence to assure himself whether there is or excess of jurisdiction and/or grave abuse of discretion. Predicated upon the allegations of the complaint, the respondent Judge though it had jurisdiction, which conclusion, if erroneous, was merely a judicial error or a mistake of law. When the court has jurisdiction over or a subject matter, the orders or decisions upon all questions pertaining to the cause are orders or decisions within its jurisdiction and, however irregular or erroneous they may be, they cannot be corrected by certiorari (Gala v. Cui and Rodriguez, 23 Phil. 522; Galang v. Endencia, 73 Phil. 399; Villa-Rey Transit v. Bello, G.R. No. L-21399, Jan. 31, 1964).

WHEREFORE, the petition for Certiorari and Prohibition with Writ of Preliminary Injunction, should be, as it is hereby dismissed. Costs against petitioner.

46 | P a g e

Page 47: JURISDICTION-DIGESTED-OCHOA.docx

JURISDICTION CASE DIGEST TROPANG POTCHI

Dela Cruz vsMoir

Facts: This is a petition for a writ of certiorari to be directed to the Court of First Instance of Pampanga requiring him to remit to this court all the records and papers relating to a certain election contest pending therein instituted by Juan de la Cruz against Melecio M. Trinidad, that the proceeding had in said contest may be revised by this court and certain of the steps taken therein be declared null and void on the ground that they were taken by the court without or in excess of its jurisdiction.

The petition shows that on the 20th of June, 1916, Juan de la Cruz filed a protest in the Court of First Instance of Pampanga against Melecio M. Trinidad and others challenging the validity of the election of said Trinidad to the office of municipal president of Macabebe, Pampanga. The contest was duly brought on for hearing. After hearing and the presentation of evidence the court dismissed the proceedings on the merits. The particular acts of the court of which complaint is made are set out in the petition in this case as follows:

That in trying and dismissing the said election protest filed by the petitioner, the said respondent the Hon. Percy M. Moir, as judge, of first instance of Pampanga, has exceeded his jurisdiction in that although the said respondent judge admits that there were 193 electors of Macabebe who voted as illiterates and most of whom took no oath regarding their illiteracy before voting and who were aided only by one inspector of election and not by two as required by the election law, yet the said respondent judge has entirely and voluntarily failed, omitted and refused to separate the votes of the said illiterates and to declare the same as null and void, and act which is not only in excess of his jurisdiction but is manifestly against the law, for the votes of illiterates who have not taken the necessary oath regarding their illiteracy before voting and who were aided only by one inspector of election are null and void.

That the respondent Judge failed to make findings of the facts presented wherein the petitioner contends that these facts were sufficient to annul an election.

ISSUE: Whether or not the petitioner filed at the proper venue with a proper petition.

RULING: We are of the opinion that the petition does not state facts sufficient to warrant the relief prayed for. We have held in a long line of decisions that certiorari will not lie under the law of the Philippine Islands except in cases where the court has acted without or in excess of its jurisdiction and the acts thus performed are void. It has been repeatedly held by this court that a writ of certiorari will not be issued unless it clearly appears that the court to which it is to be directed acted without or in excess of jurisdiction. It will not be issued to cure errors in the proceedings or to correct erroneous conclusions of law or of fact. If the court has jurisdiction of the subject-

matter and of the person, decisions upon all question pertaining to the cause are decisions within its jurisdiction and, however irregular or erroneous they may be, cannot be corrected by certiorari

Jurisdiction is the authority to hear and determine a cause - the right to act in a case. Since it is the power to hear and determined, it does not depend either upon the regularity of the exercise of that power or upon the rightfulness of the decisions made. Jurisdiction should therefore be distinguished from the exercise of jurisdiction. The authority to decide a cause at all, and not the decision rendered therein, is what makes up jurisdiction. Where there is jurisdiction of the person and subject-matter, as we have said before, the decision of all other questions arising in the case is but an exercise of that jurisdiction.

A full and thorough examination of all the decided cases in this court touching the question of certiorari and prohibition fully supports the proposition already stated that, where a Court of First Instance has jurisdiction of the subject-matter and of the person, its decision of any question pertaining to the cause, however erroneous, cannot be reviewed by certiorari, but must be corrected by appeal

None of the acts set out in the petition affect the jurisdiction of the court. They are acts performed in the exercise of jurisdiction; and even though the decision of the court upon each one of the questions presented by the allegations of the petition was wrong in fact and in law, his jurisdiction would remain unaffected. As we have said so many times, it is always necessary to bear in mind the difference between jurisdiction and the exercise of jurisdiction. When a court exercises its jurisdiction an error committed while engaged in that exercise does not deprive it of the jurisdiction which it is exercising when the error is committed. If it did, every error committed by a court would deprive if of jurisdiction and every erroneous judgment would be a void judgment. This, of course, is not possible. The administration of justice would not survive such a rule. The decision of the trial court in the case before us holding that certain ballots cast by illiterates which had been prepared for them by the inspectors of election were legal and valid although the illiterates did not, previous to the preparation of their ballots by the inspectors, take the oath required by law showing that they could not cast their ballots without assistance, is a decision entirely within the jurisdiction of the court, even if we concede for the sake or argument that the court was wrong in that decision. The same could be said if he had held the precise contrary and had excluded all of the ballots cast by illiterates; and, of course, it necessarily follows that his failure to separate those which the petitioner claims were illegal from those he claims were legal does not alter the situation. The court has power and authority to conduct the case as he believes law and justice require and whatever he does is within his jurisdiction so long as he does not violate the principle of due process of law or transcend the limits of the case before him.

We reach the same result in discussing the other alleged errors. The denial of the motion to separate the illegal from the legal ballots, the rejection of the offer to prove that the majority of the illiterate voters could read and write, the refusal to annul 62 ballots in favor of Melecio M. Trinidad in spite of the fact that they were marked with

certain letters, the finding that a certain ballot box had been tampered with after the ballots have been counted and the box sealed by the inspectors, the refusal to annul the election in the municipality of Macabebe on account of certain irregularities and violations of law which took place in the conduct thereof, and the counting of more than 40 marked ballots in favor of Trinidad and the refusal to count legal ballots in favor of Juan de la Cruz, do not touch the jurisdiction of the court in any way. They are all decisions made in the exercise of its jurisdiction and however erroneous they may be, if they or any of them are erroneous at all, it had power and authority to make them and they cannot be attacked on the ground of lack of jurisdiction.

On the oral argument it was said by counsel for the petitioner that some of the findings of the trial court were without evidence to support them. There is no allegation to this effect in the petition and no question of that sort is presented by the pleadings. We might say, however, that, even if the question were presented and it were admitted that certain findings were without evidence to support them, that would not mean necessarily that the judgment on the whole case was rendered without jurisdiction and was void. If the judgment was utterly without basis in the record as a whole to such an extent that it was a purely arbitrary act of the court, then a different question might be presented. That is not the case here and we have no reason to consider or decide such a question

The petition is dismissed on the merits, with costs. So ordered.

Republic of the Philippines

SUPREME COURT

Manil

G.R. No. 152642 November 13, 2012

HON. PATRICIA A. STO.TOMAS, ROSALINDA BALDOZ and LUCITA LAZO, Petitioners,

vs.

REY SALAC, WILLIE D. ESPIRITU, MARIO MONTENEGRO, DODGIE BELONIO, LOLIT SALINEL and BUDDY BONNEVIE, Respondents.

G.R. No. 152710

HON. PATRICIA A. STO. TOMAS, in her capacity as Secretary of Department of Labor and Employment (DOLE), HON. ROSALINDA D. BALDOZ, in her capacity as Administrator, Philippine Overseas Employment Administration (POEA), and the PHILIPPINE OVERSEAS EMPLOYMENT ADMINISTRATION GOVERNING BOARD, Petitioners,

47 | P a g e

Page 48: JURISDICTION-DIGESTED-OCHOA.docx

JURISDICTION CASE DIGEST TROPANG POTCHI

vs.

HON. JOSE G. PANEDA, in his capacity as the Presiding Judge of Branch 220, Quezon City, ASIAN RECRUITMENT COUNCIL PHILIPPINE CHAPTER, INC. (ARCOPHIL), for itself and in behalf of its members: WORLDCARE PHILIPPINES SERVIZO INTERNATIONALE, INC., STEADFAST INTERNATIONAL RECRUITMENT CORP., VERDANT MANPOWER MOBILIZATION CORP., BRENT OVERSEAS PERSONNEL, INC., ARL MANPOWER SERVICES, INC., DAHLZEN INTERNATIONAL SERVICES, INC., INTERWORLD PLACEMENT CENTER, INC., LAKAS TAO CONTRACT SERVICES LTD. CO., SSC MULTI-SERVICES, DMJ INTERNATIONAL, and MIP INTERNATIONAL MANPOWER SERVICES, represented by its proprietress, MARCELINA I. PAGSIBIGAN, Respondents.

G.R. No. 167590

REPUBLIC OF THE PHILIPPINES, represented by the HONORABLE EXECUTIVE SECRETARY, the HONORABLE SECRETARY OF LABOR AND EMPLOYMENT (DOLE), the PHILIPPINE OVERSEAS EMPLOYMENT ADMINISTRATION (POEA), the OVERSEAS WORKERS WELFARE ADMINISTRATION (OWWA), the LABOR ARBITERS OF THE NATIONAL LABOR RELATIONS COMMISSION (NLRC), the HONORABLE SECRETARY OF JUSTICE, the HONORABLE SECRETARY OF FOREIGN AFFAIRS and the COMMISSION ON AUDIT (COA), Petitioners,

vs.

PHILIPPINE ASSOCIATION OF SERVICE EXPORTERS, INC. (P ASEI), Respondent.

G.R. Nos. 182978-79

BECMEN SERVICE EXPORTER AND PROMOTION, INC., Petitioner,

vs.

SPOUSES SIMPLICIO AND MILA CUARESMA (for and in behalf of daughter, Jasmin G. Cuaresma), WHITE FALCON SERVICES, INC., and JAIME ORTIZ (President of White Falcon Services, Inc.), Respondents.

G.R. Nos. 184298-99

SPOUSES SIMPLICIO AND MILA CUARESMA (for and in behalf of deceased daughter, Jasmin G. Cuaresma), Petitioners,

vs.

WHITE FALCON SERVICES, INC. and BECMEN SERVICES EXPORTER AND PROMOTION, INC., Respondents.

D E C I S I O N

ABAD, J.:

These consolidated cases pertain to the constitutionality of certain provisions of Republic Act 8042, otherwise known as the Migrant Workers and Overseas Filipinos Act of 1995.

The Facts and the Case

On June 7, 1995 Congress enacted Republic Act (R.A.) 8042 or the Migrant Workers and Overseas Filipinos Act of 1995 that, for among other purposes, sets the Government’s policies on overseas employment and establishes a higher standard of protection and promotion of the welfare of migrant workers, their families, and overseas Filipinos in distress.

G.R. 152642 and G.R. 152710

(Constitutionality of Sections 29 and 30, R.A. 8042)

Sections 29 and 30 of the Act1 commanded the Department of Labor and Employment (DOLE) to begin deregulating within one year of its passage the business of handling the recruitment and migration of overseas Filipino workers and phase out within five years the regulatory functions of the Philippine Overseas Employment Administration (POEA).

On January 8, 2002 respondents Rey Salac, Willie D. Espiritu, Mario Montenegro, DodgieBelonio, LolitSalinel, and Buddy Bonnevie (Salac, et al.) filed a petition for certiorari, prohibition and mandamus with application for temporary restraining order (TRO) and preliminary injunction against petitioners, the DOLE Secretary, the POEA Administrator, and the Technical Education and Skills Development Authority (TESDA) Secretary-General before the Regional Trial Court (RTC) of Quezon City, Branch 96.2

Salac, et al. sought to: 1) nullify DOLE Department Order 10 (DOLE DO 10) and POEA Memorandum Circular 15 (POEA MC 15); 2) prohibit the DOLE, POEA, and TESDA from implementing the same and from further issuing rules and regulations that would regulate the recruitment and placement of overseas Filipino workers (OFWs); and 3) also enjoin them to comply with the policy of deregulation mandated under Sections 29 and 30 of Republic Act 8042.

On March 20, 2002 the Quezon City RTC granted Salac, et al.’s petition and ordered the government agencies mentioned to deregulate the recruitment and placement of OFWs.3 The RTC also annulled DOLE DO 10, POEA MC 15, and all other orders, circulars and issuances that are inconsistent with the policy of deregulation under R.A. 8042.

Prompted by the RTC’s above actions, the government officials concerned filed the present petition in G.R. 152642 seeking to annul the RTC’s decision and have the same enjoined pending action on the petition.

On April 17, 2002 the Philippine Association of Service Exporters, Inc. intervened in the case before the Court, claiming that the RTC March 20, 2002 Decision gravely affected them since it paralyzed the deployment abroad of OFWs and performing artists. The Confederated Association of Licensed Entertainment Agencies, Incorporated (CALEA) intervened for the same purpose.4

On May 23, 2002 the Court5 issued a TRO in the case, enjoining the Quezon City RTC, Branch 96, from enforcing its decision.

In a parallel case, on February 12, 2002 respondents Asian Recruitment Council Philippine Chapter, Inc. and others (Arcophil, et al.) filed a petition for certiorari and prohibition with application for TRO and preliminary injunction against the DOLE Secretary, the POEA Administrator, and the TESDA Director-General,6 before the RTC of Quezon City, Branch 220, to enjoin the latter from implementing the 2002 Rules and Regulations Governing the Recruitment and Employment of Overseas Workers and to cease and desist from issuing other orders, circulars, and policies that tend to regulate the recruitment and placement of OFWs in violation of the policy of deregulation provided in Sections 29 and 30 of R.A. 8042.

On March 12, 2002 the Quezon City RTC rendered an Order, granting the petition and enjoining the government agencies involved from exercising regulatory functions over the recruitment and placement of OFWs. This prompted the DOLE Secretary, the POEA Administrator, and the TESDA Director-General to file the present action in G.R. 152710. As in G.R. 152642, the Court issued on May 23, 2002 a TRO enjoining the Quezon City RTC, Branch 220 from enforcing its decision.

On December 4, 2008, however, the Republic informed7 the Court that on April 10, 2007 former President Gloria Macapagal-Arroyo signed into law R.A. 94228 which expressly repealed Sections 29 and 30 of R.A. 8042 and adopted the policy of close government regulation of the recruitment and deployment of OFWs. R.A. 9422 pertinently provides:

SEC. 1. Section 23, paragraph (b.1) of Republic Act No. 8042, otherwise known as the "Migrant Workers and Overseas Filipinos Act of 1995" is hereby amended to read as follows:

(b.1) Philippine Overseas Employment Administration – The Administration shall regulate private sector participation in the recruitment and overseas placement of workers by setting up a licensing and registration system. It shall also formulate and implement, in coordination with appropriate entities concerned, when necessary, a system for promoting and monitoring the overseas employment of Filipino workers taking into consideration their welfare and the domestic manpower requirements.

48 | P a g e

Page 49: JURISDICTION-DIGESTED-OCHOA.docx

JURISDICTION CASE DIGEST TROPANG POTCHI

In addition to its powers and functions, the administration shall inform migrant workers not only of their rights as workers but also of their rights as human beings, instruct and guide the workers how to assert their rights and provide the available mechanism to redress violation of their rights.

In the recruitment and placement of workers to service the requirements for trained and competent Filipino workers of foreign governments and their instrumentalities, and such other employers as public interests may require, the administration shall deploy only to countries where the Philippines has concluded bilateral labor agreements or arrangements: Provided, That such countries shall guarantee to protect the rights of Filipino migrant workers; and: Provided, further, That such countries shall observe and/or comply with the international laws and standards for migrant workers.

SEC. 2. Section 29 of the same law is hereby repealed.

SEC. 3. Section 30 of the same law is also hereby repealed.

On August 20, 2009 respondents Salac, et al. told the Court in G.R. 152642 that they agree9 with the Republic’s view that the repeal of Sections 29 and 30 of R.A. 8042 renders the issues they raised by their action moot and academic. The Court has no reason to disagree. Consequently, the two cases, G.R. 152642 and 152710, should be dismissed for being moot and academic.

G.R. 167590

(Constitutionality of Sections 6, 7, and 9 of R.A. 8042)

On August 21, 1995 respondent Philippine Association of Service Exporters, Inc. (PASEI) filed a petition for declaratory relief and prohibition with prayer for issuance of TRO and writ of preliminary injunction before the RTC of Manila, seeking to annul Sections 6, 7, and 9 of R.A. 8042 for being unconstitutional. (PASEI also sought to annul a portion of Section 10 but the Court will take up this point later together with a related case.)

Section 6 defines the crime of "illegal recruitment" and enumerates the acts constituting the same. Section 7 provides the penalties for prohibited acts. Thus:

SEC. 6.Definition. – For purposes of this Act, illegal recruitment shall mean any act of canvassing, enlisting, contracting, transporting, utilizing, hiring, procuring workers and includes referring, contract services, promising or advertising for employment abroad, whether for profit or not, when undertaken by a non-license or non-holder of authority contemplated under Article 13(f) of Presidential Decree No. 442, as amended, otherwise known as the Labor Code of the Philippines: Provided, That such non-license or non-holder, who, in any manner, offers or promises for a fee employment abroad to two or more persons shall be deemed so engaged. It shall likewise include the following acts, whether committed by any person, whether a non-licensee, non-holder, licensee or holder of authority:

SEC. 7.Penalties. –

(a) Any person found guilty of illegal recruitment shall suffer the penalty of imprisonment of not less than six (6) years and one (1) day but not more than twelve (12) years and a fine not less than two hundred thousand pesos (P200,000.00) nor more than five hundred thousand pesos (P500,000.00).

(b) The penalty of life imprisonment and a fine of not less than five hundred thousand pesos (P500,000.00) nor more than one million pesos (P1,000,000.00) shall be imposed if illegal recruitment constitutes economic sabotage as defined herein.

Provided, however, That the maximum penalty shall be imposed if the person illegally recruited is less than eighteen (18) years of age or committed by a non-licensee or non-holder of authority.10

Finally, Section 9 of R.A. 8042 allowed the filing of criminal actions arising from "illegal recruitment" before the RTC of the province or city where the offense was committed or where the offended party actually resides at the time of the commission of the offense.

The RTC of Manila declared Section 6 unconstitutional after hearing on the ground that its definition of "illegal recruitment" is vague as it fails to distinguish between licensed and non-licensed recruiters11 and for that reason gives undue advantage to the non-licensed recruiters in violation of the right to equal protection of those that operate with government licenses or authorities.

But "illegal recruitment" as defined in Section 6 is clear and unambiguous and, contrary to the RTC’s finding, actually makes a distinction between licensed and non-licensed recruiters. By its terms, persons who engage in "canvassing, enlisting, contracting, transporting, utilizing, hiring, or procuring workers" without the appropriate government license or authority are guilty of illegal recruitment whether or not they commit the wrongful acts enumerated in that section. On the other hand, recruiters who engage in the canvassing, enlisting, etc. of OFWs, although with the appropriate government license or authority, are guilty of illegal recruitment only if they commit any of the wrongful acts enumerated in Section 6.

The Manila RTC also declared Section 7 unconstitutional on the ground that its sweeping application of the penalties failed to make any distinction as to the seriousness of the act committed for the application of the penalty imposed on such violation. As an example, said the trial court, the mere failure to render a report under Section 6(h) or obstructing the inspection by the Labor Department under Section 6(g) are penalized by imprisonment for six years and one day and a minimum fine of P200,000.00 but which could unreasonably go even as high as life imprisonment if committed by at least three persons.

Apparently, the Manila RTC did not agree that the law can impose such grave penalties upon what it believed were specific acts that were not as condemnable as the others in

the lists. But, in fixing uniform penalties for each of the enumerated acts under Section 6, Congress was within its prerogative to determine what individual acts are equally reprehensible, consistent with the State policy of according full protection to labor, and deserving of the same penalties. It is not within the power of the Court to question the wisdom of this kind of choice. Notably, this legislative policy has been further stressed in July 2010 with the enactment of R.A. 1002212 which increased even more the duration of the penalties of imprisonment and the amounts of fine for the commission of the acts listed under Section 7.

Obviously, in fixing such tough penalties, the law considered the unsettling fact that OFWs must work outside the country’s borders and beyond its immediate protection. The law must, therefore, make an effort to somehow protect them from conscienceless individuals within its jurisdiction who, fueled by greed, are willing to ship them out without clear assurance that their contracted principals would treat such OFWs fairly and humanely.

As the Court held in People v. Ventura,13 the State under its police power "may prescribe such regulations as in its judgment will secure or tend to secure the general welfare of the people, to protect them against the consequence of ignorance and incapacity as well as of deception and fraud." Police power is "that inherent and plenary power of the State which enables it to prohibit all things hurtful to the comfort, safety, and welfare of society."14

The Manila RTC also invalidated Section 9 of R.A. 8042 on the ground that allowing the offended parties to file the criminal case in their place of residence would negate the general rule on venue of criminal cases which is the place where the crime or any of its essential elements were committed. Venue, said the RTC, is jurisdictional in penal laws and, allowing the filing of criminal actions at the place of residence of the offended parties violates their right to due process. Section 9 provides:

SEC. 9.Venue. – A criminal action arising from illegal recruitment as defined herein shall be filed with the Regional Trial Court of the province or city where the offense was committed or where the offended party actually resides at the time of the commission of the offense: Provided, That the court where the criminal action is first filed shall acquire jurisdiction to the exclusion of other courts: Provided, however, That the aforestated provisions shall also apply to those criminal actions that have already been filed in court at the time of the effectivity of this Act.

But there is nothing arbitrary or unconstitutional in Congress fixing an alternative venue for violations of Section 6 of R.A. 8042 that differs from the venue established by the Rules on Criminal Procedure. Indeed, Section 15(a), Rule 110 of the latter Rules allows exceptions provided by laws. Thus:

SEC. 15. Place where action is to be instituted.— (a) Subject to existing laws, the criminal action shall be instituted and tried in the court of the municipality or territory where the

49 | P a g e

Page 50: JURISDICTION-DIGESTED-OCHOA.docx

JURISDICTION CASE DIGEST TROPANG POTCHI

offense was committed or where any of its essential ingredients occurred. (Emphasis supplied)

Section 9 of R.A. 8042, as an exception to the rule on venue of criminal actions is, consistent with that law’s declared policy15 of providing a criminal justice system that protects and serves the best interests of the victims of illegal recruitment.

G.R. 167590, G.R. 182978-79,16 and G.R. 184298-9917

(Constitutionality of Section 10, last sentence of 2nd paragraph)

G.R. 182978-79 and G.R. 184298-99 are consolidated cases. Respondent spouses Simplicio and Mila Cuaresma (the Cuaresmas) filed a claim for death and insurance benefits and damages against petitioners Becmen Service Exporter and Promotion, Inc. (Becmen) and White Falcon Services, Inc. (White Falcon) for the death of their daughter JasminCuaresma while working as staff nurse in Riyadh, Saudi Arabia.

The Labor Arbiter (LA) dismissed the claim on the ground that the Cuaresmas had already received insurance benefits arising from their daughter’s death from the Overseas Workers Welfare Administration (OWWA). The LA also gave due credence to the findings of the Saudi Arabian authorities that Jasmin committed suicide.

On appeal, however, the National Labor Relations Commission (NLRC) found Becmen and White Falcon jointly and severally liable for Jasmin’s death and ordered them to pay the Cuaresmas the amount of US$113,000.00 as actual damages. The NLRC relied on the Cabanatuan City Health Office’s autopsy finding that Jasmin died of criminal violence and rape.

Becmen and White Falcon appealed the NLRC Decision to the Court of Appeals (CA).18 On June 28, 2006 the CA held Becmen and White Falcon jointly and severally liable with their Saudi Arabian employer for actual damages, with Becmen having a right of reimbursement from White Falcon. Becmen and White Falcon appealed the CA Decision to this Court.

On April 7, 2009 the Court found Jasmin’s death not work-related or work-connected since her rape and death did not occur while she was on duty at the hospital or doing acts incidental to her employment. The Court deleted the award of actual damages but ruled that Becmen’s corporate directors and officers are solidarily liable with their company for its failure to investigate the true nature of her death. Becmen and White Falcon abandoned their legal, moral, and social duty to assist the Cuaresmas in obtaining justice for their daughter. Consequently, the Court held the foreign employer Rajab and Silsilah, White Falcon, Becmen, and the latter’s corporate directors and officers jointly and severally liable to the Cuaresmas for: 1) P2,500,000.00 as moral damages; 2) P2,500,000.00 as exemplary damages; 3) attorney’s fees of 10% of the total monetary award; and 4) cost of suit.

On July 16, 2009 the corporate directors and officers of Becmen, namely, EufrocinaGumabay, Elvira Taguiam, Lourdes Bonifacio and Eddie De Guzman (Gumabay, et al.) filed a motion for leave to Intervene. They questioned the constitutionality of the last sentence of the second paragraph of Section 10, R.A. 8042 which holds the corporate directors, officers and partners jointly and solidarily liable with their company for money claims filed by OFWs against their employers and the recruitment firms. On September 9, 2009 the Court allowed the intervention and admitted Gumabay, et al.’s motion for reconsideration.

The key issue that Gumabay, et al. present is whether or not the 2nd paragraph of Section 10, R.A. 8042, which holds the corporate directors, officers, and partners of recruitment and placement agencies jointly and solidarilyliable for money claims and damages that may be adjudged against the latter agencies, is unconstitutional.

In G.R. 167590 (the PASEI case), the Quezon City RTC held as unconstitutional the last sentence of the 2nd paragraph of Section 10 of R.A. 8042. It pointed out that, absent sufficient proof that the corporate officers and directors of the erring company had knowledge of and allowed the illegal recruitment, making them automatically liable would violate their right to due process of law.

The pertinent portion of Section 10 provides:

SEC. 10. Money Claims. – x xx

The liability of the principal/employer and the recruitment/placement agency for any and all claims under this section shall be joint and several. This provision shall be incorporated in the contract for overseas employment and shall be a condition precedent for its approval. The performance bond to be filed by the recruitment/placement agency, as provided by law, shall be answerable for all money claims or damages that may be awarded to the workers. If the recruitment/placement agency is a juridical being, the corporate officers and directors and partners as the case may be, shall themselves be jointly and solidarily liable with the corporation or partnership for the aforesaid claims and damages. (Emphasis supplied)

But the Court has already held, pending adjudication of this case, that the liability of corporate directors and officers is not automatic. To make them jointly and solidarily liable with their company, there must be a finding that they were remiss in directing the affairs of that company, such as sponsoring or tolerating the conduct of illegal activities.19 In the case of Becmen and White Falcon,20 while there is evidence that these companies were at fault in not investigating the cause of Jasmin’s death, there is no mention of any evidence in the case against them that intervenorsGumabay, et al., Becmen’s corporate officers and directors, were personally involved in their company’s particular actions or omissions in Jasmin’s case.

As a final note, R.A. 8042 is a police power measure intended to regulate the recruitment and deployment of OFWs. It aims to curb, if not eliminate, the injustices

and abuses suffered by numerous OFWs seeking to work abroad. The rule is settled that every statute has in its favor the presumption of constitutionality. The Court cannot inquire into the wisdom or expediency of the laws enacted by the Legislative Department. Hence, in the absence of a clear and unmistakable case that the statute is unconstitutional, the Court must uphold its validity.

WHEREFORE, in G.R. 152642 and 152710, the Court DISMISSES the petitions for having become moot and academic.1âwphi1

In G.R. 167590, the Court SETS ASIDE the Decision of the Regional Trial Court ofManila dated December 8, 2004 and DECLARES Sections 6, 7, and 9 of Republic Act 8042 valid and constitutional.

In G.R. 182978-79 and G.R. 184298-99 as well as in G.R. 167590, the Court HOLDS the last sentence of the second paragraph of Section 10 of Republic Act 8042 valid and constitutional. The Court, however, RECONSIDERS and SETS ASIDE the portion of its Decision in G.R. 182978-79 and G.R. 184298-99 that held intervenorsEufrocinaGumabay, Elvira Taguiam, Lourdes Bonifacio, and Eddie De Guzman jointly and solidarily liable with respondent Becmen Services Exporter and Promotion, Inc. to spouses Simplicia and Mila Cuaresma for lack of a finding in those cases that such intervenors had a part in the act or omission imputed to their corporation.

SO ORDERED.

ROBERTO A. ABAD

Associate Justice

Civil Service Commission vs CA

Facts

Respondents Dante G. Guevarra (Guevarra) and Augustus F. Cezar (Cezar) were the Officer-in-Charge/President and the Vice President for Administration, respectively, of the Polytechnic University of the Philippines (PUP)2 in 2005.

On September 27, 2005, petitioner Honesto L. Cueva (Cueva), then PUP Chief Legal Counsel, filed an administrative case against Guevarra and Cezar for gross dishonesty, grave misconduct, falsification of official documents, conduct prejudicial to the best interest of the service, being notoriously undesirable, and for violating Section 4 of Republic Act (R.A.) No. 6713.3 Cueva charged Guevarra with falsification of a public document, specifically the Application for Bond of Accountable Officials and Employees of the Republic of the Philippines, in which the latter denied the existence of his pending criminal and administrative cases. As the head of the school, Guevarra was required to

50 | P a g e

Page 51: JURISDICTION-DIGESTED-OCHOA.docx

JURISDICTION CASE DIGEST TROPANG POTCHI

be bonded in order to be able to engage in financial transactions on behalf of PUP.4 In his Application for Bond of Accountable Officials and Employees of the Republic of the Philippines (General Form No. 58-A), he answered Question No. 11 in this wise:

11. Do you have any criminal or administrative records? – NO. If so, state briefly the nature thereof – NO.5

This was despite the undisputed fact that, at that time, both Guevarra and Cezar admittedly had 17 pending cases for violation of Section 3(e) of R.A. No. 3019 before the Sandiganbayan.6 Cezar, knowing fully well that both he and Guevarra had existing cases before the Sandiganbayan, endorsed and recommended the approval of the application.7

The respondents explained that they believed "criminal or administrative records" to mean final conviction in a criminal or administrative case.8 Thus, because their cases had not yet been decided by the Sandiganbayan, they asserted that Guevarra responded to Question No. 11 in General Form No. 58-A correctly and in good faith.9

On March 24, 2006, the Civil Service Commission (CSC) issued Resolution No. 06052110 formally charging Guevarra with Dishonesty and Cezar with Conduct Prejudicial to the Best Interest of the Service after a prima facie finding that they had committed acts punishable under the Civil Service Law and Rules.

Subsequently, the respondents filed their Motion for Reconsideration and Motion to Declare Absence of Prima Facie Case11 praying that the case be suspended immediately and that the CSC declare a complete absence of a prima facie case against them. Cueva, on the other hand, filed an Urgent Ex-Parte Motion for the Issuance of Preventive Suspension12 and an Omnibus Motion13 seeking the issuance of an order of preventive suspension against Guevarra and Cezar and the inclusion of the following offenses in the formal charge against them: Grave Misconduct, Falsification of Official Document, Conduct Prejudicial to the Best Interest of the Service, Being Notoriously Undesirable, and Violation of Section 4 of R.A. No. 6713.

In Resolution No. 061141, dated June 30, 2006,14 the CSC denied the motion for reconsideration filed by the respondents for being a non-responsive pleading, akin to a motion to dismiss, which was a prohibited pleading under Section 16 of the Uniform Rules on Administrative Cases in the Civil Service Commission.15 It also denied Cueva’s motion to include additional charges against the respondents. The CSC, however, placed Guevarra under preventive suspension for ninety (90) days, believing it to be necessary because, as the officer-in-charge of PUP, he was in a position to unduly influence possible witnesses against him.

Aggrieved, Guevarra and Cezar filed a petition for certiorari and prohibition before the CA essentially questioning the jurisdiction of the CSC over the administrative complaint

filed against them by Cueva. On December 29, 2006, the CA rendered its Decision granting the petition and nullifying and setting aside the questioned resolutions of the CSC for having been rendered without jurisdiction. According to the CA, Section 47, Chapter 7, Subtitle A, Title I, Book V of Executive Order No. 292 (The Administrative Code of 1987), the second paragraph of which states that heads of agencies and instrumentalities "shall have jurisdiction to investigate and decide matters involving disciplinary action against officers and employees under their jurisdiction," bestows upon the Board of Regents the jurisdiction to investigate and decide matters involving disciplinary action against respondents Guevarra and Cezar. In addition, the CA noted that the CSC erred in recognizing the complaint filed by Cueva, reasoning out that the latter should have exhausted all administrative remedies by first bringing his grievances to the attention of the PUP Board of Regents.

ISSUE:Whether or not the Civil Service Commission has original concurrent jurisdiction over administrative cases falling under the jurisdiction of heads of agencies.

RULING:

The petitions are meritorious.

Both CSC and Cueva contend that because the CSC is the central personnel agency of the government, it has been expressly granted by Executive Order (E.O.) No. 292 the authority to assume original jurisdiction over complaints directly filed with it. The CSC explains that under the said law, it has appellate jurisdiction over all administrative disciplinary proceedings and original jurisdiction over complaints against government officials and employees filed before it by private citizens.16 Accordingly, the CSC has concurrent original jurisdiction, together with the PUP Board of Regents, over the administrative case against Guevarra and Cezar and it can take cognizance of a case filed directly with it, despite the fact that the Board of Regents is the disciplining authority of university employees.

Respondents Guevarra and Cezar, on the other hand, fully adopted the position of the CA in its questioned decision and propounded the additional argument that the passage of R.A. No. 8292 has effectively removed from the CSC the authority to hear and decide on cases filed directly with it.

CSC has jurisdiction over cases

filed directly with it, regardless of

who initiated the complaint

The CSC, as the central personnel agency of the government, has the power to appoint and discipline its officials and employees and to hear and decide administrative cases instituted by or brought before it directly or on appeal.17 Section 2(1), Article IX(B) of the 1987 Constitution defines the scope of the civil service:

The civil service embraces all branches, subdivisions, instrumentalities, and agencies of the Government, including government-owned or controlled corporations with original charters.

By virtue of Presidential Decree (P.D.) No. 1341,18 PUP became a chartered state university, thereby making it a government-owned or controlled corporation with an original charter whose employees are part of the Civil Service and are subject to the provisions of E.O. No. 292.19

The parties in these cases do not deny that Guevarra and Cezar are government employees and part of the Civil Service. The controversy, however, stems from the interpretation of the disciplinary jurisdiction of the CSC as specified in Section 47, Chapter 7, Subtitle A, Title I, Book V of E.O. No. 292:

SECTION 47. Disciplinary Jurisdiction.— (1) The Commission shall decide upon appeal all administrative disciplinary cases involving the imposition of a penalty of suspension for more than thirty days, or fine in an amount exceeding thirty days’ salary, demotion in rank or salary or transfer, removal or dismissal from office. A complaint may be filed directly with the Commission by a private citizen against a government official or employee in which case it may hear and decide the case or it may deputize any department or agency or official or group of officials to conduct the investigation. The results of the investigation shall be submitted to the Commission with recommendation as to the penalty to be imposed or other action to be taken.

(2) The Secretaries and heads of agencies and instrumentalities, provinces, cities and municipalities shall have jurisdiction to investigate and decide matters involving disciplinary action against officers and employees under their jurisdiction. Their decisions shall be final in case the penalty imposed is suspension for not more than thirty days or fine in an amount not exceeding thirty days’ salary. In case the decision rendered by a bureau or office head is appealable to the Commission, the same may be initially appealed to the department and finally to the Commission and pending appeal, the same shall be executory except when the penalty is removal, in which case the same shall be executory only after confirmation by the Secretary concerned. [Emphases and underscoring supplied]

While in its assailed decision, the CA conceded that paragraph one of the same provision abovequoted allows the filing of a complaint directly with the CSC, it makes a distinction between a complaint filed by a private citizen and that of an employee under the jurisdiction of the disciplining authority involved. The CA resolved that because Cueva was then the Dean of the College of Law and the Chief Legal Counsel of PUP when he filed the complaint with the CSC, he was under the authority of the PUP Board of

51 | P a g e

Page 52: JURISDICTION-DIGESTED-OCHOA.docx

JURISDICTION CASE DIGEST TROPANG POTCHI

Regents. Thus, it is the Board of Regents which had exclusive jurisdiction over the administrative case he initiated against Guevarra and Cezar.

The Court finds itself unable to sustain the reading of the CA.

The issue is not novel.

The understanding by the CA of Section 47, Chapter 7, Subtitle A, Title I, Book V of E.O. No. 292 which states that "a complaint may be filed directly with the Commission by a private citizen against a government official or employee" is that the CSC can only take cognizance of a case filed directly before it if the complaint was made by a private citizen.

The Court is not unaware of the use of the words "private citizen" in the subject provision and the plain meaning rule of statutory construction which requires that when the law is clear and unambiguous, it must be taken to mean exactly what it says. The Court, however, finds that a simplistic interpretation is not in keeping with the intention of the statute and prevailing jurisprudence. It is a well-established rule that laws should be given a reasonable interpretation so as not to defeat the very purpose for which they were passed. As such, "a literal interpretation is to be rejected if it would be unjust or lead to absurd results."20 In Secretary of Justice v. Koruga,21 the Court emphasized this principle and cautioned us on the overzealous application of the plain meaning rule:

The general rule in construing words and phrases used in a statute is that in the absence of legislative intent to the contrary, they should be given their plain, ordinary, and common usage meaning. However, a literal interpretation of a statute is to be rejected if it will operate unjustly, lead to absurd results, or contract the evident meaning of the statute taken as a whole. After all, statutes should receive a sensible construction, such as will give effect to the legislative intention and so as to avoid an unjust or an absurd conclusion. Indeed, courts are not to give words meanings that would lead to absurd or unreasonable consequences.22

A literal interpretation of E.O. 292 would mean that only private citizens can file a complaint directly with the CSC. For administrative cases instituted by government employees against their fellow public servants, the CSC would only have appellate jurisdiction over those. Such a plain reading of the subject provision of E.O. 202 would effectively divest CSC of its original jurisdiction, albeit shared, provided by law. Moreover, it is clearly unreasonable as it would be tantamount to disenfranchising government employees by removing from them an alternative course of action against erring public officials.

There is no cogent reason to differentiate between a complaint filed by a private citizen and one filed by a member of the civil service, especially in light of Section 12(11), Chapter 3, Subtitle A, Title I, Book V of the same E.O. No. 292 which confers upon the CSC the power to "hear and decide administrative cases instituted by or brought before it directly or on appeal" without any qualification.

In the case of Camacho v. Gloria,23 the Court stated that "under E.O. No. 292, a complaint against a state university official may be filed with either the university’s Board of Regents or directly with the Civil Service Commission."24 It is important to note that the Court did not interpret the Administrative Code as limiting such authority to exclude complaints filed directly with it by a member of the civil service.

Moreover, as early as in the case of Hilario v. Civil Service Commission,25 the Court interpreted Section 47, Chapter 7, Subtitle A, Title I, Book V of E.O. No. 292 as allowing the direct filing with the CSC by a public official of a complaint against a fellow government employee. In the said case, Quezon City Vice-Mayor CharitoPlanas directly filed with the CSC a complaint for usurpation, grave misconduct, being notoriously undesirable, gross insubordination, and conduct prejudicial to the best interest of the service against the City Legal Officer of Quezon City. The CSC issued a resolution ruling that the respondent official should not be allowed to continue holding the position of legal officer. In a petition to the Supreme Court, the official in question asserted that the City Mayor was the only one who could remove him from office directly and not the CSC. The Court upheld the decision of the CSC, citing the same provision of the Administrative Code:

Although respondent Planas is a public official, there is nothing under the law to prevent her from filing a complaint directly with the CSC against petitioner. Thus, when the CSC determined that petitioner was no longer entitled to hold the position of City Legal Officer, it was acting within its authority under the Administrative Code to hear and decide complaints filed before it.26 [Underscoring supplied]

It has been argued that Hilario is not squarely in point.27 While it is true that the circumstances present in the two cases are not identical, a careful reading of Hilario reveals that petitioner therein questioned the authority of the CSC to hear the disciplinary case filed against him, alleging that the CSC’s jurisdiction was only appellate in nature. Hence, the reference to the abovequoted passage in Hilario is very appropriate in this case as respondents herein pose a similar query before us.

It cannot be overemphasized that the identity of the complainant is immaterial to the acquisition of jurisdiction over an administrative case by the CSC. The law is quite clear that the CSC may hear and decide administrative disciplinary cases brought directly before it or it may deputize any department or agency to conduct an investigation.

CSC has concurrent original jurisdiction

with the Board of Regents over

administrative cases

The Uniform Rules on Administrative Cases in the Civil Service28 (the Uniform Rules) explicitly allows the CSC to hear and decide administrative cases directly brought before it:

Section 4.Jurisdiction of the Civil Service Commission. – The Civil Service Commission shall hear and decide administrative cases instituted by, or brought before it, directly or on appeal, including contested appointments, and shall review decisions and actions of its offices and of the agencies attached to it.

Except as otherwise provided by the Constitution or by law, the Civil Service Commission shall have the final authority to pass upon the removal, separation and suspension of all officers and employees in the civil service and upon all matters relating to the conduct, discipline and efficiency of such officers and employees. [Emphases and underscoring supplied]

The CA construed the phrase "the Civil Service Commission shall have the final authority to pass upon the removal, separation and suspension of all officers and employees in the civil service" to mean that the CSC could only step in after the relevant disciplinary authority, in this case the Board of Regents of PUP, had investigated and decided on the charges against the respondents. Regrettably, the CA failed to take into consideration the succeeding section of the same rules which undeniably granted original concurrent jurisdiction to the CSC and belied its suggestion that the CSC could only take cognizance of cases on appeal:

Section 7.Jurisdiction of Heads of Agencies. – Heads of Departments, agencies, provinces, cities, municipalities and other instrumentalities shall have original concurrent jurisdiction, with the Commission, over their respective officers and employees.29 [Emphasis supplied]

It was also argued that although Section 4 of the Uniform Rules is silent as to who can file a complaint directly with the CSC, it cannot be construed to authorize one who is not a private citizen to file a complaint directly with the CSC. This is because a rule issued by a government agency pursuant to its law-making power cannot modify, reduce or enlarge the scope of the law which it seeks to implement.30

Following the earlier disquisition, it can be said that the Uniform Rules does not contradict the Administrative Code. Rather, the former simply provides a reasonable interpretation of the latter. Such action is perfectly within the authority of the CSC, pursuant to Section 12(2), Chapter 3, Subtitle A, Title I, Book V of E.O. No. 292, which gives it the power to "prescribe, amend and enforce rules and regulations for carrying into effect the provisions of the Civil Service Law and other pertinent laws."

Another view has been propounded that the original jurisdiction of the CSC has been further limited by Section 5 of the Uniform Rules, such that the CSC can only take cognizance of complaints filed directly with it which: (1) are brought against personnel of the CSC central office, (2) are against third level officials who are not presidential appointees, (3) are against officials and employees, but are not acted upon by the

52 | P a g e

Page 53: JURISDICTION-DIGESTED-OCHOA.docx

JURISDICTION CASE DIGEST TROPANG POTCHI

agencies themselves, or (4) otherwise require direct or immediate action in the interest of justice:

Section 5.Jurisdiction of the Civil Service Commission Proper. – The Civil Service Commission Proper shall have jurisdiction over the following cases:

A. Disciplinary

1. Decisions of the Civil Service Regional Offices brought before it on petition for review;

2. Decisions of heads of departments, agencies, provinces, cities, municipalities and other instrumentalities, imposing penalties exceeding thirty days suspension or fine in an amount exceeding thirty days salary brought before it on appeal;

3. Complaints brought against Civil Service Commission Proper personnel;

4. Complaints against third level officials who are not presidential appointees;

5. Complaints against Civil Service officials and employees which are not acted upon by the agencies and such other complaints requiring direct or immediate action, in the interest of justice;

6. Requests for transfer of venue of hearing on cases being heard by Civil Service Regional Offices;

7. Appeals from the Order of Preventive Suspension; and

8. Such other actions or requests involving issues arising out of or in connection with the foregoing enumerations.

It is the Court’s position that the Uniform Rules did not supplant the law which provided the CSC with original jurisdiction. While the Uniform Rules may have so provided, the Court invites attention to the cases of Civil Service Commission v. Alfonso31 and Civil Service Commission v. Sojor,32 to be further discussed in the course of this decision, both of which buttressed the pronouncement that the Board of Regents shares its authority to discipline erring school officials and employees with the CSC. It can be presumed that, at the time of their promulgation, the members of this Court, in Alfonso and Sojor, were fully aware of all the existing laws and applicable rules and regulations pertaining to the jurisdiction of the CSC, including the Uniform Rules. In fact, Sojor specifically cited the Uniform Rules in support of its ruling allowing the CSC to take cognizance of an administrative case filed directly with it against the president of a state university. As the Court, in the two cases, did not consider Section 5 of the Uniform Rules as a limitation to the original concurrent jurisdiction of the CSC, it can be stated that Section 5 is merely implementary. It is merely directory and not restrictive of the

CSC’s powers. The CSC itself is of this view as it has vigorously asserted its jurisdiction over this case through this petition.

The case of Alfonso33 is on all fours with the case at bench. The case involved a complaint filed before the CSC against a PUP employee by two employees of the same university. The CA was then faced with the identical issue of whether it was the CSC or the PUP Board of Regents which had jurisdiction over the administrative case filed against the said PUP employee. The CA similarly ruled that the CSC could take cognizance of an administrative case if the decisions of secretaries or heads of agencies, instrumentalities, provinces, cities and municipalities were appealed to it or if a private citizen directly filed with the CSC a complaint against a government official or employee. Because the complainants in the said case were PUP employees and not private citizens, the CA held that the CSC had no jurisdiction to hear the administrative case. It further posited that even assuming the CSC had the authority to do so, immediate resort to the CSC violated the doctrine of exhaustion of administrative remedies as the complaint should have been first lodged with the PUP Board of Regents to allow them the opportunity to decide on the matter. This Court, however, reversed the said decision and declared the following:

Admittedly, the CSC has appellate jurisdiction over disciplinary cases decided by government departments, agencies and instrumentalities. However, a complaint may be filed directly with the CSC, and the Commission has the authority to hear and decide the case, although it may opt to deputize a department or an

agency to conduct the investigation.

We are not unmindful of certain special laws that allow the creation of disciplinary committees and governing bodies in different branches, subdivisions, agencies and instrumentalities of the government to hear and decide administrative complaints against their respective officers and employees. Be that as it may, we cannot interpret the creation of such bodies nor the passage of laws such as – R.A. Nos. 8292 and 4670 allowing for the creation of such disciplinary bodies – as having divested the CSC of its inherent power to supervise and discipline government employees, including those in the academe. To hold otherwise would not only negate the very purpose for which the CSC was established, i.e. to instill professionalism, integrity, and accountability in our civil service, but would also impliedly amend the Constitution itself.

But it is not only for this reason that Alfonso’s argument must fail. Equally significant is the fact that he had already submitted himself to the jurisdiction of the CSC when he filed his counter-affidavit and his motion for reconsideration and requested for a change of venue, not from the CSC to the BOR of PUP, but from the CSC-Central Office to the CSC-NCR. It was only when his motion was denied that he suddenly had a change of heart and raised the question of proper jurisdiction. This cannot be allowed because it would violate the doctrine of res judicata, a legal principle that is applicable to administrative cases as well. At the very least, respondent’s active participation in the

proceedings by seeking affirmative relief before the CSC already bars him from impugning the Commission’s authority under the principle of estoppel by laches.

In this case, the complaint-affidavits were filed by two PUP employees. These complaints were not lodged before the disciplinary tribunal of PUP, but were instead filed before the CSC, with averments detailing respondent’s alleged violation of civil service laws, rules and regulations. After a fact-finding investigation, the Commission found that a prima facie case existed against Alfonso, prompting the Commission to file a formal charge against the latter. Verily, since the complaints were filed directly with the CSC, and the CSC has opted to assume jurisdiction over the complaint, the CSC’s exercise of jurisdiction shall be to the exclusion of other tribunals exercising concurrent jurisdiction. To repeat, it may, however, choose to deputize any department or agency or official or group of officials such as the BOR of PUP to conduct the investigation, or to delegate the investigation to the proper regional office. But the same is merely permissive and not mandatory upon the Commission.34 [Emphases and underscoring supplied]

It has been opined that Alfonso does not apply to the case at bar because respondent therein submitted himself to the jurisdiction of the CSC when he filed his counter-affidavit before it, thereby preventing him from later questioning the jurisdiction of the CSC. Such circumstance is said to be totally absent in this case.35

The records speak otherwise. As in Alfonso, respondents herein submitted themselves to the jurisdiction of the CSC when they filed their Joint Counter-Affidavit.36 It was only when their Motion for Reconsideration and Motion to Declare Absence of Prima Facie Case37 was denied by the CSC that they thought to put in issue the jurisdiction of the CSC before the CA, clearly a desperate attempt to evade prosecution by the CSC. As in Alfonso, respondents are also estopped from questioning the jurisdiction of the CSC.

Based on all of the foregoing, the inescapable conclusion is that the CSC may take cognizance of an administrative case filed directly with it against an official or employee of a chartered state college or university. This is regardless of whether the complainant is a private citizen or a member of the civil service and such original jurisdiction is shared with the Board of Regents of the school.

Gaoiran not applicable

In its decision, the CA relied heavily on Gaoiran v. Alcala38 to support its judgment that it is the Board of Regents, and not the CSC, which has jurisdiction over the administrative complaint filed against the respondents.

A thorough study of the said case, however, reveals that it is irrelevant to the issues discussed in the case at bench. Gaoiran speaks of a complaint filed against a high school teacher of a state-supervised school by another employee of the same school. The complaint was referred to the Legal Affairs Service of the Commission on Higher Education (LAS-CHED). After a fact-finding investigation established the existence of a

53 | P a g e

Page 54: JURISDICTION-DIGESTED-OCHOA.docx

JURISDICTION CASE DIGEST TROPANG POTCHI

prima facie case against the teacher, the Officer-in-Charge of the Office of the Director of LAS-CHED issued a formal charge for Grave Misconduct and Conduct Prejudicial to the Best Interest of the Service, together with the Order of Preventive Suspension. The newly-appointed Director of LAS-CHED, however, dismissed the administrative complaint on the ground that the letter-complaint was not made under oath. Unaware of this previous resolution, the Chairman of the CHED issued another resolution finding petitioner therein guilty of the charges against him and dismissing him from the service. The trial court upheld the resolution of the director of LAS-CHED but on appeal, this was reversed by the CA, affirming the decision of the CHED chairman removing petitioner from service. One of the issues raised therein before this Court was whether the CA erred in disregarding the fact that the complaint was not made under oath as required by the Omnibus Rules Implementing Book V of E.O. 292.

In the said case, the Court concurred with the findings of the CA that it was the formal charge issued by the LAS-CHED which constituted the complaint, and because the same was initiated by the appropriate disciplining authority, it need not be subscribed and sworn to and CHED acquired jurisdiction over the case. The Court further affirmed the authority of the heads of agencies to investigate and decide matters involving disciplinary action against their officers and employees. It bears stressing, at this point, that there is nothing in the case that remotely implies that this Court meant to place upon the Board of Regent exclusive jurisdiction over administrative cases filed against their employees.

In fact, following the ruling in Gaoiran, it can be argued that it was CSC Resolution No. 060521 which formally charged respondents that constituted the complaint, and since the complaint was initiated by the CSC itself as the disciplining authority, the CSC properly acquired jurisdiction over the case.

R.A. No. 8292 is not in conflict

with E.O. No. 292.

In addition, the respondents argue that R.A. No. 8292, which granted to the board of regents or board of trustees disciplinary authority over school employees and officials of chartered state colleges and universities, should prevail over the provisions of E.O. No. 292.39 They anchor their assertion that the Board of Regents has exclusive jurisdiction over administrative cases on Section 4 of R.A. No. 8292,40 to wit:

Section 4.Powers and duties of Governing Boards. – The governing board shall have the following specific powers and duties in addition to its general powers of administration and the exercise of all the powers granted to the board of directors of a corporation under Section 36 of Batas PambansaBlg. 68 otherwise known as the Corporation Code of the Philippines;

(h) to fix and adjust salaries of faculty members and administrative officials and employees subject to the provisions of the revised compensation and classification

system and other pertinent budget and compensation laws governing hours of service, and such other duties and conditions as it may deem proper; to grant them, at its discretion, leaves of absence under such regulations as it may promulgate, any provisions of existing law to the contrary not with standing; and to remove them for cause in accordance with the requirements of due process of law. [Emphasis supplied]

The respondents are mistaken.

Basic is the principle in statutory construction that interpreting and harmonizing laws is the best method of interpretation in order to form a uniform, complete, coherent, and intelligible system of jurisprudence, in accordance with the legal maxim interpretare et concordarelegeslegibusestoptimusinterpretandi modus.41 Simply because a later statute relates to a similar subject matter as that of an earlier statute does not result in an implied repeal of the latter.42

A perusal of the abovequoted provision clearly reveals that the same does not indicate any intention to remove employees and officials of state universities and colleges from the ambit of the CSC. What it merely states is that the governing board of a school has the authority to discipline and remove faculty members and administrative officials and employees for cause. It neither supersedes nor conflicts with E.O. No. 292 which allows the CSC to hear and decide administrative cases filed directly with it or on appeal.

In addition to the previously cited case of Alfonso, the case of The Civil Service Commission v. Sojor43 is likewise instructive. In the said case, this Court ruled that the CSC validly took cognizance of the administrative complaints directly filed with it concerning violations of civil service rules committed by a university president. This Court acknowledged that the board of regents of a state university has the sole power of administration over a university, in accordance with its charter and R.A. No. 8292. With regard to the disciplining and removal of its employees and officials, however, such authority is not exclusive to it because all members of the civil service fall under the jurisdiction of the CSC:

Verily, the BOR of NORSU has the sole power of administration over the university. But this power is not exclusive in the matter of disciplining and removing its employees and officials. Although the BOR of NORSU is given the specific power under R.A. No. 9299 to discipline its employees and officials, there is no showing that such power is exclusive. When the law bestows upon a government body the jurisdiction to hear and decide cases involving specific matters, it is to be presumed that such jurisdiction is exclusive unless it be proved that another body is likewise vested with the same jurisdiction, in which case, both bodies have concurrent jurisdiction over the matter.

All members of the civil service are under the jurisdiction of the CSC, unless otherwise provided by law. Being a non-career civil servant does not remove respondent from the ambit of the CSC.

Career or non-career, a civil service official or employee is within the jurisdiction of the CSC.44 [Emphases and underscoring supplied]

It has been pointed out that the case of Sojor is not applicable to the case at bar because the distinction between a complaint filed by a private citizen and one filed by a government employee was not taken into consideration in the said case.45 The dissent fails to consider that Sojor is cited in the ponencia to support the ruling that R.A. No. 8292 is not in conflict with E.O. No. 292 and to counter respondents’ flawed argument that the passage of R.A. No. 8292 granted the Board of Regents exclusive jurisdiction over administrative cases against school employees and officials of chartered state colleges and universities. Also noteworthy is the fact that the complainants before the CSC in Sojor were faculty members of a state university and were, thus, government employees. Nevertheless, despite this, the Court allowed the CSC to assert jurisdiction over the administrative case, proclaiming that the power of the Board of Regents to discipline its officials and employees is not exclusive but is concurrent with the CSC.46

The case of University of the Philippines v. Regino47 was also cited to bolster the claim that original jurisdiction over disciplinary cases against government officials is vested upon the department secretaries and heads of agencies and instrumentalities, provinces, cities and municipalities, whereas the CSC only enjoys appellate jurisdiction over such cases.48 The interpretation therein of the Administrative Code supposedly renders effectual the provisions of R.A. No. 8292 and does not "deprive the governing body of the power to discipline its own officials and employees and render inutile the legal provisions on disciplinary measures which may be taken by it."49

The Court respectfully disagrees. Regino is obviously inapplicable to this case because there, the school employee had already been found guilty and dismissed by the Board of Regents of the University of the Philippines. Therefore, the issue put forth before this Court was whether the CSC had appellate jurisdiction over cases against university employees, considering the university charter which gives it academic freedom allegedly encompassing institutional autonomy. In contrast, no administrative case was filed before the Board of Regents of PUP because the case was filed directly with the CSC and so, the question here is whether the CSC has original concurrent jurisdiction over disciplinary cases. Rationally, the quoted portions in Regino find no application to the case at bench because those statements were made to uphold the CSC’s appellate jurisdiction which was being contested by petitioner therein. At the risk of being repetitive, it is hereby stressed that the authority of the CSC to hear cases on appeal has already been established in this case. What is in question here is its original jurisdiction over administrative cases.

A different interpretation of the Administrative Code was suggested in order to harmonize the provisions of R.A. No. 8292 and E.O. 292. By allowing only a private citizen to file a complaint directly with the CSC, the CSC maintains its power to review on

54 | P a g e

Page 55: JURISDICTION-DIGESTED-OCHOA.docx

JURISDICTION CASE DIGEST TROPANG POTCHI

appeal decisions of the Board of Regents while at the same time the governing board is not deprived of its power to discipline its officials and employees.50

To begin with, there is no incongruity between R.A. No. 8292 and E.O. No. 292, as previously explained in Sojor. Moreover, the Court fails to see how a complaint filed by a private citizen is any different from one filed by a government employee. If the grant to the CSC of concurrent original jurisdiction over administrative cases filed by private citizens against public officials would not deprive the governing bodies of the power to discipline their own officials and employees and would not be violative of R.A. No. 8292, it is inconceivable that a similar case filed by a government employee would do so. Such a distinction between cases filed by private citizens and those by civil servants is simply illogical and unreasonable. To accede to such a mistaken interpretation of the Administrative Code would be a great disservice to our developing jurisprudence.1âwphi1

It is therefore apparent that despite the enactment of R.A. No. 8292 giving the board of regents or board of trustees of a state school the authority to discipline its employees, the CSC still retains jurisdiction over the school and its employees and has concurrent original jurisdiction, together with the board of regents of a state university, over administrative cases against state university officials and employees.

Finally, with regard to the concern that the CSC may be overwhelmed by the increase in number of cases filed before it which would result from our ruling,51 it behooves us to allay such worries by highlighting two important facts. Firstly, it should be emphasized that the CSC has original concurrent jurisdiction shared with the governing body in question, in this case, the Board of Regents of PUP. This means that if the Board of Regents first takes cognizance of the complaint, then it shall exercise jurisdiction to the exclusion of the CSC.52 Thus, not all administrative cases will fall directly under the CSC. Secondly, Section 47, Chapter 7, Subtitle A, Title I, Book V of the Administrative Code affords the CSC the option of whether to decide the case or to deputize some other department, agency or official to conduct an investigation into the matter, thereby considerably easing the burden placed upon the CSC.

Having thus concluded, the Court sees no need to discuss the other issues raised in the petitions.

WHEREFORE, the petitions are GRANTED. The December 29, 2006 Decision of the Court of Appeals is hereby REVERSED and SET ASIDE. Resolution Nos. 060521 and 061141 dated March 24, 2006 and June 30, 2006, respectively, of the Civil Service Commission are REINSTATED.

THIRD DIVISION

G.R. No. 149588 September 29, 2009

FRANCISCO R. LLAMAS and CARMELITA C. LLAMAS, Petitioners,

vs.

THE HONORABLE COURT OF APPEALS, BRANCH 66 OF THE REGIONAL TRIAL COURT IN MAKATI CITY and THE PEOPLE OF THE PHILIPPINES, Respondents.

D E C I S I O N

NACHURA, J.:

In this petition captioned as "Annulment of Judgment and Certiorari, with Preliminary Injunction," petitioners assail, on the ground of lack of jurisdiction, the trial court’s decision convicting them of "other form of swindling" penalized by Article 316, paragraph 2, of the Revised Penal Code (RPC).

The antecedent facts and proceedings that led to the filing of the instant petition are pertinently narrated as follows:

On August 16, 1984, petitioners were charged before the Regional Trial Court (RTC) of Makati with, as aforesaid, the crime of "other forms of swindling" in the Information,1 docketed as Criminal Case No. 11787, which reads:

That on or about the 20th day of November, 1978, in the municipality of Parañaque, Metro Manila, Philippines and within the jurisdiction of this Honorable Court, the above-named accused, conspiring and confederating together and mutually helping and aiding one another, well knowing that their parcel of land known as Lot No. 11, Block No. 6 of the Subdivision Plan (LRC) Psd 67036, Cadastral Survey of Parañaque, LRC Record No. N-26926, Case No. 4869, situated at Barrio San Dionisio, Municipality of Parañaque, Metro Manila, was mortgaged to the Rural Bank of Imus, did then and there willfully, unlawfully and feloniously sell said property to one Conrado P. Avila, falsely representing the same to be free from all liens and encumbrances whatsoever, and said Conrado P. Avila bought the aforementioned property for the sum of P12,895.00 which was paid to the accused, to the damage and prejudice of said Conrado P. Avila in the aforementioned amount of P12,895.00.

Contrary to law.2

After trial on the merits, the RTC rendered its Decision3 on June 30, 1994, finding petitioners guilty beyond reasonable doubt of the crime charged and sentencing them to suffer the penalty of imprisonment for two months and to pay the fine of P18,085.00 each.

On appeal, the Court of Appeals, in its February 19, 1999 Decision4 in CA-G.R. CR No. 18270, affirmed the decision of the trial court. In its December 22, 1999 Resolution,5 the appellate court further denied petitioners’ motion for reconsideration.

Assailing the aforesaid issuances of the appellate court, petitioners filed before this Court, on February 11, 2000, their petition for review, docketed as G.R. No. 141208.6 The Court, however, on March 13, 2000, denied the same for petitioners’ failure to state the material dates. Since it subsequently denied petitioners’ motion for reconsideration on June 28, 2000,7 the judgment of conviction became final and executory.

With the consequent issuance by the trial court of the April 19, 2001 Warrant of Arrest,8 the police arrested, on April 27, 2001, petitioner Carmelita C. Llamas for her to serve her 2-month jail term. The police, nevertheless, failed to arrest petitioner Francisco R. Llamas because he was nowhere to be found.9

On July 16, 2001, petitioner Francisco moved for the lifting or recall of the warrant of arrest, raising for the first time the issue that the trial court had no jurisdiction over the offense charged.10

There being no action taken by the trial court on the said motion, petitioners instituted, on September 13, 2001, the instant proceedings for the annulment of the trial and the appellate courts’ decisions.

The Court initially dismissed on technical grounds the petition in the September 24, 2001 Resolution,11 but reinstated the same, on motion for reconsideration, in the October 22, 2001 Resolution.12

After a thorough evaluation of petitioners’ arguments vis-à-vis the applicable law and jurisprudence, the Court denies the petition.

In People v. Bitanga,13 the Court explained that the remedy of annulment of judgment cannot be availed of in criminal cases, thus —

Section 1, Rule 47 of the Rules of Court, limits the scope of the remedy of annulment of judgment to the following:

Section 1.Coverage. — This Rule shall govern the annulment by the Court of Appeals of judgments or final orders and resolutions in civil actions of Regional Trial Courts for which the ordinary remedies of new trial, appeal, petition for relief or other appropriate remedies are no longer available through no fault of the petitioner.a1f

The remedy cannot be resorted to when the RTC judgment being questioned was rendered in a criminal case. The 2000 Revised Rules of Criminal Procedure itself does not permit such recourse, for it excluded Rule 47 from the enumeration of the provisions of the 1997 Revised Rules of Civil Procedure which have suppletory application to criminal cases. Section 18, Rule 124 thereof, provides:

Sec. 18.Application of certain rules in civil procedure to criminal cases. – The provisions of Rules 42, 44 to 46 and 48 to 56 relating to procedure in the Court of Appeals and in

55 | P a g e

Page 56: JURISDICTION-DIGESTED-OCHOA.docx

JURISDICTION CASE DIGEST TROPANG POTCHI

the Supreme Court in original and appealed civil cases shall be applied to criminal cases insofar as they are applicable and not inconsistent with the provisions of this Rule.

There is no basis in law or the rules, therefore, to extend the scope of Rule 47 to criminal cases. As we explained in Macalalag v. Ombudsman, when there is no law or rule providing for this remedy, recourse to it cannot be allowed

Here, petitioners are invoking the remedy under Rule 47 to assail a decision in a criminal case. Following Bitanga, this Court cannot allow such recourse, there being no basis in law or in the rules.

In substance, the petition must likewise fail. The trial court which rendered the assailed decision had jurisdiction over the criminal case.

Jurisdiction being a matter of substantive law, the established rule is that the statute in force at the time of the commencement of the action determines the jurisdiction of the court.15 In this case, at the time of the filing of the information, the applicable law was Batas PambansaBilang 129,16 approved on August 14, 1981, which pertinently provides:

Section 20.Jurisdiction in criminal cases. — Regional Trial Courts shall exercise exclusive original jurisdiction in all criminal cases not within the exclusive jurisdiction of any court, tribunal or body, except those now falling under the exclusive and concurrent jurisdiction of the Sandiganbayan which shall hereafter be exclusively taken cognizance of by the latter.

Section 32.Jurisdiction of Metropolitan Trial Courts, Municipal Trial Courts and Municipal Circuit Trial Courts in criminal cases. — Except in cases falling within the exclusive original jurisdiction of Regional Trial Courts and of the Sandiganbayan, the Metropolitan Trial Courts, Municipal Trial Courts, and Municipal Circuit Trial Courts shall exercise:

(1) Exclusive original jurisdiction over all violations of city or municipal ordinances committed within their respective territorial jurisdiction; and

(2) Exclusive original jurisdiction over all offenses punishable with imprisonment of not exceeding four years and two months, or a fine of not more than four thousand pesos, or both such fine and imprisonment, regardless of other imposable accessory or other penalties, including the civil liability arising from such offenses or predicated thereon, irrespective of kind, nature, value, or amount thereof: Provided, however, That in offenses involving damage to property through criminal negligence they shall have exclusive original jurisdiction where the imposable fine does not exceed twenty thousand pesos.

Article 316(2) of the RPC, the provision which penalizes the crime charged in the information, provides that —

Article 316. Other forms of swindling.—The penalty of arresto mayor in its minimum and medium periods and a fine of not less than the value of the damage caused and not more than three times such value, shall be imposed upon:

2. Any person who, knowing that real property is encumbered, shall dispose of the same, although such encumbrance be not recorded.

The penalty for the crime charged in this case is arresto mayor in its minimum and medium periods, which has a duration of 1 month and 1 day to 4 months, and a fine of not less than the value of the damage caused and not more than three times such value. Here, as alleged in the information, the value of the damage caused, or the imposable fine, is P12,895.00. Clearly, from a reading of the information, the jurisdiction over the criminal case was with the RTC and not the Metropolitan Trial Court (MeTC). The MeTC could not have acquired jurisdiction over the criminal action because at the time of the filing of the information, its jurisdiction was limited to offenses punishable with a fine of not more than P4,000.00.17

WHEREFORE, premises considered, the petition is DENIED.

SO ORDERED.

ANTONIO EDUARDO B. NACHURA

Associate Justice

WE CONCUR:

Republic of the Philippines

SUPREME COURT

Manila

SPECIAL THIRD DIVISION

G.R. No. 154243 December 22, 2007

DEPUTY DIRECTOR GENERAL ROBERTO LASTIMOSO, ACTING CHIEF PHILIPPINE NATIONAL POLICE (PNP), DIRECTORATE FOR PERSONNEL AND RECORDS MANAGEMENT (DPRM), INSPECTOR GENERAL, P/CHIEF SUPT. RAMSEY OCAMPO and P/SUPT. ELMER REJANO, petitioners,

vs.

P/SENIOR INSPECTOR JOSE J. ASAYO, respondent.

R E S O L U T I O N

AUSTRIA-MARTINEZ, J.:

Before the Court is respondent’s Motion for Reconsideration of the Decision promulgated on March 6, 2007. In said Decision, the Court granted the petition, holding that the Philippine National Police (PNP) Chief had jurisdiction to take cognizance of the civilian complaint against respondent and that the latter was accorded due process during the summary hearing.

Respondent argues that the decision should be reconsidered for the following reasons:

1. The summary proceeding was null and void because no hearing was conducted; and

2. The evidence presented at the summary hearing does not prove that respondent is guilty of the charges against him.

Respondent insists that the summary hearing officer did not conduct any hearing at all but only relied on the affidavits and pleadings submitted to him, without propounding further questions to complainant's witnesses, or calling in other witnesses such as PO2 Villarama. It should, however, be borne in mind that the fact that there was no full-blown trial before the summary hearing officer does not invalidate said proceedings. In Samalio v. Court of Appeals,1 the Court reiterated the time-honored principle that:

Due process in an administrative context does not require trial-type proceedings similar to those in courts of justice. Where opportunity to be heard either through oral arguments or through pleadings is accorded, there is no denial of procedural due process. A formal or trial-type hearing is not at all times and in all instances essential. The requirements are satisfied where the parties are afforded fair and reasonable opportunity to explain their side of the controversy at hand. The standard of due process that must be met in administrative tribunals allows a certain degree of latitude as long as fairness is not ignored. In other words, it is not legally objectionable for being violative of due process for an administrative agency to resolve a case based solely on position papers, affidavits or documentary evidence submitted by the parties as affidavits of witnesses may take the place of their direct testimony.2 (Emphasis supplied)

The first issue presented by respondent must, therefore, be struck down.

To resolve the second issue, respondent would have the Court re-calibrate the weight of evidence presented before the summary hearing officer, arguing that said evidence is insufficient to prove respondent's guilt of the charges against him.

However, it must be emphasized that the action commenced by respondent before the Regional Trial Court is one for certiorari under Rule 65 of the Rules of Court and as held in People v. Court of Appeals,3 where the issue or question involved affects the wisdom

56 | P a g e

Page 57: JURISDICTION-DIGESTED-OCHOA.docx

JURISDICTION CASE DIGEST TROPANG POTCHI

or legal soundness of the decision – not the jurisdiction of the court to render said decision – the same is beyond the province of a special civil action for certiorari.

Yet, respondent-movant's arguments and the fact that the administrative case against respondent was filed way back in 1997, convinced the Court to suspend the rules of procedure.

The general rule is that the filing of a petition for certiorari does not toll the running of the period to appeal.4

However, Section 1, Rule 1 of the Rules of Court provides that the Rules shall be liberally construed in order to promote their objective of securing a just, speedy and inexpensive disposition of every action and proceeding. In Ginete v. Court of Appeals5 and Sanchez v. Court of Appeals,6 the Court saw it proper to suspend rules of procedure in order to promote substantial justice where matters of life, liberty, honor or property, among other instances, are at stake.

The present case clearly involves the honor of a police officer who has rendered years of service to the country.

In addition, it is also understandable why respondent immediately resorted to the remedy of certiorari instead of pursuing his motion for reconsideration of the PNP Chief’s decision as an appeal before the National Appellate Board (NAB). It was quite easy to get confused as to which body had jurisdiction over his case. The complaint filed against respondent could fall under both Sections 41 and 42 of Republic Act (R.A.) No. 6975 or the Department of the Interior and Local Government Act of 1990. Section 41 states that citizens' complaints should be brought before the People's Law Enforcement Board (PLEB), while Section 42 states that it is the PNP Chief who has authority to immediately remove or dismiss a PNP member who is guilty of conduct unbecoming a police officer.

It was only in Quiambao v. Court of Appeals,7 promulgated in 2005 or after respondent had already filed the petition for certiorari with the trial court, when the Court resolved the issue of which body has jurisdiction over cases that fall under both Sections 41 and 42 of R.A. No. 6975. The Court held that the PLEB and the PNP Chief and regional directors have concurrent jurisdiction over administrative cases filed against members of the PNP which may warrant dismissal from service, but once a complaint is filed with the PNP Chief or regional directors, said authorities shall acquire exclusive original jurisdiction over the case.

With the foregoing peculiar circumstances in this case, respondent should not be deprived of the opportunity to fully ventilate his arguments against the factual findings of the PNP Chief. He may file an appeal before the NAB, pursuant to Section 45, R.A. No. 6925. It is a settled jurisprudence that in administrative proceedings, technical rules of procedure and evidence are not strictly applied.8 In Land Bank of the Philippines v. Celada,9 the Court stressed thus:

After all, technical rules of procedure are not ends in themselves but are primarily devised to help in the proper and expedient dispensation of justice. In appropriate cases, therefore, the rules may be construed liberally in order to meet and advance the cause of substantial justice.10

Thus, the opportunity to pursue an appeal before the NAB should be deemed available to respondent in the higher interest of substantial justice.

WHEREFORE, respondent's Motion for Reconsideration is partly GRANTED. The Decision of the Court dated March 6, 2007 is MODIFIED such that respondent is hereby allowed to file his appeal with the National Appellate Board within ten (10) days from finality of herein Resolution.

SO ORDERED.

GARCIA VS. SANDIGANBAYAN

Facts: This is a petition filed by Clarita Garcia, wife of retired Major Gen. Carlos F. Garcia,with application for injunctive relief in order issued by the Fourth Division of Sandiganbayandenying the motion to quash or dismiss Civil Case No. 0193, a suit for the forfeiturecommenced by the Republic against petitioner and her immediate family.The forfeiture suit was to recover unlawfully acquired funds and properties that theGarcias allegedly acquired and amassed. Then Republic then filed with the Sandiganbaythrough the OMB a petition for forfeiture of those alleged unlawfully acquired properties of theGarcias. The case was docketed as Civil Case 0193(Forfeiture I) and subsequently another caseof forfeiture involving the same parties was filed docketed as Civil Case 0196(Forfeiture II).Thus the two cases were consolidated for convenience and clarity. Before the filing of Forfeiture II but subsequent to the filing of Forfeiture I, the OMB charged the Garciaswithviolation of RA 7080(plunder) and the case raffled to the second division of SB. The plunder charge covered substantially the same properties identified in both Forfeiture I and II.Petitioner now contends, after denying there motion to

dismiss the Forfeiture I case, thatthe the plunder case and the Forfeiture I case should be consolidated in the 2nd division of SB pursuant to RA 8249.On May 20, 2005, the SB 4th Division denied the motion for the reason that the forfeiturecase is not the corresponding civil action for the recovery of civil liability arising from thecriminal case of plunder. Arguments Petitioner: (a) the filing of the plunder case ousted the SB 4thDivision of jurisdiction over theforfeiture case; (b) that the consolidation is imperative in order to avoid possible double jeopardy entanglements.

Issue: Whether or not the Fourth Division of the SB has acquired jurisdiction over the person of petitioner and her three sons considering that, first ,vis-à-vis Civil Case Nos. 0193 (Forfeiture I)and 0196 (Forfeiture II), summons against her have been ineffectively or improperly serve and, second, that the plunder case Crim. Case No. 28107 has already been filed and pendingwith the 2nddivision of the SB.

HELD: The court ruled that the forfeiture cases and plunder cases have different causes of action. Theformer is civil in nature and the latter is criminal. On the matter of double jeopardy the courtheldthat:“Double jeopardy, as a criminal law concept, refers to jeopardy of punishment for thesame offense, suggesting that double jeopardy presupposes two separate criminal prosecutions.Proceedings under RA 1379 are, to repeat, civil in nature. As a necessary corollary, one who issued under RA 1379 may be proceeded against for a criminal offense. Thus, the filing of a caseunder that law is not barred by the conviction or acquittal of the defendant in Crim. Case 28107for plunder.”The court also said that RA 7080(plunder) did not repeal RA 1379(forfeiture) stating that: Nowhere in RA 7080 can we find any provision that would indicate a repeal, expressly or impliedly, of RA 1379. RA 7080 is a penal statute which, at its most basic, aims to penalize theact of any public officer who by himself or in connivance with members of his family amasses,accumulates or acquires ill-gotten wealth in the aggregate amount of at least PhP 50 million. Onthe other hand, RA 1379 is not penal in nature, in that it does not make a crime the act of a public official acquiring during his incumbency an amount of property manifestly out of proportion of his salary and other legitimate income. RA 1379 aims to enforce the right of theState to recover the properties which were not lawfully acquired by the officer.However on the matter on Jurisdiction over the person of Clarita Garcia and his sons the courtsaid that the 4 th

division of SB did not acquire jurisdiction because there was no validsubstituted services of summons made, the SB did not acquire jurisdiction over the persons of petitioner and her children for the reason that there was also no voluntary appearance since theyquestioned the jurisdiction of the 4th division of SB through their motion to dismiss and quashalfor lack of jurisdiction into their person . And perforce, the proceedings in the subject forfeiturecases, insofar as petitioner and her three children are concerned, are null and void for lack of jurisdiction. Thus, the order declaring them in default must be set aside and voided insofar as petitioner and her three children are concerned. For the forfeiture case to proceed against them,it is, thus, imperative for the SB to serve anew summons or alias summons on the petitioner andher three children in order to acquire jurisdiction over their persons. WHEREFORE, the petitions for certiorari and mandamus are PARTIALLY GRANTED. TheSandiganbayan, Fourth Division has not

57 | P a g e

Page 58: JURISDICTION-DIGESTED-OCHOA.docx

JURISDICTION CASE DIGEST TROPANG POTCHI

acquired jurisdiction over petitioner Clarita D. Garciaand her three children. The proceedings in Civil Case Nos. 0193 and 0196 before the Sandiganbayan, Fourth Division, insofar as they pertain to petitioner and her three children, are VOID for lack of jurisdiction over their persons. No costs.

ALEJANDRO VS OFFICE OF THE OMBUDSMAN FACT FINDING AND INTELLIGENCE BUREAU

FACTS:

The Head of the Non-Revenue Water Reduction Department of the Manila Water Services, Inc. (MWSI) received a report that Mico Car Wash (MICO), an establishment owned by Alfredo Rap Alejandro, has been illegally opening an MWSI fire hydrant and using it to operate its car-wash business in Binondo, Manila. Acting on this report, the MWSI, in coordination with the Philippine National Police Criminal Investigation and Detection Group (PNP-CIDG), conducted an anti-water pilferage operation against MICO. During the anti-water pilferage operation, the PNP-CIDG discovered that MICO’s car-wash boys indeed had been illegally getting water from an MWSI fire hydrant. The PNP-CIDG arrested the car-wash boys and confiscated the containers used in getting water. At this point, the petitioner, Alfredo’s father and the Barangay Chairman of Barangay 293, Zone 28, Binondo, Manila (Alejandro), interfered with the PNP-CIDG’s operation by ordering several men to unload the confiscated containers. This intervention caused further commotion and created an opportunity for the apprehended car-wash boys to escape. The Office of the Ombudsman Fact Finding and Intelligence Bureau, after conducting its initial investigation, filed with the Office of the Overall Deputy Ombudsman an administrative complaint against Alejandro for his blatant refusal to recognize a joint legitimate police activity, and for his unwarranted intervention. In its decision, the Office of the Deputy Ombudsman found Alejandro guilty of grave misconduct and ordered his dismissal from the service. The Deputy Ombudsman ruled that Alejandro cannot overextend his authority as Barangay Chairman and induce other people to disrespect proper authorities. The Deputy Ombudsman also added that Alejandro had tolerated the illegal acts of MICO’s car-wash boys. Alejandro filed a motion for reconsideration which the Office of the Deputy Ombudsman denied. Alejandro appealed to the CA via a petition for review under Rule 43 of the Rules of Court. The CA dismissed the petition for premature filing and ruled that Alejandro failed to exhaust proper administrative remedies because he did not appeal the Deputy Ombudsman’s decision to the Ombudsman. Alejandro moved for the reconsideration of the CA ruling, which the CA denied.

ISSUE: Whether the Office of the Ombudsman has jurisdiction over elective officials and has the power to order their dismissal from the service.

RULING:

YES.

The Office of the Ombudsman was created by no less than the Constitution. It is tasked to exercise disciplinary authority over all elective and appointive officials, save only for impeachable officers. While Section 21 of The Ombudsman Act and the Local Government Code both provide for the procedure to discipline elective officials, the seeming conflicts between the two laws have been resolved in cases decided by [the Supreme] Court. In Hagad v. Gozo-Dadole, [the Court] pointed out that “there is nothing in the Local Government Code to indicate that it has repealed, whether expressly or impliedly, the pertinent provisions of the Ombudsman Act. The two statutes on the specific matter in question are not so inconsistent as to compel us to only uphold one and strike down the other.” The two laws may be reconciled by understanding the primary jurisdiction and concurrent jurisdiction of the Office of the Ombudsman. The Ombudsman has primary jurisdiction to investigate any act or omission of a public officer or employee who is under the jurisdiction of the Sandiganbayan. The Sandiganbayan’s jurisdiction extends only to public officials occupying positions corresponding to salary grade 27 and higher. Consequently, as [the Court] held in Office of the Ombudsman v. Rodriguez, any act or omission of a public officer or employee occupying a salary grade lower than 27 is within the concurrent jurisdiction of the Ombudsman and of the regular courts or other investigative agencies. In administrative cases involving the concurrent jurisdiction of two or more disciplining authorities, the body where the complaint is filed first, and which opts to take cognizance of the case, acquires jurisdiction to the exclusion of other tribunals exercising concurrent jurisdiction. In this case, the petitioner is a Barangay Chairman, occupying a position corresponding to salary grade 14. Under RA 7160, the sangguniangpanlungsod or sangguniangbayan has disciplinary authority over any elective barangay official[.] Since the complaint against the petitioner was initially filed with the Office of the Ombudsman, the Ombudsman's exercise of jurisdiction is to the exclusion of the sangguniangbayan whose exercise of jurisdiction is concurrent.

Smart Communications IncvsAldecoaet. Al

GR NO. 166330

SEPTEMBER 11, 2013

Facts:

0 Petitioner is a domestic corporation engaged in the telecommunications business.

0 It entered into a contract of leasewith FlorentinoSebastian whereinAllarilla Construction, immediately constructed and installed a cellular base station.

0 Inside the cellular base station is a communications tower, rising as high as150 feet, with antennas and transmitters; as well as a power house open on three sides containing a 25KVA diesel power generator. Around and close

to the cellular base station are houses, hospitals, clinics, and establishments, including the properties of respondents

0 Respondents filed a Complaint against petitioner for abatement of nuisance and injunction with prayer for temporary restraining order and writ of preliminary injunction alleging that:

0 Susceptible to collapse

0 Emits noxious and deleterious fumes

0 Radiates ultra high frequency (UHF) radio wave emissions

0 Violates the law by constructing the tower without the necessary public hearing, permit of the barangay, as well as that of the municipality, the Environmental Compliance Certificate of the [Department of Environment and Natural Resources (DENR)],construction permit, and other requirements of the National Telecommunications Commission (NTC)

0 Petitioner sought the dismissal of the complaint.

0 The respondents averred that among others that it belies the petitioner’s claim that it sought the consent of the majority of the respondents surrounding the tower site as there where nly a handful of residents signed the document prepared by petitioner and the contents of which were misrepresented by a Sangguniang Bayan Member in the person of Nick Sebastian who is an interested party being the owner of the land where the tower is constructed.

0 The petitioner filed as well a Motion for Summary Judgment which was, later on, granted by RTC.

0 The resident filed a memorandum stating that the test was conducted on November 14 and 15, 2000 and the result shows that the petitioner’s power generator failed the noise emission test, day and night time.

0 RTC ruled in favor of Smart dismissing the complaint as the allegations therein are purely speculative and hence no basis in fact to warrant further proceedings of this case.

0 Appeal to CA which declared the cellular base station of petitioner a nuisance that endangered the health and safety of the residents of Barangay Vira, Roxas, Isabela because: (1) the locational clearance granted to petitioner was a nullity due to the lack of approval by majority of the actual residents of the barangay and a barangay resolution endorsing the

58 | P a g e

Page 59: JURISDICTION-DIGESTED-OCHOA.docx

JURISDICTION CASE DIGEST TROPANG POTCHI

construction of the cellular base station; and (2) the sound emission of the generator at the cellular base station exceeded the Department of Environment and Natural Resources (DENR) standards.

Issue:

0 WON CA erred when it encroached upon an executive function of determining the validity of a locational clearance when it declared, contrary to the administrative findings of the Housing Land Use and Regulatory Board ("HLURB"), that the locational clearance of Petitioner was void.

Held

0 Based on the principle of exhaustion of administrative remedies and its corollary doctrine of primary jurisdiction, it was premature for the Court of Appeals to take cognizance of and rule upon the issue of the validity or nullity of petitioner’s locational clearance for its cellular base station.

0 The general rule is that before a party may seek the intervention of the court, he should first avail of all the means afforded him by administrative processes. The issues which administrative agencies are authorized to decide should not be summarily taken from them and submitted to a court without first giving such administrative agency the opportunity to dispose of the same after due deliberation.

0 Corollary to the doctrine of exhaustion of administrative remedies is the doctrine of primary jurisdiction; that is, courts cannot or will not determine a controversy involving a question which is within the jurisdiction of the administrative tribunal prior to the resolution of that question by the administrative tribunal, where the question demands the exercise of sound administrative discretion requiring the special knowledge, experience and services of the administrative tribunal to determine technical and intricate matters of fact

0 In this case, there is no showing that respondents availed themselves of the afore-mentioned administrative remedies prior to instituting the case before the RTC. While there are accepted exceptions to the principle of exhaustion of administrative remedies and the doctrine of primary jurisdiction,30 respondents never asserted nor argued any of them. Thus, there is no cogent reason for the Court to apply the exceptions instead of the general rule to this case.

Bank of Commerce, petitioner v. Planters Development Bank and BangkoSentralngPilipinas,respondents/BangkoSentralngPilipinas, petitioner v. Planters Development Bank, respondent

Facts: For the 1st set of CB bills, Rizal Commercial Banking Corporation (RCBC) was the registered owner of 7 Central Bank (CB) bills with a total face value of Php 70 million, which were eventually sold to Bank of Commerce (BOC), which, in turn, sold these CB bills to Planters Development Bank (PDB) as evidenced by a “Detached Assignment.” A week later, PDB sold to the BOC Treasury Bills worth Php 70 million as evidenced by a Trading Order and Confirmation of Sale. For the 2nd set of CB bills, RCBC sold 2 CB bills with a total face value of Php 20 million to the PDB and delivered to PDB the corresponding Detached Assignment. PDB delivered to Bancap the 2CB bills which in turn sold the CB bills to Al-Amanah Islamic Investment Bank of the Philippines, which also sold it to the BOC. Upon learning of the transfers involving the CB Bills, the PDB informed the officer-in-charge of the BSP’s Government Securities Department of the PDB’s claim over these CB bills, based on the

Detached Assignments in its possession. The requests of PDB were denied by the officer-in-charge which prompted the petitioner to file an action so as to compel the BSP to determine the party legally entitled to the proceeds of the subject CB bills.

Issue: Whether or not the BangkoSentralngPilipinas has jurisdiction in determining the party legally entitled to the proceeds of the CB bills.

Held: Under the New Central Bank Act (RA 7653), the BSP is given the responsibility of providing policy directions in the areas of money, banking and credit; it is given the primary objective of maintaining price stability, conducive to a balanced and sustainable growth of the economy and of promoting and maintaining monetary stability and convertibility of the peso. Moreover, the Constitution expressly grants the BSP the power of supervision over the operation of banks. While RA 7653 empowers the BSP to conduct administrative hearings and render judgment for or against an entity under its supervisory and regulatory powers, the grant of quasi-judicial authority to the BSP cannot possibly extend to situation which do not call for the exercise by the BSP of its supervisory or regulatory functions over entities within its jurisdiction. The fact alone that the parties involved are banking institutions does not necessarily call for the exercise by the BSP of its quasi-judicial powers under the law.

ADDITION HILLS MANDALUYONG CIVIC & SOCIAL ORGANIZATION, INC. (AHMCSO), PetitionervsMEGAWORLD PROPERTIES & HOLDING, INC., WILFREDO I. IMPERIAL, in his capacity as Director, NCR, and HOUSING AND LANDUSE REGULATORY BOARD (HLRUB), DEPARTMENT OF NATURAL RESOURCES (DENR), Respondents

Facts:Megaworld (Private respondent) was the registered owner of a parcel of land located in Brgy. Addition Hills, MandaluyongCity, covered by a TCT Title issued by the Register of Deeds on which it conceptualized the construction of a residentialcondominium complex called as the “Wack-Wack Heights Condominium”.

Megaworld secured the necessary clearances, licenses and permits for the project, including:1.Certificate of Locational Viability and Development permit issued by HLRUB2.Environmental Compliance Certificate issued by DENR3. Building Permit issued by the office of the Building Official of Mandaluyong City4. Barangay Clearance issued by the Chairman of Barangay Addition Hills, Mandaluyong City.Pending construction, plaintiff filed a complaint before the RTC of Pasig City, Branch 158 for:1. Annulment of the Building Permit, CLV, ECC and Development Permit;2. Prohibit the issuance to Megaworld of Certificate of Registration and License to Sell Condominium units and;3. Permanently enjoin Local and National Building Officials from issuing licenses and permits.Megaworld filed a Motion to Dismiss the complaint for lack of cause of action and that jurisdiction over the case was withHLRUB and not with the regular courts, however the RTC denied the motion, which lead to Megaworld filing its Answer to thecomplaint, and the trial on the merits ensued, rendering decision in favour to the petitioner.Aggrieved with the RTC’s decision, respondent appealed the rendered decision to the Court of Appeals which reversed and setaside such lower courts’Decision finding petitioner’s failure to exhaust administrative remedies before seeking judicial intervention fromcourts. As expected, petitioner filed a motion for reconsideration which however the CA denied. Hence, petitioner filed the instant petition for review on certiorari under Rule 45 to the Supreme Court.

Issue:Whether or Not the CA erred in its finding about the petitioner’s failure to exhaust administrative remedies before seeking judicial intervention?

Held: The court finds the petition without merit. At the outset, the parties discusses issues, although ostensibly legal, actually require the court to make findings of facts. It is long settled by law and jurisprudence that the court is not a trier of facts. Therefore, the only relevant issue to be resolved in this case is whether or not the remedy sought by the petitioner in the trial court is in violation of the legal principle of the exhaustion of administrative remedies. Citing the case of Republic vs. Lacap, the court expounded on the doctrine of exhaustion of administrative remedies and related doctrine of primary jurisdiction in this wise:1. The general rule is that before a party may seek the intervention of the court, he should first avail of all the means afforded him by administrative processes. The issues which administrative agencies are authorized to decide should not be summarily taken from them and submitted to a court without first giving such administrative agency the opportunity to dispose of the same after due deliberation.2. Corollary to the doctrine of exhaustion of administrative remedies is the doctrine of primary jurisdiction; that is, courts cannot or will not determine a controversy involving a question which is within the jurisdiction of the administrative tribunal prior to the resolution of that question by the administrative tribunal, where the question demands the exercise of sound administrative discretion, requiring special knowledge, experience and services of the administrative tribunal to determine technical and intricate matters of fact.3.

59 | P a g e

Page 60: JURISDICTION-DIGESTED-OCHOA.docx

JURISDICTION CASE DIGEST TROPANG POTCHI

Nonetheless, the doctrines enunciated which are based on sound public policy and practical considerations are not inflexible rules. There are many accepted exceptions, such as:A.Where there is estoppel on the part of the party invoking the doctrine;B.Where the challenged administrative act is patently illegal, amounting to lack of jurisdiction;C.Where there is unreasonable delay or official inaction that will irretrievably prejudice the complainant;D.Where the amount involved is relatively small so as to make the rule impractical and oppressive;E.Where the question involved is purely legal and will ultimately have to be decided by the courts of justice;F.Where judicial intervention is urgent;G.When its application may cause great and irreparable damage;H.Where the controverted acts violate due process;I.When the issue of non-exhaustion of administrative remedies has been rendered moot;J.When there is no other plain, speedy and adequate remedy;K.When strong public interest is involved;L.In quo warranto proceedings. In view of the foregoing, the court finds that none of the aforementioned exceptions exist in the case at bar. Hence, the court concludes that the Court of Appeals committed no reversible error in setting aside the trial court decision and dismissing saidcomplaint.WHEREFORE, premises considered, the petition is hereby DENIED and the assailed decision and resolution of the Court ofAppeals are hereby AFFIRMED.

SAAVEDRA VS SEC

FACTS:

1954: Naga Telephone Company (Natelco), Inc. was organized

with P100K authorized capital

1974: Natelco decided to increase its authorized capital to P3,000,000.00

As required by the Public Service Act, Natelco filed an application for the approval

of the increased authorized capital with the then Board of Communications (BOC)

January 8, 1975: approved with conditions:

That the issuance of the shares of stocks will be for a period of one year from the

date hereof, "after which no further issues will be made without previous

authority from this Board."

Natelco filed its Amended Articles of Incorporation with the SEC

the original authorized capital of P100K was already paid

increased capital of P2.9M the subscribers subscribed to P580K of which P145K

was fully paid

capital stock of Natelco was divided into 213K CS and 87K PS, both at a par value

of P10/shares

April 12, 1977: Without no prior authorization from the BOC (now National

Telecommunications Commission) (NTC), Natelco entered into a contract with

Communication Services, Inc. (CSI) for the "manufacture, supply, delivery and

installation" of telephone equipment

Natelco issued 24K shares of CS to CSI as downpayment

May 5, 1979: issued another 12K shares of CS to CSI

May 19, 1979: annual stockholders' meeting to elect their 7 directors to their BOD

for the year 1979-1980

Pedro Lopez Dee (Dee) was unseated as Chairman of the Board and President but

was elected as one of the directors, together with his wife, Amelia Lopez Dee

CSI was able to gain control when their legal counsel, Atty. Luciano Maggay

(Maggay) won a seat in the Board

Atty. Maggay became president upon reorganization

Among the directors: Mr. Justino de Jesus, Sr., Mr. Pedro Lopez Dee and Mrs

Amelia C. Lopez Dee never attended the Maggay Board thereby only Maggay

representatives and Atty. Maggay attended

as per contract they issued 113,800 shares of stock in favor of CSI

Dee having been unseated filed a petition in the SEC questioning the validity of the

elections

ground: no valid list of stockholders through which the right to vote could be

determined

As prayed for a restraining order was issued by the SEC placing officers of the

1978-1979 Natelco Board in hold-over capacity

Upon elevation to the SC: dismissed the petition for being premature; restraining

order was restrained

resulted in the unseating of the Maggay group from the BOD in a "hold-over"

capacity

SEC: ordering the holding of special stockholder' meeting to elect the new

members of the BOD based on its findings of who are entitled to vote

June 23, 1981: Dee filed a petition for certiorari/appeal with the SEC en banc

SEC en banc: dismissed for lack of merit

May 20, 1982: Antonio Villasenor filed w/ the CFI claiming that he was an assignee

of an option to repurchase 36K shares of CS of Natelco under a Deed of

Assignment executed in his favor

May 21, 1982: restraining order dwas issued by the lower court commanding

desistance from the scheduled election until further orders

May 22, 1982: controlling majority of the stockholders proceeded with the

elections under the supervision of the SEC representatives

May 25, 1982: SEC recognized the election and the duly elected directors

Lopez Dee group headed by Messrs. Justino De Jesus and Julio Lopez Dee kept

insisting no elections were held and refused to vacate their positions

May 28, 1982: SEC issued another order directing the hold-over directors and

officers to turn over their respective posts and directing the Sheriff of Naga City

and other enforcement agencies to enforce its order

May 29, 1982: hold-over officers peacefully vacated

June 2, 1982: Villasenor filed a charge for contempt

September 7, 1982: lower court rendered CSI Nilda Ramos, Luciano Maggay,

Desiderio Saavedra, Augusto Federis and Ernesto Miguel, guilty of contempt of

court

September 17, 1982: CSI group filed a petition for certiorari and prohibition with

preliminary injunction or restraining order against the CFI

April 14, 1983: IAC: Annuling contempt charge

ISSUES:

60 | P a g e

Page 61: JURISDICTION-DIGESTED-OCHOA.docx

JURISDICTION CASE DIGEST TROPANG POTCHI

1. W/N SEC has the power and jurisdiction to declare null and void shares of stock

issued by NATELCO to CSI for violation of Sec. 20 (h) of the Public Service Act -

NO

2. W/N Natelco stockholders have a right of preemption to the 113,800 shares

3. W/N the May 22, 1982 election was valid

HELD: Dismissed for lack of merit

1. NO

The jurisdiction of the SEC is limited to matters intrinsically connected with the

regulation of corporations, partnerships and associations and those dealing with

internal affairs of such entities; P.D. 902-A does not confer jurisdiction to SEC over

all matters affecting corporations

The jurisdiction of the SEC is limited to deciding the controversy in the election of

the directors and officers of Natelco

The SEC is empowered by P.D. 902-A to decide intra-corporate controversies and

that is precisely the only issue in this case.

2. NO

There is distinction between:

an order to issue shares on or before May 19, 1979; and

actual issuance of the shares after May 19, 1979 - CSI was in control of voting

shares and the Board

The power to issue shares of stocks in a corporation is lodged in the board of

directors and nostockholders meeting is required to consider it because additional

issuance of shares of stocks does not need approval of the stockholders - no

violation of preemptive right

3. YES.

Clear from records that it was held

within the jurisdiction of the lower court as it does not involve an intra-corporate

matter but merely a claim of a private party of the right to repurchase

common shares of stock of Natelco and that the restraining order was not meant

to stop the election duly called for by the SEC and a matter purely within the

exclusive jurisdiction of the SEC

temporary restraining order amounted to an injunctive relief against the SEC

since the trial judge in the lower court did not have jurisdiction in issuing the

questioned restraining order, disobedience thereto did not constitute contempt

BLUE BAR COCONUT PHILIPPINES VS THE HONORABLE FRANCISCO S. TANTUICO

Facts: The President issued Presidential Decree No. 276 establishing a coconut stabilization fund. Under this decree, the Philippine Coconut Authority, in addition to its powers granted under Presidential Decree No. 232, was authorized to formulate and immediately implement a stabilization scheme for coconut-based consumer goods,

Rules and Regulations governing the collection and disposition of the Coconut Consumers Stabilization Fund (CCSF) promulgated by the Coconut Consumer Stabilization Committee provides that the collection of levy in every first sale of copra resecada or its equivalent in terms of whole nuts shall take effect on August 10, 1973. The petitioners are all end-users and as such, are levy-collectors and remitters.

The respondent Acting Chairman of the Commission on Audit initiated a special audit of coconut end-user companies, which include herein petitioners, with respect to their Coconut Consumers Stabilization Fund levy collections and the subsidies they had received. As a result of the initial findings of the Performance Audit Office with respect only to the petitioners, respondent Acting COA Chairman directed the Chairman, the Administrator, and the Military Supervisor of PCA and the Manager of the Coconut Consumers Stabilization Fund, in various letters to them (Annexes G-2 H, I, J, L and N of petition) to collect the short levies and overpaid subsidies, and to apply subsidy claims to the settlement of short levies should the petitioners fail to remit the amount due.

The remaining issues all revolve on the question—After the Philippine Coconut Authority—the authority vested by law to implement the stabilization scheme for the coconut industry under P.D. 276, which includes the collection of the levy to support the Stabilization Fund—had acted, can the Commission on Audit say that the rules and decisions of the PCA are erroneous and nullify them, to the prejudice of petitioners who obediently complied with said rules and decisions?"

Issue: Whether or not the respondent COA Chairman was correct in disregarding the two resolutions of the PCA Governing Board for being ultra vires is the main issue in this petition. This issue became academic when the then President of the Philippines informed the Solicitor General that the Governing Board of the PCA would continue to function until the formal organization of the new Governing Board. Following this ruling, the respondent COA Chairman reconsidered his earlier stand and allowed the

petitioners to get their subsidy claims which he had earlier refused. In effect, the respondent COA Chairman eventually acknowledged the validity of the two questioned PCA resolutions.

The petitioners also question the respondents' authority to audit them. They contend that they are outside the ambit of respondents' "audit" power which is confined to government-owned or controlled corporations.

Held: his argument has no merit. Section 2 (1) of Article IX-D of the Constitution provides that "The Commission on Audit shall have the power, authority and duty to examine, audit, and settle all accounts pertaining to the revenues and receipts of, and expenditures or uses of funds and property, owned or held in trust by or pertaining to, the Government, or any of its subdivisions, agencies or instrumentalities, including government-owned or controlled corporation with original charters, and on a post-audit basis. ... (d) such non-governmental entities receiving subsidy or equity directly or indirectly from or through the Government which are required by law or the granting institution to submit to such audit as a condition of subsidy or equity." (Emphasis supplied) The Constitution formally embodies the long established rule that private entities who handle government funds or subsidies in trust may be examined or audited in their handling of said funds by government auditors.

doctrine of primary jurisdiction ... the courts cannot or will not determine a controversy involving a question which is within the jurisdiction of an administrative tribunal prior to the decision of that question by the administrative tribunal, where the question demands the exercise of sound administrative discretion requiring the special knowledge, experience, and services of the administrative tribunal to determine technical and intricate matters of fact, and a uniformity of ruling is essential to comply with the Purposes of the regulatory statute administered."

In the case at bar, the petitioners have not shown through the laying down of concrete factual foundations that the respondents' questioned acts were done with grave abuse of discretion amounting to lack of jurisdiction.

GARCIA VS EXECUTIVE SECRETARY

Facts: The Supreme Court has dismissed the petition of former Armed Forces of the Philippines (AFP) comptroller Major General Garlos F. Garcia that sought to annul the September 9, 2011 Confirmation of Sentence by the Office of the President (OP). The sentence handed down by the Special General Court Martial No. 2 had ordered his dishonorable discharge from service, forfeiture of all his pay and allowances, and confinement for two years in a penitentiary. On September 16, 2011, or a week after the OP confirmed the sentence of the court martial against him, Garcia was arrested and detained and continues to be detained at the maximum security compound of the National Penitentiary in Muntinlupa. Garcia, tried by the Special General Court Martial NR 2, was charged with and convicted of violation of the 96th Article of War (Conduct Unbecoming an Officer and Gentleman) and violation of the 97th Article of War

61 | P a g e

Page 62: JURISDICTION-DIGESTED-OCHOA.docx

JURISDICTION CASE DIGEST TROPANG POTCHI

(Conduct Prejudicial to Good Order and Military Discipline) for failing to disclose all his assets in his Sworn Statement of Assets and Liabilities and Networth for the year 2003 as required by RA 3019, as amended in relation to RA 6713.

Issue: Whether or not the office of the President commit grave abuse of discretion

Held: The court held that the OP did not commit any grave abuse of discretion in issuing the Confirmation of Sentence.The Court upheld the authority of the President, as Commander-in-Chief, to confirm the sentence. It held that the General Court Martial had jurisdiction over the case since it was indisputable that Garcia was an officer in the active service of AFP when he committed the violations until his arraignment. Garcia’s mandatory retirement on November 18, 2004 did not divest the General Court Martial of its jurisdiction. And since the General Court Martial has jurisdiction, the Court held that the President, as Commander-in-Chief, also acquired jurisdiction as mandated under Article 47 of the Articles of War.

The Court stressed that Article 48 of the Articles of War vests on the President, as Commander-in-Chief, the power to approve or disapprove the entire or any part of the sentence given by the court martial, while Article 49 of the same grants the President the power to mitigate or remit a sentence.

“Thus, the power of the President to confirm, mitigate and remit a sentence of erring military personnel is a clear recognition of the superiority of civilian authority over the military. However, although the law (Articles of War) which conferred those powers to the President is silent as to the deduction of the period of preventive confinement to the penalty imposed, as discussed earlier, such is also the right of an accused provided for by Article 29 of the RPC,” held the Court.

OPTIMA REALTY CORP. VS. HERTZ PHIL. EXCLUSIVE CARS, INC.

FACTS: Optima is engaged in the business of leasing and renting out commercial spaces and buildings to its tenants. Sometime in 2002,Optima and Hertz entered into a Contract of Lease with over a 131-square-meter office unit and a parking lot in the Optima Building for a period of three years commencingand ending. The parties amended their lease agreement by shortening the lease period to two years and five months.

Renovations in the Optima Building commenced in January and ended. As a result, Hertz alleged that it experienced a 50% drop in monthly sales and a significant decrease in its personnel’s productivity. It then requested a 50% discount on its rent for the months of May, June, July and August 2005.

Optima granted the request of Hertz.However, the latter still failed to pay its rentals. In addition, Hertz likewise failed to pay its utility bills

Hertz filed a Complaint against Optima, what optima did was he wrote a letter through his counsel that Herzt should vacate the building, but Hertz refused to do so. Accordingly Optima was constrained to file charges before MeTC. MeTC ruled in favor of the Optima thus Hertz appealed in RTC but also in favor of Optima. Not satisfied with the decision of the appellate courts Hertz filed an appeal before Supreme Court, contending that MeTC has no jurisdiction over the case at bar due to improper service of summon.

ISSUE:Whether the MeTC properly acquired jurisdiction over the person of respondent Hertz;

HELD: Yes.

The MeTC acquired jurisdiction over the person of respondent Hertz.

In civil cases, jurisdiction over the person of the defendant may be acquired either by service of summons or by the defendant’s voluntary appearance in court and submission to its authority.In this case, the MeTC acquired jurisdiction over the person of respondent Hertz by reason of the latter’s voluntary appearance in court.

In this case, the records show that the following statement appeared in respondent’s Motion for Leave to File Answer:

In spite of the defective service of summons, the defendant opted to file the instant Answer with Counterclaim with Leave of Court, upon inquiring from the office of the clerk of court of this Honorable Court and due to its notice of hearing on March 29, 2005 application for TRO/Preliminary Mandatory Injunction was received on March 26, 2006.

Furthermore, the Answer with Counterclaim filed by Hertz never raised the defense of improper service of summons. The defenses that it pleaded were limited to litispendentia, pari delicto, performance of its obligations and lack of cause of action.38 Finally, it even asserted its own counterclaim against Optima.

Measured against the standards in Philippine Commercial International Bank, these actions lead to no other conclusion than that Hertz voluntarily appeared before the court a quo. We therefore rule that, by virtue of the voluntary appearance of respondent Hertz before the MeTC, the trial court acquired jurisdiction over respondent’s.

ELICE AGRO-INDUSTIRAL CORP. VS. YOUNG

FACTS:The Respondents and Ellice Agro-Industrial Corporation (EAIC), represented by its alleged corporate secretary and attorney-in-fact, Guia G. Domingo (Domingo), entered into a Contract to Sell, under certain terms and conditions, wherein EAIC agreed to sell

to the respondents a 30,000 square-meter portion of a parcel of land located in Lutucan, Quezon and registered under EAIC’s name in consideration P1,050,000.00 Pesos.

Pursuant to the Contract to Sell, respondents paid EAIC, through Domingo, the aggregate amount of P545,000.00 Pesos as partial payment for the acquisition of the subject property. Despite such payment, EAIC failed to deliver to respondents the owner’s duplicate certificate of title of the subject property and the corresponding deed of sale as required under the Contract to Sell.

Prompted by the failure of EAIC to comply with its obligation, respondents had their Affidavit of Adverse Claim annotated in TCT No. T-157038.

Thus, Respondents filed a Complaintbefore the RTC.

The initial attempt to serve the summons and a copy of the complaint and its annexes on EAIC, through Domingo, on Rizal Street, Sariaya, Quezon, was unsuccessful as EAIC could not be located in the said address.

Another attempt was made to serve the alias summons on EAIC at 996 Maligaya Street, Singalong, Manila, the residence of Domingo. The second attempt to serve the alias summons to Domingo was, this time, successful.

On the scheduled pre-trial conference, neither Domingo nor her counsel appeared. As a result of EAIC’s failure to appear in the pre-trial conference, respondents were allowed to present their evidence ex parte.

EAIC, represented by Gala, filed its Petition for Relief from JudgmentRTC Decision before the same court. The petition for relief from judgment was premised on the alleged fraud committed by Domingo in concealing the existence of both the Contract to Sellfrom EAIC.

The RTC denied the petition for relief from judgment for being clearly filed out of time under Section 3, Rule 38 of the Rules of Court.

Thus filed before the CA, but unfortunately CA dismisses the petition because it had already been rejected with finality, EAIC could not be permitted to invoke the same ground in a petition for annulment of judgment.

ISSUE:Whether or not the RTC validly acquired jurisdiction over the person of EAIC.

HELD: NO.Service of summons upon a private domestic corporation, to be effective and valid, should be made on the party involved in the said case. Conversely, service of summons on anyone other than the president, manager, secretary, cashier, agent, or director, is not valid. The purpose is to render it reasonably certain that the corporation will receive prompt and proper notice in an action against it or to insure that the summons be served on a representative so integrated with the corporation that such person will know what to do with the legal papers served on him.

62 | P a g e

Page 63: JURISDICTION-DIGESTED-OCHOA.docx

JURISDICTION CASE DIGEST TROPANG POTCHI

In the present case,the pertinent document showing EAIC’s composition at the time the summons was served upon it, through Domingo, will readily reveal that she was not its president, manager, secretary, cashier, agent or director. Due to this fact, the Court is of the view that her honest belief that she was the authorized corporate secretary was clearly mistaken because she was evidently not the corporate secretary she claimed to be. In view of Domingo’s lack of authority to properly represent EAIC, the Court is constrained to rule that there was no valid service of summons binding on it.

In addition, at the time she filed the Answer with Counterclaim, Domingo was clearly not an officer of EAIC, much less duly authorized by any board resolution or secretary’s certificate from EAIC to file the said Answer with Counterclaim in behalf of EAIC. Undoubtedly, Domingo lacked the necessary authority to bind EAIC to Civil Case No. 96-177 before the RTC despite the filing of an Answer with Counterclaim. EAIC cannot be bound or deemed to have voluntarily appeared before the RTC by the act of an unauthorized stranger.

It is a settled rule that jurisdiction over the defendant is acquired either upon a valid service of summons or the defendant’s voluntary appearance in court. When the defendant does not voluntarily submit to the court’s jurisdiction or when there is no valid service of summons, any judgment of the court which has no jurisdiction over the person of the defendant is null and void.27 The purpose of summons is not only to acquire jurisdiction over the person of the defendant, but also to give notice to the defendant that an action has been commenced against it and to afford it an opportunity to be heard on the claim made against it. The requirements of the rule on summons must be strictly followed, otherwise, the trial court will not acquire jurisdiction over the defendant.

AFADAL VS. CARLOS

FACTS:Respondent Romeo Carlos filed a complaint for unlawful detainer and damages against petitioners, ZenaidaGuijabar (Guijabar), John Doe, Peter Doe, Juana Doe, and all persons claiming rights under them before the Municipal Trial Court, Biñan, Laguna (MTC). Respondent alleged that petitioners, Guijabar, and all other persons claiming rights under them were occupying, by mere tolerance, a parcel of land in respondent’s name covered by Transfer Certificate of Title No. T-530139 in the Registry of Deeds Calamba, Laguna. Respondent claimed that petitioner AbubakarAfdal (petitioner Abubakar) sold the property to him but that he allowed petitioners to stay in the property. Respondent demanded that petitioners, Guijabar, and all persons claiming rights under them turn over the property to him because he needed the property for his personal use. Respondent further alleged that petitioners refused to heed his demand and he was constrained to file a complaint before the LuponngTagapamayapa (Lupon). According to respondent, petitioners ignored the notices and the Lupon issued a "certificate to file action." Then, respondent filed the complaint before the MTC.

According to the records, there were three attempts to serve the summons and complaint on petitioners – 14 January, 3 and 18 February 2004. However, petitioners failed to file an answer.

Respondent filed an ex-parte motion and compliance with position paper submitting the case for decision based on the pleadings on record.

In its Decision, the MTC ruled in favor of respondent. Petitioners filed a petition for relief from judgment with the MTC. Respondent filed a motion to dismiss or strike out the petition for relief. Subsequently, petitioners manifested their intention to withdraw the petition for relief after realizing that it was a prohibited pleading under the Revised Rule on Summary Procedure. the MTC granted petitioners’ request to withdraw the petition for relief. But instead they file the relief before the RTC.

RTC issued the assailed Order dismissing the petition for relief. The RTC said it had no jurisdiction over the petition because the petition should have been filed before the MTC in accordance with Section 1 of Rule 38 of the Rules of Court which provides that a petition for relief should be filed "in such court and in the same case praying that the judgment, order or proceeding be set aside."

ISSUE: Whether or not the RTC erred in dismissing the case for the reason that it is not within their jurisdiction

HELD: Yes.In the present case, petitioners cannot file the petition for relief with the MTC because it is a prohibited pleading in an unlawful detainer case. Petitioners cannot also file the petition for relief with the RTC because the RTC has no jurisdiction to entertain petitions for relief from judgments of the MTC. Therefore, the RTC did not err in dismissing the petition for relief from judgment of the MTC.

The remedy of petitioners in such a situation is to file a petition for certiorari with the RTC under Rule 65 of the Rules of Court on the ground of lack of jurisdiction of the MTC over the person of petitioners in view of the absence of summons to petitioners. Here, we shall treat petitioners’ petition for relief from judgment as a petition for certiorari before the RTC.

RAPID CITY REALTY AND DEVELOPMENT CORP. VS. SPOUSES VILLA

FACTS: Sometime in 2004, Rapid City Realty and Development Corporation (petitioner) filed a complaint for declaration of nullity of subdivision plans, mandamus and damages against several defendants including Spouses Orlando and Lourdes Villa (respondents). The complaint, which was docketed at the Regional Trial Court of Antipolo City, was lodged at Branch 71 thereof.

After one failed attempt at personal service of summons, Gregorio Zapanta (Zapanta),

court process server, resorted to substituted service by serving summons upon

respondents’ househelp who did not acknowledge receipt thereof and refused to

divulge their names.

Despite substituted service, respondents failed to file their Answer, prompting petitioner to file a “Motion to Declare Defendants[-herein respondents] in Default” which the trial court granted.

Eight months thereafter, respondents filed a Motion to Lift Order of Default. Claiming that on January 27, 2006 they “officially received all pertinent papers such as Complaint and Annexes.

The trial court set aside the Order of Default and gave herein respondents five

days to file their Answer. Respondents just the same did not file an Answer, drawing

petitioner to again file a Motion to declare them in default, which the trial

courtagain granted.

And then again, respondents filed an Omnibus Motion for reconsideration of the second

order declaring them in default and to vacate proceedings, this time claiming that the

trial court did not acquire jurisdiction over their persons due to invalid service of

summons.

ISSUE: WON the trial court has jurisdiction over the case at bar

HELD: Yes.It is settled that if there is no valid service of summons, the court can still

acquire jurisdiction over the person of the defendant by virtue of the latter’s voluntary

appearance. Thus Section 20 of Rule 14 of the Rules of Court provides:

Sec. 20. Voluntary appearance. – The defendant’s voluntary appearance in the action shall be equivalent to service of summons. The inclusion in a motion to dismiss of other grounds aside from lack of jurisdiction over the person shall not be deemed a voluntary appearance.

Respondents did not, in said motion, allege that their filing thereof was a special

appearance for the purpose only to question the jurisdiction over their persons. Clearly,

they had acquiesced to the jurisdiction of the court.

63 | P a g e

Page 64: JURISDICTION-DIGESTED-OCHOA.docx

JURISDICTION CASE DIGEST TROPANG POTCHI

Supreme Court ruling:

WHEREFORE, the petition is GRANTED. The assailed Court of Appeals Decision of April

29, 2008 is REVERSED and SET ASIDE.

PHILIPPINE NATIONAL BANK VS. COURT OF APPEALS

FACTS:EpifanioMatienzo and FlorenciaMatienzo were the original owners of a 4,161 sq. meter-lot registered situated at Cabinitan, Virac, Catanduanes.

The business partners, Domingo, Orlando and Ireneo, all surnamed Molina, were looking for a lot in Virac where they could put up a furniture shop and a 'chicharon' factory. Respondents Matienzos offered to sell their lot.

The Molinas wanted to buy only one-half of the property. An affidavit was executed by the parties pertaining the sale of the said property

Orlando Molina issued a promissory note for P20,800, payable in installments as the price of the 2,080-sq.-meter portion of Matienzo's land. Both the affidavit and the promissory note were signed by Orlando Molina.

However, when the Deed of Absolute Sale was presented to Matienzo for signing, Domingo Molina appeared as vendee, instead of Orlando. The spouses wondered why that was so, but they were assured that there would be no problem because Domingo Molina was one of the partners to the proposed business venture. Convinced of the group's sincerity, the respondent spouses signed the Deed of Absolute Sale. Thus, the sale was proceeded. The defendants were able to pay Matienzo only P3,350 out of the P20,000 price of the sale of one-half of their property. Despite repeated demands for payment, the Molinasdefaulted, they were also defaulted in paying the PNB loan.

Tatienzosthen filed an action for Reconveyance of Title, Recovery of Ownership and Possession, Annulment of the Document and Damages.

The RTC dismissed the case thus it was appealed before the CA. However, CA reversed the decision of the RTC. As a result the PND filed a petition for certiorari before the Supreme Court asserting that CA has no jurisdiction to render the judgment against it.

ISSUE:Whether or not CA has jurisdiction in rendering judgment over the case

HELD: No.Since the PNB was not a party in the suit (only its manager was sued and then dropped from the complaint) both the trial court and the Court of Appeals did not

acquire jurisdiction over said Bank. The Court of Appeals' decision ordering the Bank to reconvey to Matienzo one-half of the land which it had purchased at the foreclosure sale, was therefore null and void for lack of jurisdiction

Mere service of the appellants' brief on PNB did not operate to bring the Bank into the case. Jurisdiction over a person is acquired by service of summons and copy of the complaint on him (Rule 14, Rules of Court). PNB could not be expected to answer the appellants' brief not only because it was not a party in the appeal (nor in the trial court) but also because the dismissal of the complaint against the PNB manager in Virac was not one of the errors argued in the appellants' brief.

The Province of Aklan v. Jody King Construction and Development Corp.

Facts: The Province of Aklan (“Aklan”) and Jody King Construction and Development Corp. (“JKCDC”) entered into a contract for the design and construction of the Caticlan Jetty Port and Terminal (Phase I) in Malay, Aklan. In the course of construction, Aklan issued variation/change orders for additional works, whichagreed upon by the parties.

Aklan entered into a negotiated contract with JKCDC for the construction of Passenger Terminal Building (Phase II) also at Caticlan Jetty Port in Malay, Aklan. JKCDC made a demand for the total amount of P22,419,112.96 covering the items which Aklan allegedly failed to settle. JKCDC then filed a civil case with the RTC of Marikina City (“RTC”) against Aklan for the collection of said amount. The RTC issued a writ of preliminary attachment against Aklan. The RTC later ruled in favor of JKCDC (“RTC Decision”). Since Aklan’s motion for reconsideration of the RTC Decision was filed out of time, a writ of execution was later issued. The sheriff served notices of garnishment on Land Bank of the Philippines, Philippine National Bank and Development Bank of the Philippines at their branches in Kalibo, Aklan for the satisfaction of the judgment debt from the funds deposited under Aklan’s account. Said banks, however, refused to give due course to the court order, citing the relevant provisions of statutes, circulars and jurisprudence on the determination of government monetary liabilities, their enforcement and satisfaction.Aklan’s Notice of Appeal was denied by the RTC (“RTC Order”). Aklan moved for reconsideration of the RTC Order, which was also denied by the RTC.Aklan filed a Petition for Certiorari with the Court of Appeals (“CA”) to assail the writ of execution. The CA dismissed said petition, on the ground, among others, that the issue on the execution of the RTC decision had been rendered moot by Aklan’s filing of a petition before the Commission on Audit (“COA”).

Aklan also filed a Petition for Certiorari with the CA to assail the denial of its Notice of Appeal. The CA dismissed said petition and ruled, among others, that Akla was estopped from invoking the doctrine of primary jurisdiction of the COA as it only raised it after Aklan’s Notice of Appeal was denied and a writ of execution was issued against it.

Issue: WON the COA have primary jurisdiction over JKCDC’s money claims against Aklan?

Held: YES. Under Commonwealth Act No. 327, as amended by Section 26 of Presidential Decree No. 1445, it is the COA, which has primary jurisdiction over money claims against government agencies and instrumentalities. Section 1 of Rule VIII and Section 1 of rule II of COA’s 2009 Revised Rules of Procedure also provide, among others, that the COA

shall have original jurisdiction over money claim against the Government, and exclusive jurisdiction over money claims due from or owing to any government agency, respectively.The doctrine of primary jurisdiction holds that if a case is such that its determination requires the expertise, specialized training and knowledge of the proper administrative bodies, relief must first be obtained in an administrative proceeding before a remedy is supplied by the courts even if the matter may well be within their proper jurisdiction. It applies where a claim is originally cognizable in the courts, and comes into play whenever enforcement of the claim requires the resolution of issues which, under a regulatory scheme, have been placed within the special competence of an administrative agency. In such a case, the court in which the claim is sought to be enforced may suspend the judicial process pending referral of such issues to the administrative body for its view or, if the parties would not be unfairly disadvantaged, dismiss the case without prejudice.JKCDC sought to enforce a claim for sums of money allegedly owed by Aklan, a local government unit. As JKCDC’s collection suit was against a local government unit, such money claim should have been first brought to the COA. The RTC should have suspended the proceedings and referred the filing of the claim before the COA.Are there exceptions to the doctrine of primary jurisdiction?There are established exceptions to the doctrine of primary jurisdiction, such as: (a) where there is estoppel on the part of the party invoking the doctrine; (b) where the challenged administrative act is patently illegal, amounting to lack of jurisdiction; (c) where there is unreasonable delay or official inaction that will irretrievably prejudice the complainant; (d) where the amount involved is relatively small so as to make the rule impractical and oppressive; (e) where the question involved is purely legal and will ultimately have to be decided by the courts of justice; (f) where judicial intervention is urgent; (g) when its application may cause great and irreparable damage; (h) where the controverted acts violate due process; (i) when the issue of non-exhaustion of administrative remedies has been rendered moot; (j) when there is no other plain, speedy and adequate remedy; (k) when strong public interest is involved; and, (l) in quo warranto proceedings.None of the foregoing circumstances are applicable to this case.Is Aklan estopped from raising the issue of jurisdiction before the CA, after the denial of its notice of appeal?

No. The doctrine of primary jurisdiction does not warrant a court to arrogate unto itself authority to resolve a controversy the jurisdiction over which is initially lodged with an administrative body of special competence. All the proceedings of the court in violation of the doctrine and all orders and decisions rendered thereby are null and void.Since a judgment rendered by a body or tribunal that has no jurisdiction over the subject matter of the case is no judgment at all, it cannot be the source of any right or the creator of any obligation. All acts pursuant to it and all claims emanating from it have no legal effect and the void judgment can never be final and any writ of execution based on it is likewise void.KJCDC’s belated compliance with the formal requirements of presenting its money claim before the COA did not cure the serious errors committed by the RTC in implementing its void decision. The RTC's orders implementing its judgment rendered without jurisdiction must be set aside because a void judgment can never be validly executed.

Boston vs. CA

64 | P a g e

Page 65: JURISDICTION-DIGESTED-OCHOA.docx

JURISDICTION CASE DIGEST TROPANG POTCHI

Facts: On 24 December 1997, petitioner filed a complaint for sum of money with a prayer for the issuance of a writ of preliminary attachment against the spouses Manuel and Lolita Toledo.6 Herein respondent filed an Answer dated 19 March 1998 but on 7 May 1998, she filed a Motion for Leave to Admit Amended Answer7 in which she alleged, among others, that her husband and co-defendant, Manuel Toledo (Manuel), is already dead.8 The death certificate9 of Manuel states "13 July 1995" as the date of death. As a result, petitioner filed a motion, dated 5 August 1999, to require respondent to disclose the heirs of Manuel.10 In compliance with the verbal order of the court during the 11 October 1999 hearing of the case, respondent submitted the required names and addresses of the heirs.11 Petitioner then filed a Motion for Substitution,12 dated 18 January 2000, praying that Manuel be substituted by his children as party-defendants. It appears that this motion was granted by the trial court in an Order dated 9 October 2000.13

Pre-trial thereafter ensued and on 18 July 2001, the trial court issued its pre-trial order containing, among others, the dates of hearing of the case.14

The trial of the case then proceeded. Herein petitioner, as plaintiff, presented its evidence and its exhibits were thereafter admitted.

On 26 May 2004, the reception of evidence for herein respondent was cancelled upon agreement of the parties. On 24 September 2004, counsel for herein respondent was given a period of fifteen days within which to file a demurrer to evidence.15 However, on 7 October 2004, respondent instead filed a motion to dismiss the complaint, citing the following as grounds: (1) that the complaint failed to implead an indispensable party or a real party in interest; hence, the case must be dismissed for failure to state a cause of action; (2) that the trial court did not acquire jurisdiction over the person of Manuel pursuant to Section 5, Rule 86 of the Revised Rules of Court; (3) that the trial court erred in ordering the substitution of the deceased Manuel by his heirs; and (4) that the court must also dismiss the case against Lolita Toledo in accordance with Section 6, Rule 86 of the Rules of Court.16

The trial court, in an Order dated 8 November 2004, denied the motion to dismiss for having been filed out of time, citing Section 1, Rule 16 of the 1997 Rules of Court which states that: "Within the time for but before filing the answer to the complaint or pleading asserting a claim, a motion to dismiss may be made x x x."17Respondent’s motion for reconsideration of the order of denial was likewise denied on the ground that "defendants’ attack on the jurisdiction of this Court is now barred by estoppel by laches" since respondent failed to raise the issue despite several chances to do so.18

Aggrieved, respondent filed a petition for certiorari with the Court of Appeals alleging that the trial court seriously erred and gravely abused its discretion in denying her motion to dismiss despite discovery, during the trial of the case, of evidence that would constitute a ground for dismissal of the case

Issue:whether or not respondent is estopped fromquestioning the jurisdiction of the trial court.

Held: At the outset, it must be here stated that, as the succeeding discussions will demonstrate, jurisdiction over the person of Manuel should not be an issue in this case. A protracted discourse on jurisdiction is, nevertheless, demanded by the fact that jurisdiction has been raised as an issue from the lower court, to the Court of Appeals and, finally, before this Court. For the sake of clarity, and in order to finally settle the controversy and fully dispose of all the issues in this case, it was deemed imperative to resolve the issue of jurisdiction.

1. Aspects of Jurisdiction

Petitioner calls attention to the fact that respondent’s motion to dismiss questioning the trial court’s jurisdiction was filed more than six years after her amended answer was filed. According to petitioner, respondent had several opportunities, at various stages of the proceedings, to assail the trial court’s jurisdiction but never did so for six straight years. Citing the doctrine laid down in the case of Tijam, et al. v. Sibonghanoy, et al.30 petitioner claimed that respondent’s failure to raise the question of jurisdiction at an earlier stage bars her from later questioning it, especially since she actively participated in the proceedings conducted by the trial court.

Petitioner’s argument is misplaced, in that, it failed to consider that the concept of jurisdiction has several aspects, namely: (1) jurisdiction over the subject matter; (2) jurisdiction over the parties; (3) jurisdiction over the issues of the case; and (4) in cases involving property, jurisdiction over the res or the thing which is the subject of the litigation.

The aspect of jurisdiction which may be barred from being assailed as a result of estoppel by laches is jurisdiction over the subject matter. Thus, in Tijam, the case relied upon by petitioner, the issue involved was the authority of the then Court of First Instance to hear a case for the collection of a sum of money in the amount of P1,908.00 which amount was, at that time, within the exclusive original jurisdiction of the municipal courts.

In subsequent cases citing the ruling of the Court in Tijam, what was likewise at issue was the jurisdiction of the trial court over the subject matter of the case. Accordingly, in Spouses Gonzaga v. Court of Appeals,32 the issue for consideration was the authority of the regional trial court to hear and decide an action for reformation of contract and damages involving a subdivision lot, it being argued therein that jurisdiction is vested in the Housing and Land Use Regulatory Board pursuant to PD 957 (The Subdivision and Condominium Buyers Protective Decree). In Lee v. Presiding Judge, MTC, Legaspi City, petitioners argued that the respondent municipal trial court had no jurisdiction over the complaint for ejectment because the issue of ownership was raised in the pleadings.

Finally, in People v. Casuga,34 accused-appellant claimed that the crime of grave slander, of which she was charged, falls within the concurrent jurisdiction of municipal courts or city courts and the then courts of first instance, and that the judgment of the court of first instance, to which she had appealed the municipal court's conviction, should be deemed null and void for want of jurisdiction as her appeal should have been filed with the Court of Appeals or the Supreme Court.

In all of these cases, the Supreme Court barred the attack on the jurisdiction of the respective courts concerned over the subject matter of the case based on estoppel by laches, declaring that parties cannot be allowed to belatedly adopt an inconsistent posture by attacking the jurisdiction of a court to which they submitted their cause voluntarily.

Here, what respondent was questioning in her motion to dismiss before the trial court was that court’s jurisdiction over the person of defendant Manuel. Thus, the principle of estoppel by laches finds no application in this case. Instead, the principles relating to jurisdiction over the person of the parties are pertinent herein.

Padlan v. Dinglasan,G.R. No. 180321March 20, 2012

FACTS:Respondent was the registered owner of a parcel of land. Whileon board a jeepney, respondent’s mother, Lilia, had a conversation with oneMaura regarding the sale of the said property. Believing that Maura was areal estate agent, Lilia borrowed the owner’s copy of the TCT fromrespondent and gave it to Maura. Maura then subdivided the property intoseveral lots. Through a falsified deed of sale, Maura was able to sell the lotsto different buyers. Maura sold one of the lots to one Lorna who sold thesame to petitioner for P4,000.00. Respondents filed a case Cancellation of Transfer Certificate of Title before the RTC. Summons was, thereafter,served to petitioner through her mother, Anita Padlan. The RTC rendered aDecision finding petitioner to be a buyer in good faith and, consequently,dismissed the complaint. The CA reversed and set aside the Decision of theRTC and ordered the cancellation of the TCT.

ISSUE:Whether the court acquired jurisdiction over the subject matter

HELD:NO.In order to determine which court has jurisdiction over theaction, an examination of the complaint is essential. Basic as a hornbook principle is that jurisdiction over the subject matter of a case is conferred bylaw and determined by the allegations in the complaint which comprise aconcise statement of the ultimate facts constituting the plaintiff's cause of action.The Court has already held that a complaint must allege the assessedvalue of the real property subject of the complaint or the interest thereon todetermine which court has jurisdiction over the action. In the case at bar, theonly basis of valuation of the subject property is the value alleged in thecomplaint that the lot was sold by Lorna to petitioner in the amountof P4,000.00.

65 | P a g e

Page 66: JURISDICTION-DIGESTED-OCHOA.docx

JURISDICTION CASE DIGEST TROPANG POTCHI

Since the amount alleged in the Complaint by respondents forthe disputed lot is only P4,000.00, the MTC and not the RTC has jurisdiction over the action. Therefore, all proceedings in the RTC are nulland void.

People vsEstrbella

Facts: After due trial, the court, rendered a decision 1 the dispositive portion reading as follows, the Court finds accused Romeo Estrebella guilty beyond reasonable doubt of the crime of Rape and hereby sentences him to suffer the penalty of reclusion perpetua, to indemnify the complaining witness, Joy Alcala y Advincula, in the sum of P30,000.00 and to pay the costs.

He argued that The trial court gravely erred in trying the case on ground of lack of jurisdiction.

Issue: Whether or not the trial court gravely erred in trying the case on ground of lack of jurisdiction.

Held: Appellant's argument holds no water.

It is of course well-settled that jurisdiction over the subject matter of an action—in this case the crime of rape—is and may be conferred only by law, and that jurisdiction over a given crime not vested by law upon a particular court, may not be conferred thereon by the parties involved in the offense. (Manila Railroad v. Atty. General, 20 Phil. 523; Perkins v. Roxas, 72 Phil. 514, cited in Valdepenas vs. People, 16 SCRA 871). But the aforementioned provision of Art. 344 does not determine the jurisdiction of our courts over the offenses therein enumerated. It could not affect said jurisdiction, because the same with respect to the instant crime is governed by the Judiciary Act of 1948, not by the Revised Penal Code, which deals primarily with the definition of crimes and the factors pertinent to the punishment of the culprits. The complaint required in said Art. 344 is merely a condition precedent to the exercise by the proper authorities of the power to prosecute the guilty parties. And such condition has been imposed out of consideration for the offended woman and her family who might prefer to suffer the outrage in silence rather than go through with the scandal of a public trial. (Samilin v. Court of First Instance of Pangasinan, 57 Phil. 298, 304, cited in Valdepenas v. People, supra)

In the case at bar, while the complaint may have been technically in the sense that complainant was incompetent, this defect has been cured when complainant's brother Fernando Alcala took the witness stand for the prosecution. The brother's testimony shows the consent and willingness of the family of complainant, who can not give her consent obviously, to have the private offense committed against the latter publicly tried. Substantially, this is what is required by the rules. Evidently, by undergoing trial, the family of complainant chose to denounce the injustice committed against the latter in public and thus agreed to bear the personal effects of said exposure. Undoubtedly, therefore, the trial court had jurisdiction to try the case.

66 | P a g e